由买买提看人间百态

boards

本页内容为未名空间相应帖子的节选和存档,一周内的贴子最多显示50字,超过一周显示500字 访问原贴
Parenting版 - WISC-V 智商测试
相关主题
老师说我女儿是gifted kid (转载)9岁, 如何报名考amc10
忍不住上来问问,我娃的数学是啥水平(6岁)?求科普: 奥数 vs. Math Olympiad vs. Math Count vs. Math circle vs. 超前学数学
陶天才论天才 (转载)有明天考AMC 8的吗?
NIH 证实天才和疯子都源于同一型基因:DARPP-32算不算有数学天分
放养妈进来看看大家怎么推孩子的弱项?
最新研究证明心算AIME大幅度促进心理和情感健康少年智力开发
来看一下各位的观点[转载] 为什么说usamo 简单
是不是任我儿子继续迷象棋二年级女儿的report
相关话题的讨论汇总
话题: memory话题: chunking话题: working话题: gt话题: temporal
进入Parenting版参与讨论
1 (共1页)
t*******s
发帖数: 48
1
申请的学校需要提供这个测试,10岁男生。Full Scale IQ=146, GAI =152. 我怀疑这
个数字的可信度,为什么一些学校需要这个?有经验的父母说说吧
T********s
发帖数: 100
2
146 千分之一了吧,算天才儿童了,恭喜。
ps 是自费测的吗?多少钱?
t*******s
发帖数: 48
3
自费600,保险不能用。

【在 T********s 的大作中提到】
: 146 千分之一了吧,算天才儿童了,恭喜。
: ps 是自费测的吗?多少钱?

s***n
发帖数: 1280
4
很多GT/Magnet学校/班看这些智商测试。还有些好私校,也会参考这些测试。对智商中
上的学生来说,美国这边学校成绩主要体现学生听话程度,看不太出智力水平,所以很
多GT/Magnet要这些测试.

【在 t*******s 的大作中提到】
: 申请的学校需要提供这个测试,10岁男生。Full Scale IQ=146, GAI =152. 我怀疑这
: 个数字的可信度,为什么一些学校需要这个?有经验的父母说说吧

T********s
发帖数: 100
5
有些Aspies 其实数学,艺术极有天赋的,但是由于他们特有的毛病,智商测试总分可
能很差劲,如果按照135的标准一刀切,那些GT/Magnet学校/班都不够格进吧?
从哪里能看出智力水平呢?

【在 s***n 的大作中提到】
: 很多GT/Magnet学校/班看这些智商测试。还有些好私校,也会参考这些测试。对智商中
: 上的学生来说,美国这边学校成绩主要体现学生听话程度,看不太出智力水平,所以很
: 多GT/Magnet要这些测试.

s***n
发帖数: 1280
6
这里是家长聊天,所以大家说个总分。真正的智商测试是有几个小项,比如WISC-V,会
考核语言理解能力,视觉空间分析能力,推理能力,工作记忆能力,进程处理速度等几
个小项。好的GT program招生会看这些小项,不会只看总分。比如我们这的初中HGT
program是文理分科。理科部招生的时候就看重的是逻辑推理,空间分析方面的小分,
而不是总分。

【在 T********s 的大作中提到】
: 有些Aspies 其实数学,艺术极有天赋的,但是由于他们特有的毛病,智商测试总分可
: 能很差劲,如果按照135的标准一刀切,那些GT/Magnet学校/班都不够格进吧?
: 从哪里能看出智力水平呢?

t******l
发帖数: 10908
7
不管总分还是小分,底线是 “phonics 勉强过关” + “九九乘法表也背不顺溜” 的
娃就不要去测啥 WISC-V 了,丢人又现眼。。。
t******l
发帖数: 10908
8
“比如WISC-V,会考核语言理解能力,视觉空间分析能力,推理能力,工作记忆能力,
进程处理速度等等”
我突然有了一种森森闯入英特尔在马来西亚的芯片封装及硬件测试车间的幻觉。。。
s***n
发帖数: 1280
9
如果你不是开玩笑的话,那我只能说你对智商测试不了解,看问题的出发点也有问题。
智商测试不是为了给家长挣脸,而是为了更好地了解孩子。

【在 t******l 的大作中提到】
: 不管总分还是小分,底线是 “phonics 勉强过关” + “九九乘法表也背不顺溜” 的
: 娃就不要去测啥 WISC-V 了,丢人又现眼。。。

t******l
发帖数: 10908
10
认真的说,我觉得看具体情况。
我个人觉得智商测试,对于没有特别的凸显能力,同时适应学校生活也没有障碍的普通
娃,我觉得没有任何必要。
这是因为除了学校的各种测试以外,每个娃实际上还每年被学校不同的老师 de facto
的 “图灵测试”。
而且普通娃在小学阶段,基础阅读和基础算术,外加 free play 和 playground 各种
集体活动啥的,基本都时间不够用了。一般连学校老师的建议都做不完,只能按优先级
能办多少是多少。我确实看不出来对于普通娃,有用智商测试来 override 学校老师建
议的需要。

:如果你不是开玩笑的话,那我只能说你对智商测试不了解,看问题的出发点也有问题
。智商测试不是为了给家长挣脸,而是为了更好地了解孩子。
相关主题
最新研究证明心算AIME大幅度促进心理和情感健康9岁, 如何报名考amc10
来看一下各位的观点求科普: 奥数 vs. Math Olympiad vs. Math Count vs. Math circle vs. 超前学数学
是不是任我儿子继续迷象棋有明天考AMC 8的吗?
进入Parenting版参与讨论
t******l
发帖数: 10908
11
当然对于普通娃,我其实不反对学校老师从专职教师的角度 order 智商测试,作为评
判的信息之一。
但对于普通娃家长而言,虽然学校老师存在不上心,甚至政治正确而 conflict of
interest 的可能。但是 neuro-typical 的家长的 self-awareness,对自家娃测试结
果的 “大脑自我修改记忆型误读”,并不见得比学校至少中性立场的老师,会更少的
误读。

:认真的说,我觉得看具体情况。
t******l
发帖数: 10908
12
或者举个小娃更实际的例子,小学低年级最最重要的一件事,定当前 reading level。
这个学校老师直接就给了建议,我估计基本就是根据 reading level 的测试。
然后好玩得是,我家小娃更喜欢图书馆几本内容有趣的书,但是但是娃你有没有搞错,
那些是 lexile 700L 左右的书啊啊啊。不过古人云,不撞南墙不回头。为了不打击娃
的积极性,我只好直说,这些是 lexile 700L 左右了,不过只要你有兴趣,我陪你查
online 字典 Google 照片。于是小娃读那两本 lexile 700L 左右的书,读了四五天后
读得那个昏天黑地啊,终于意识到这磕磕碰碰的实在太特么难。于是回头读老师建议的
reading level,一会儿就一本读下来。虽然内容相对而言稍微无趣那么一点点,但毕
竟有成就感不是?
当然我上面说的是普通型娃,普通 reading test 就够了。对于超常 GT 型,或者
hyperlexia 那些,那普通大路货型 reading level 测试估计真心不行。这种确实要上
专门的智商测试。

:当然对于普通娃,我其实不反对学校老师从专职教师的角度 order 智商测试,作为评
:判的信息之一。
h*****m
发帖数: 1034
13
审题很重要啊,同学。
人家楼主明明说的是申请的学校要求做这个测试的,分明是老师要用这个信息做决定给
建议,哪里有提到“用智商测试来 override 学校老师建议”?
下笔千言离题万里,你确定小时候语文不是一屁股叉叉?

facto

【在 t******l 的大作中提到】
: 认真的说,我觉得看具体情况。
: 我个人觉得智商测试,对于没有特别的凸显能力,同时适应学校生活也没有障碍的普通
: 娃,我觉得没有任何必要。
: 这是因为除了学校的各种测试以外,每个娃实际上还每年被学校不同的老师 de facto
: 的 “图灵测试”。
: 而且普通娃在小学阶段,基础阅读和基础算术,外加 free play 和 playground 各种
: 集体活动啥的,基本都时间不够用了。一般连学校老师的建议都做不完,只能按优先级
: 能办多少是多少。我确实看不出来对于普通娃,有用智商测试来 override 学校老师建
: 议的需要。
:

t******l
发帖数: 10908
14
那种要智商测试的申请学校的,说白了都不是普通学校普通娃。
如果老师要自己衡量一下自己看看,老师自己 order 一套模拟题测测就完事了,也不
用搞那么正式。
我就是跟帖说这些智商测试,跟普通学校普通娃都没啥关系。

:审题很重要啊,同学。
:人家楼主明明说的是申请的学校要求做这个测试的,分明是老师要用这个信息做决定
给建议,哪里有提到“用智商测试来 override 学校老师建议”?
t******l
发帖数: 10908
15
而且申请学校需要测试,这种显然不是为了提供反馈的目的。这种显然就是为了保证制
度公平。
这就好比如果仅仅要知道自己英语水平,不用花 600 快钱去考个出国申请学校用的托
福。

:审题很重要啊,同学。
:人家楼主明明说的是申请的学校要求做这个测试的,分明是老师要用这个信息做决定
给建议,哪里有提到“用智商测试来 override 学校老师建议”?
c***x
发帖数: 1826
16

挺好,挺好,黄药师终于回来了,周伯通再也无需左右互搏了 :-)
T********s
发帖数: 100
17
这样听起来比较合理。
还有一个测试斯坦福比奈,也是按照这几个小项分类测的吗?

【在 s***n 的大作中提到】
: 这里是家长聊天,所以大家说个总分。真正的智商测试是有几个小项,比如WISC-V,会
: 考核语言理解能力,视觉空间分析能力,推理能力,工作记忆能力,进程处理速度等几
: 个小项。好的GT program招生会看这些小项,不会只看总分。比如我们这的初中HGT
: program是文理分科。理科部招生的时候就看重的是逻辑推理,空间分析方面的小分,
: 而不是总分。

t*******s
发帖数: 48
18
仔细再看了看报告,的确是测5个部分。每部分包含两个单项。分值如下。仔细想想,
part 4 的确符合家长多年观察和从前老师的各种抱怨。其他分值高的部分,家长没明
显觉得他本身match 这些分值。希望这里给各个家长一个借鉴
Part 1: Verbal Comprehension ( Similarities & Vocabulary) - 155/>99.9%
Part 2: Visual Spatial (Block Design & Visual Puzzles) – 138/99%
Part 3: Fluid Reasoning (Matrix Reasoning & Figure Weights) – 144/8
Part 4: Working Memory (Digit Span & Picture Span) – 107/68%
Part 5: Processing Speed (Symbol Search & Coding) – 129/97%

【在 s***n 的大作中提到】
: 这里是家长聊天,所以大家说个总分。真正的智商测试是有几个小项,比如WISC-V,会
: 考核语言理解能力,视觉空间分析能力,推理能力,工作记忆能力,进程处理速度等几
: 个小项。好的GT program招生会看这些小项,不会只看总分。比如我们这的初中HGT
: program是文理分科。理科部招生的时候就看重的是逻辑推理,空间分析方面的小分,
: 而不是总分。

t******l
发帖数: 10908
19
马克思说过,海蜇头以不同的角度解释世界,但更重要的是改变世界。
而从这个角度说,比方就说这个 working memory 的测试结果,我真心觉得对普通娃的
实践指导意义不大。
举个例子,跟 reading level 的测试比。reading level 的测试决定选书的 text
complexity 范围。但 working memory 这一项,普通娃咋个改善法?
再进一步,如果 reading level 测试结果不准确,那在阅读图书馆书籍的时候,娃和
大人自然会知道(特殊情况比如 dyslexia / hyperlexia 例外)。所以能够过滤掉
false positive / false negative。但 working memory 测试万一是 false positive
/ false negative,这个真心不好过滤。

:仔细再看了看报告,的确是测5个部分。每部分包含两个单项。分值如下。仔细想想,
:part 4 的确符合家长多年观察和从前老师的各种抱怨。其他分值高的部分,家长没明
t******l
发帖数: 10908
20
这个 working memory 测试的 false positive / negative 的原因,是因为实践中的
working memory 的效率,是 largely rely on chunking / encoding 的;而
chunking / encoding 又 largely rely on schema,以及 perceptual memory 等等其
他的 implicit memory。
举了例子就是 super fast reader 不一定就是 working memory 的 the magic # 7 本
身多几个 plus 2,更有可能的是 superior on context-based predict / sight
words(比如阿汤哥的电影,第一秒就 predict 阿汤哥死不了,哪有导演让自己挂的不
是?),甚至可能 context-based predict / sight larger linguistic structure。
而从这点说,隔壁楼的背诵九九乘法表型 flash card 记忆词汇,在 reading speed
方面,就有相当的劣势。因为 acquiring 的环境跟 applying 的环境差别更大,导致
more effective acquiring 不一定 end up more effective applying in long run。
这也是 Bloom's Taxonomy 的理论根本弱点。
当然,反过来过,acquire through reading 的 vocabulary,经常也会发生看报纸认
字没问题,但是没报纸时常常 spelling 错得一塌糊涂。。。俗称:我是高小文盲,但
我 neuro-typical 不是?

:马克思说过,海蜇头以不同的角度解释世界,但更重要的是改变世界。
相关主题
算不算有数学天分[转载] 为什么说usamo 简单
大家怎么推孩子的弱项?二年级女儿的report
少年智力开发觉得美帝数学进度慢的进来看一下
进入Parenting版参与讨论
t******l
发帖数: 10908
21
或者说,军医学的简化模型的思想的大方向并不错。
但是军医学有可能出的问题,就好比马工系的开会型老板,时而不时冒出来一句比如:
你们 conjugated gradient solver 能不能先把五百万 x 变量们统统先解完到精确值
,然后回过头来再解五百万 y 变量。多么 clean 的解决方案不是?

:这个 working memory 测试的 false positive / negative 的原因,是因为实践中
的 working memory 的效率,是 largely rely on chunking / encoding 的;而
:chunking / encoding 又 largely rely on schema,以及 perceptual memory 等等
其他的 implicit memory。
t******l
发帖数: 10908
22
当然实际上开会型老板一般也不会这么玩。一般就话说三成,然后 theory of mind 一
下 engineer,然后开个玩笑就完事了。人本职工作就是协调平衡抹稀泥争议。

:或者说,军医学的简化模型的思想的大方向并不错。
t******l
发帖数: 10908
23
如果大伙儿对 WSIC 智商测试还抱有幻想的话,看一看下面的 link 好了,我保证大伙
儿一定有恍然走进英特尔在马来西亚的硬件测试车间的感觉。
http://www.thinktonight.com/WISC_IV_subtests_s/331.htm
或者 frankly 的说吧,我觉得这些测试在及格以后,高分还是低分,真不一定比能不
能阅读 lexile 1600L+ 的书本,或者能不能轻松心算 AIME 15 道填空题,有更多的说
服力。
t******l
发帖数: 10908
24
或者更进一步好了。下面的这个智商测试如果做慢或者做的磕磕碰碰,对解 “球面上
每一圈上升一米,求路径总长度” 的题目,有没有哪怕特么一点点影响。
或者这个寓言的 moral story 就是:
(1)再复杂的 2-D 能力,不一定跟最最简单的 3-D 和 high-dimension 的能力,有
那么一点点相关性。
(2)再复杂的 point-point 的 perception,不一定对最最简单的 point-slope 的
perception(一阶导数)的能力,有那么一点点相关性。
诚然,每个娃的 perception 的成长过程都是从 2-D / 3-D 到 high-dimension,从
point-point 到 point-slope 到 slope-on-slope。但是:
(1)这不是说一定要成为 2-D 三八红旗手阿姨,实际上 2-D 三八红旗手阿姨最后多
半 end-up 在猴子,8 billion 猴子。
(2)大脑在掌握 high-order thinking skill 以后,为了防止自己成为金三胖的体型
而 neuro-atypical,不一定保留 low-order thinking skill,可能会 prune 掉大部
分无用的 low order thinking 能力,低碳环保不是?或者从进化优势的角度,这就是
为啥大猩猩有 photographic memory,而智人不再有 photographic memory 的原因。

【在 t******l 的大作中提到】
: 如果大伙儿对 WSIC 智商测试还抱有幻想的话,看一看下面的 link 好了,我保证大伙
: 儿一定有恍然走进英特尔在马来西亚的硬件测试车间的感觉。
: http://www.thinktonight.com/WISC_IV_subtests_s/331.htm
: 或者 frankly 的说吧,我觉得这些测试在及格以后,高分还是低分,真不一定比能不
: 能阅读 lexile 1600L+ 的书本,或者能不能轻松心算 AIME 15 道填空题,有更多的说
: 服力。

c*********2
发帖数: 2752
25
测这些没什么意思吧,说明不了啥啊
s***n
发帖数: 1280
26
新版本的斯坦福比奈测试会测小项。潮流。

【在 T********s 的大作中提到】
: 这样听起来比较合理。
: 还有一个测试斯坦福比奈,也是按照这几个小项分类测的吗?

s***n
发帖数: 1280
27
我觉得你孩子是或接近是highly gifted。不过他的working memory分数一般,所以他
的天分可能没有在学习和日常生活中表现出来。由于working memory和其它能力的不平
衡,他可能会有些类似多动的行为问题或注意力不容易集中的情况,但应该不严重。
后面是乱猜的,不知准不准。

65533;.8

【在 t*******s 的大作中提到】
: 仔细再看了看报告,的确是测5个部分。每部分包含两个单项。分值如下。仔细想想,
: part 4 的确符合家长多年观察和从前老师的各种抱怨。其他分值高的部分,家长没明
: 显觉得他本身match 这些分值。希望这里给各个家长一个借鉴
: Part 1: Verbal Comprehension ( Similarities & Vocabulary) - 155/>99.9%
: Part 2: Visual Spatial (Block Design & Visual Puzzles) – 138/99%
: Part 3: Fluid Reasoning (Matrix Reasoning & Figure Weights) – 144/8
: Part 4: Working Memory (Digit Span & Picture Span) – 107/68%
: Part 5: Processing Speed (Symbol Search & Coding) – 129/97%

t******l
发帖数: 10908
28
我觉得小孩如果其他方面都是表现正常的话,(外加父母对追藤兴趣不大的话),从网
上的样题来看,我真心觉得可以直接忽略那个 working memory 的分数。(毕竟是军医
学的打分,而不是医学的血检结果)。
原因就是那个 working memory 的测试,非常简单,非常 explicit-memory-wise。比
如老师说 12837, 138273482, 3819378492823, 1903849290849283 四个数字,然后娃
要在三秒钟内 reverse order 回答出正确答案 3829480929483091, 3282948739183,
284372831, 73821 这种。
我觉得第一个问题是,娃娃为啥要 cooperate 这么沙比的测试。如果军医学认为这个
问题不需要回答,那图灵说,图灵测试已经挂了。
比如我家小娃也有个行为问题,就是老师要求有个 empty folder tar-in tar-out 的
task 她老是不干。老师说过一次以后,她还是连续三天都忘了。然后我那天早上问他
咋地又忘了。小娃想了想,回答说:“There is nothing in the folder, you know”。
但如果觉得小娃是故意的,maybe not。大脑会自动 prioritize task,属于 implicit
-memory / 潜意识控制的玩意儿。你永远不知道小娃晚上读完书以后,上学路上心算计
算题以后,白天时大脑会不会在 implicit-memory / 潜意识里,是不是继续使用空闲
时间片后台处理那些新获得的信息。
总之我的看法就是,如果娃看起来正常融入学校没有问题的话,“一脚踹飞上帝” 的
活父母就别太热心了。鲁迅同志说:“你永远不知道一脚踹飞洗脚水的时候,是不是把
水盆里的娃娃也给一起踹飞了”。而洋人曰:“Nature usually has its own course
of action”。当然,有志于追藤的,另说。
t*******s
发帖数: 48
29
嗯,前面所有年级的老师都有抱怨他注意力不集中。比如说几个reading advanced 的
孩子一起读故事,读完后老师让大家重复故事内容或重复主人公名字,大部分情况下他
都记不住。多动症倒是没明显表现出来。6岁时老师多次建议我们带他看心理医生,认
为他有ADHD。看过几次后没啥收效就放弃了。总之6岁到9岁在学校里属于经常给老师
reduce recess time的孩子。

【在 s***n 的大作中提到】
: 我觉得你孩子是或接近是highly gifted。不过他的working memory分数一般,所以他
: 的天分可能没有在学习和日常生活中表现出来。由于working memory和其它能力的不平
: 衡,他可能会有些类似多动的行为问题或注意力不容易集中的情况,但应该不严重。
: 后面是乱猜的,不知准不准。
:
: 65533;.8

s***n
发帖数: 1280
30
智商测试对很多孩子的教育有一定的指导意义,其中比较显著的就是在"普通"娃中发现
GT娃。
我前面提到好的GT program招生会参考智商测试。言外之意就是不好的GT program用学
习成绩或老师推荐来挑GT娃。
美国这边公校学习要求比较低,总评成绩也轻考试重平时练习。很多普通智商偏上的孩
子凭着努力或良好的学习习惯学习自觉性就能取得很好地成绩。最适合这些孩子的是普
通学校的教学方式和进度。如果因为这些孩子成绩好,就把他们拉入GT班的话,反而会
让他们因为不适应GT教学方式和进度学习成绩下降。
另一方面也有很多GT孩子,由于种种原因学习成绩一般。或者因为学习习惯不好,或者
因为贪玩学习自觉性不强,或者因为学习兴趣不高,或者因为twice exceptional。对
于这些孩子,智商测试可以帮助家长和老师发现他们的天分,更好地了解他们的特点,
找到更适合他们的教学方式和进度。
通过老师来发现GT娃,也很不靠谱。绝大多数小学老师没有判定GT娃的能力。他们往往
通过学习成绩 + 个人喜好来做判定,这样做出的评判很多时候还不如单看成绩。有些
老师会通过学生掌握新知识的能力来看天分,但学生这方面能力的表现受很多因素的影
响,比如孩子内向外向,推没推过,有没有兴趣等等。老师推荐最大的问题和看成绩选
拔一样 - 他们很难发现那些因为某些因素学习成绩不突出天分被埋没的GT娃。

facto

【在 t******l 的大作中提到】
: 认真的说,我觉得看具体情况。
: 我个人觉得智商测试,对于没有特别的凸显能力,同时适应学校生活也没有障碍的普通
: 娃,我觉得没有任何必要。
: 这是因为除了学校的各种测试以外,每个娃实际上还每年被学校不同的老师 de facto
: 的 “图灵测试”。
: 而且普通娃在小学阶段,基础阅读和基础算术,外加 free play 和 playground 各种
: 集体活动啥的,基本都时间不够用了。一般连学校老师的建议都做不完,只能按优先级
: 能办多少是多少。我确实看不出来对于普通娃,有用智商测试来 override 学校老师建
: 议的需要。
:

相关主题
分数应用题忍不住上来问问,我娃的数学是啥水平(6岁)?
我招,我是猪陶天才论天才 (转载)
老师说我女儿是gifted kid (转载)NIH 证实天才和疯子都源于同一型基因:DARPP-32
进入Parenting版参与讨论
s***n
发帖数: 1280
31
不少Gifted的孩子,特别是Highly Gifted孩子,在学龄期会出现类似多动或注意力缺
乏症的症状。很多时候这些症状是常年上课无聊乏味造成的。这些孩子只是假性ADHD,
处理得当,症状会随着年龄增长减轻消失。让这些孩子进入GT或HGT的program,也会帮
助这些孩子走出假性ADHD。
不过也有些GT娃确实有ADHD或ADD,是twice exceptional娃。这个真假问题,就需要家
长去细心观察和判断了。
我见过很多GT娃,也了解很多GT娃成长的过程。我觉得你的孩子很像是真GT娃。你可以
尝试让他去上一些难一点或进度快一点的课外班补习班,然后看孩子的接受程度和表现
。另外,你可以有意地训练他的working memory,会对学习和减轻症状都有帮助。虽然
你孩子多半是假ADHD/ADD,但一些针对ADHD/ADD的非药物辅助疗法你也可以考虑。
还提醒一点,这些有真假ADHD的孩子他们取得的成绩和他们能力不匹配。常此以往会打
击他们的自信心和自我认可,甚至出现心理问题。家长要和孩子多交心,多正面鼓励。

【在 t*******s 的大作中提到】
: 嗯,前面所有年级的老师都有抱怨他注意力不集中。比如说几个reading advanced 的
: 孩子一起读故事,读完后老师让大家重复故事内容或重复主人公名字,大部分情况下他
: 都记不住。多动症倒是没明显表现出来。6岁时老师多次建议我们带他看心理医生,认
: 为他有ADHD。看过几次后没啥收效就放弃了。总之6岁到9岁在学校里属于经常给老师
: reduce recess time的孩子。

t******l
发帖数: 10908
32
如果是 twice exceptional,我同意智商测试有非常大的意义。
如果是 Highly Gifted,我也同意智商测试有非常大的意义。
如果是普通娃里找 Gifted,有啥意义啊。普通娃的日子不是很开心?

:智商测试对很多孩子的教育有一定的指导意义,其中比较显著的就是在"普通&#
34;娃中发现GT娃。
R*2
发帖数: 225
33
噪音一下
从不做这种测试,自己不做,孩子也不做,怕万一测出是傻子影响后半辈子的心情和乐
趣。
t*******s
发帖数: 48
34
都觉得你是专家啦!不是申请的学校要这个,我都对IQ 一无所知。我几年来一直非常
讨厌见老师,听到的老是负面的各种评价。后来就随他自己在YouTube 上乱看一些科技
类的东西,这些都是多数大人不感兴趣的莫名其妙video. 有一整年天天在看human
body Wikipedia 厚书,里面贴满了各种铅笔写的粘贴纸,再后来就自己在khan
academy 网上学数学,大概8岁做完这个网上7年级的数学。我研究了上面的数学,很普
通的短video,没有什么独到之处,我觉得他不是对数学感兴趣,他唯一的动力就是如何
快速做题这样能collect 各种花样的badge。最近一年他在AoPS上学algebra, 这是我见
过最最无聊的一个学习网站,既没声音,也没video。上面一页一页的讲解翻的及其快
。每周7天,3点放学后全部在冰球队游泳队,喜欢钢琴和长号,周末在一个音乐学院的
玄乐队联系。至今没发现他在体育音乐上有任何潜能,只能说他非常喜欢,很有
passion 做这些。总而言之,我没法把他和这个所谓的“gifted” match 起来

【在 s***n 的大作中提到】
: 不少Gifted的孩子,特别是Highly Gifted孩子,在学龄期会出现类似多动或注意力缺
: 乏症的症状。很多时候这些症状是常年上课无聊乏味造成的。这些孩子只是假性ADHD,
: 处理得当,症状会随着年龄增长减轻消失。让这些孩子进入GT或HGT的program,也会帮
: 助这些孩子走出假性ADHD。
: 不过也有些GT娃确实有ADHD或ADD,是twice exceptional娃。这个真假问题,就需要家
: 长去细心观察和判断了。
: 我见过很多GT娃,也了解很多GT娃成长的过程。我觉得你的孩子很像是真GT娃。你可以
: 尝试让他去上一些难一点或进度快一点的课外班补习班,然后看孩子的接受程度和表现
: 。另外,你可以有意地训练他的working memory,会对学习和减轻症状都有帮助。虽然
: 你孩子多半是假ADHD/ADD,但一些针对ADHD/ADD的非药物辅助疗法你也可以考虑。

h*****m
发帖数: 1034
35
对智商测试这些一窍不通,但总觉得那么多心理学家认知学家这么多年建立起来的成系
统的一套方法,总不会是一无是处的吧。
对于一个个体,或许结果不一定准确,但从统计意义上,一群平均智商为130的人,应
该会比一群平均智商为100的人,学习能力更强,潜力更大。我想这也是为什么好多学
校用这个作为录取指标的原因。
至于某一个特定学生的家长会不会被这个东西误导,自己在家瞎搞,那就说不好了。

【在 t******l 的大作中提到】
: 如果大伙儿对 WSIC 智商测试还抱有幻想的话,看一看下面的 link 好了,我保证大伙
: 儿一定有恍然走进英特尔在马来西亚的硬件测试车间的感觉。
: http://www.thinktonight.com/WISC_IV_subtests_s/331.htm
: 或者 frankly 的说吧,我觉得这些测试在及格以后,高分还是低分,真不一定比能不
: 能阅读 lexile 1600L+ 的书本,或者能不能轻松心算 AIME 15 道填空题,有更多的说
: 服力。

h*****m
发帖数: 1034
36
注意“普通”是加了引号的,就是貌似“普通”容易被老师家长忽略其GT潜质的“天才
娃”。
比如说你总说你家小娃“普通”,但说不定在某些方面有GT呢。
当然她有你这么一位认知学理论极为丰富的爸爸,想来有GT潜质的话,总会被你慧眼识
出。但绝大多数娃不是没这条件么,学校搞个智商测试,把有GT潜质的娃挑出来,因材
施教,对他们才公平啊。

【在 t******l 的大作中提到】
: 如果是 twice exceptional,我同意智商测试有非常大的意义。
: 如果是 Highly Gifted,我也同意智商测试有非常大的意义。
: 如果是普通娃里找 Gifted,有啥意义啊。普通娃的日子不是很开心?
:
: :智商测试对很多孩子的教育有一定的指导意义,其中比较显著的就是在"普通&#
: 34;娃中发现GT娃。

t******l
发帖数: 10908
37
你应该试试 non-fiction。我家大娃小学高年级的 reading level 是用 non-fiction
过关的。

:都觉得你是专家啦!不是申请的学校要这个,我都对IQ 一无所知。我几年来一直非常
:讨厌见老师,听到的老是负面的各种评价。后来就随他自己在YouTube 上乱看一些科
技类的东西,这些都是多数大人不感兴趣的莫名其妙video. 有一整年天天在看human
t******l
发帖数: 10908
38
我看了这段,虽然我不懂 GT,但我个人觉得你娃确实是 Gifted,应该测一下智商然后
找合适的 GT 班。

:都觉得你是专家啦!不是申请的学校要这个,我都对IQ 一无所知。我几年来一直非常
:讨厌见老师,听到的老是负面的各种评价。后来就随他自己在YouTube 上乱看一些科
技类的东西,这些都是多数大人不感兴趣的莫名其妙video. 有一整年天天在看human
t******l
发帖数: 10908
39
你看过楼主楼上最新的描述的话,那我家小娃绝对是普通娃,真心的说。这个 gap 非
常大。
而且我对自己小时候还是有一些记忆的,我们家确实都是普通娃。虽然也是有些
annoying 的,但跟楼主相比那根本就不是一个量级的事。

:注意“普通”是加了引号的,就是貌似“普通”容易被老师家长忽略其GT潜质的“天
才娃”。
:比如说你总说你家小娃“普通”,但说不定在某些方面有GT呢。
t******l
发帖数: 10908
40
我这么说吧,如果 working memory 很差,AMC 8 基本不可能行,AMC 10 更不可能玩。
除非实在是天赋异禀,否则的话我多半认为智商测试的 working memory,那项测错了
。你娃是 false negative。加上其他项你娃就是 Highly Gifted 或者 Profoundly
Gifted。

:都觉得你是专家啦!不是申请的学校要这个,我都对IQ 一无所知。我几年来一直非常
:讨厌见老师,听到的老是负面的各种评价。后来就随他自己在YouTube 上乱看一些科
技类的东西,这些都是多数大人不感兴趣的莫名其妙video. 有一整年天天在看human
相关主题
NIH 证实天才和疯子都源于同一型基因:DARPP-32来看一下各位的观点
放养妈进来看看是不是任我儿子继续迷象棋
最新研究证明心算AIME大幅度促进心理和情感健康9岁, 如何报名考amc10
进入Parenting版参与讨论
t******l
发帖数: 10908
41
“重复故事内容或重复主人公名字”
这个首先取决于兴趣而不是 working memory,就好比能把韩剧里人名搞得一清二楚的
,估计是心理系的比较多,马工系多半不行。你懂的。
所以这个不一定是 working memory,可能就是潜意识心理层次的不合作。
而对于 non-fiction 娃,基本对小学版 lexile 800+ 的 fiction 不太会感兴趣,因
为基本是唠嗑。
而实际上在小学阶段,non-fiction 比 fiction 更难,即使是差不多的 lexile。这个
图书管理员就知道,我带小娃去图书馆去问哪里找老师建议的 reading level 的时候
,图书管理员一看,二话不说就直接带到 fiction 书架介绍图书馆为娃娃们的 color
tag。
然后我就怯生生地问了一句 non-fiction? 图书管理员说 non-fiction 按 topic 分类
,但也有娃娃的 color tag,然后带小娃到她喜欢的 topic,但离开以前想了想,留下
一句说 non-fiction 有可能要降一级 color tag,让我们自己掌握。
初中 teen 心理活动复杂以后另说。

:嗯,前面所有年级的老师都有抱怨他注意力不集中。比如说几个reading advanced 的
:孩子一起读故事,读完后老师让大家重复故事内容或重复主人公名字,大部分情况下
他都记不住。多动症倒是没明显表现出来。6岁时老师多次建议我们带他看心理医生,认
t******l
发帖数: 10908
42
统计的数字是对 “一将功成万骨枯” 而言的。
对薛定谔的炮灰猫而言,重要的是量子崩塌最后是不是崩在炮灰态。

:对智商测试这些一窍不通,但总觉得那么多心理学家认知学家这么多年建立起来的成
系统的一套方法,总不会是一无是处的吧。
t*******s
发帖数: 48
43
这里你提到reading, 这是唯一我能在他身上看到比较闪亮的与academy 有关的长处,
我觉得非常好。但是为什么像SSAT 或ISEE 这种考试,虽说分数还不错,但是他并没有
像其他牛娃孩子能做到全对或基本全对。他空余时间除了video Game 外,哪怕只有一
秒钟,他都在读,即便是视野内只有一个瓶子,他就马上读瓶子上的商标。总之这一秒
钟似乎必须保证有事情干

fiction
非常

【在 t******l 的大作中提到】
: 你应该试试 non-fiction。我家大娃小学高年级的 reading level 是用 non-fiction
: 过关的。
:
: :都觉得你是专家啦!不是申请的学校要这个,我都对IQ 一无所知。我几年来一直非常
: :讨厌见老师,听到的老是负面的各种评价。后来就随他自己在YouTube 上乱看一些科
: 技类的东西,这些都是多数大人不感兴趣的莫名其妙video. 有一整年天天在看human

t*******s
发帖数: 48
44
我大概就是因为他不是AMC的料,所以觉得他和gifted不沾边。这是我对测试分数觉得
震惊的一个主要原因

玩。
非常

【在 t******l 的大作中提到】
: 我这么说吧,如果 working memory 很差,AMC 8 基本不可能行,AMC 10 更不可能玩。
: 除非实在是天赋异禀,否则的话我多半认为智商测试的 working memory,那项测错了
: 。你娃是 false negative。加上其他项你娃就是 Highly Gifted 或者 Profoundly
: Gifted。
:
: :都觉得你是专家啦!不是申请的学校要这个,我都对IQ 一无所知。我几年来一直非常
: :讨厌见老师,听到的老是负面的各种评价。后来就随他自己在YouTube 上乱看一些科
: 技类的东西,这些都是多数大人不感兴趣的莫名其妙video. 有一整年天天在看human

t******l
发帖数: 10908
45
你的要求比较高,普通娃一般也不跟全对的牛蛙比。
但你娃确实跟普通娃有些不同好像。普通娃对瓶子标签之类的一般不会感兴趣。

:这里你提到reading, 这是唯一我能在他身上看到比较闪亮的与academy 有关的长处,
:我觉得非常好。但是为什么像SSAT 或ISEE 这种考试,虽说分数还不错,但是他并没
有像其他牛娃孩子能做到全对或基本全对。他空余时间除了video Game 外,哪怕只有一
t******l
发帖数: 10908
46
我觉得 AMC,尤其是 AMC 10 & above,比智商测试的 working memory 更有说服力。
我觉得你的智商测试的 working memory 的分数有问题,那个不足为参考依据。
但另一方面,虽然我家两娃在学校都有很 annoying 的地方,比如大娃过去三年级时,
把老师给的书粗粗看一眼就递回去说不感兴趣。小娃甚至一年级老师就说她连书页都不
肯翻,有时老师给她翻页。但这些都是细节,其实各种娃都或多或少可能都会有一些。
而在大方向上,我家两娃从来没有学校层次能拿得出手的任何 complaint,我说的,都
是跟老师 1:1 相互侃大山而已。
所以对于你这种情况,你觉得不爽而且有直接而明显的证据,而你娃智商分数又非常高
。这样我觉得像 syuan 所说,找个 GT 班看看能不能更 fit,我觉得是个解决办法。

:我大概就是因为他不是AMC的料,所以觉得他和gifted不沾边。这是我对测试分数觉得
:震惊的一个主要原因
t******l
发帖数: 10908
47
其实我对我家大娃的 AMC 成绩,也是有些 surprised,但是远远不到 shocked。因为
毕竟是相对周围娃不错,而不是说绝对 percentiles 特别高。
再加上我家大娃学校,以前课内数学成绩常常波动很大,但也不是一律低分。她比较
careless,并且 math explanation 方面很差。所以也可能就是稍有不同的学习方法/
途径。这结果也能自圆其说,虽然可能确实是比较 unique 一些(其他类似 AMC 分数
的娃,平时课堂数学不会像她那样不靠谱)。
但我觉得上面这些学习途径的差别,还是个细节问题。而娃分数有时差一点,说白了也
就是我家领导心态不爽,经常鸡飞狗跳一下。
但是大方向而言,我家两娃 fit their home school so well, even proud their
home school, and already made friends。这就是我家没事鸡飞狗跳一下的领导,也是
连 lottery school 都觉得没有必要去。
这个误解是起源于灌水时,大伙儿常常纠结细节战术问题如何对付,这也算 parenting

职业要求。但另一方面,去不去 lottery school 甚至 gifted school 这种大方向战
略层次的决定,这些常常不是细节问题决定的。就像我前面说的,严肃的说的话,在单
个事件上surprised, vs 在一群事件上 shocked,是个鸿沟。

:我大概就是因为他不是AMC的料,所以觉得他和gifted不沾边。这是我对测试分数觉得
:震惊的一个主要原因
s***n
发帖数: 1280
48
我有几个问题,如果你孩子二年级的时候自学七年级的数学,三年级的时候学九年级的
数学,还是GT/honor级的,你为什么认为他不是Gifted? 如果你当初不认为他是Gifted
,你又为什么让他超前五六个年级学数学?另外,如果他能自发地去学习无聊的数学,
你为什么认为他对数学不感兴趣?
我儿子是GT娃,身边GT的朋友同学队友也很多。我对GT教育比较感兴趣,特别是GT娃早
期的表现,鉴定和培养,所以我经常在孩子们一块上课比赛搞活动时,和其它GT娃家长
交流,了解这些GT娃的成长经历。像你孩子的情况,我觉得不用智商测试都可以认定GT
了,所以我有些奇怪是什么原因让你认为他不是GT。

【在 t*******s 的大作中提到】
: 都觉得你是专家啦!不是申请的学校要这个,我都对IQ 一无所知。我几年来一直非常
: 讨厌见老师,听到的老是负面的各种评价。后来就随他自己在YouTube 上乱看一些科技
: 类的东西,这些都是多数大人不感兴趣的莫名其妙video. 有一整年天天在看human
: body Wikipedia 厚书,里面贴满了各种铅笔写的粘贴纸,再后来就自己在khan
: academy 网上学数学,大概8岁做完这个网上7年级的数学。我研究了上面的数学,很普
: 通的短video,没有什么独到之处,我觉得他不是对数学感兴趣,他唯一的动力就是如何
: 快速做题这样能collect 各种花样的badge。最近一年他在AoPS上学algebra, 这是我见
: 过最最无聊的一个学习网站,既没声音,也没video。上面一页一页的讲解翻的及其快
: 。每周7天,3点放学后全部在冰球队游泳队,喜欢钢琴和长号,周末在一个音乐学院的
: 玄乐队联系。至今没发现他在体育音乐上有任何潜能,只能说他非常喜欢,很有

s***n
发帖数: 1280
49
LZ的孩子working memory还在平均线以上,所以孩子对数学感兴趣的话,AMC系列完全
可以尝试。
我儿子的朋友队友中有好几个确诊ADD/ADHD的孩子。这些孩子Working Memory应该比较
差,但他们AMC 8/10的成绩都还不错。这些Twice Exceptional的孩子搞数学竞赛要看
他们的正能量,负能量哪个更牛X。
最近是有些心理学家想剥离了Working Meomory和Processing Speed来看智商,所以有
了LZ提到的 GAI指数。LZ孩子GAI比IQ高,就是因为他的Working Memory把他的IQ总分
拉低了,而GAI没有受影响。

玩。
非常

【在 t******l 的大作中提到】
: 我这么说吧,如果 working memory 很差,AMC 8 基本不可能行,AMC 10 更不可能玩。
: 除非实在是天赋异禀,否则的话我多半认为智商测试的 working memory,那项测错了
: 。你娃是 false negative。加上其他项你娃就是 Highly Gifted 或者 Profoundly
: Gifted。
:
: :都觉得你是专家啦!不是申请的学校要这个,我都对IQ 一无所知。我几年来一直非常
: :讨厌见老师,听到的老是负面的各种评价。后来就随他自己在YouTube 上乱看一些科
: 技类的东西,这些都是多数大人不感兴趣的莫名其妙video. 有一整年天天在看human

s***n
发帖数: 1280
50
你孩子的working memory相对较差,容易粗心,所以题目做全对比较难,这个和他GT与
否关联不大。现在这个阶段,你不用对孩子做题正确率要求太高。花些时间训练他的
working memory,会有助于解决粗心问题。不过这不是一日之功。另外,青少年阶段孩
子的working memory水平会随年龄提高,所以绝大多数孩子你不管他,粗心问题也会慢
慢改善。

【在 t*******s 的大作中提到】
: 这里你提到reading, 这是唯一我能在他身上看到比较闪亮的与academy 有关的长处,
: 我觉得非常好。但是为什么像SSAT 或ISEE 这种考试,虽说分数还不错,但是他并没有
: 像其他牛娃孩子能做到全对或基本全对。他空余时间除了video Game 外,哪怕只有一
: 秒钟,他都在读,即便是视野内只有一个瓶子,他就马上读瓶子上的商标。总之这一秒
: 钟似乎必须保证有事情干
:
: fiction
: 非常

相关主题
求科普: 奥数 vs. Math Olympiad vs. Math Count vs. Math circle vs. 超前学数学大家怎么推孩子的弱项?
有明天考AMC 8的吗?少年智力开发
算不算有数学天分[转载] 为什么说usamo 简单
进入Parenting版参与讨论
t*******s
发帖数: 48
51
你的问题问的很好!为什么我从来没觉得他属于gifted这一类,要么就是我对gifted这
个定义有误解,要么就是他真的不是,或者说顶多算是smart这个范畴内。主要观察如
下:
1、无论是khan Academy 还是AoPS, 我觉得他对上面的各种badge和论坛的 passion 要
远大于数学本身的兴趣。他从来没有表现出自已要主动去上面研究各种难题。比如AoPS
pre-algebra 书上每章开始有21点算术,他不会坚持住自己研究各种可能性。他就是
喜欢在这些网站上留言啊,回答与数学无关的问题
2、他愿意一直在这些网站上继续学习,我认为很大一个原因是他想靠这种所谓超前学
习的机会来维持住自认为聪明的这个特征。他1到3年级一直上一个很好的私立,数学基
本上都是在比他高一年级的班上,另外有一个老师专门给他上些乱七八糟的物理什么的
,还有一个心理老师每周约他谈心一个小时。估计这些特殊情况让他觉得他比其他孩子
cool,很可能是这种感觉在drive 他愿意多学点东西。不觉得是发自内心的passion
3、再就是我所理解的gifted 孩子就是我读书时班上总是有个别男生,你从来就没觉得
他们在学习,从没看到他们努力过,但是就是学习聪明。我们从前高中住校,不存在回
家补习这种可能性。我家这孩子不是这样的,网上学习的事情都是随他选随他掌握节奏
。这点又可以说他学的估计比大部分孩子多。
4、前面提过了他爱阅读,非常爱。任何一秒钟都不能让眼睛空闲下来。4年级跨州搬家
,进了公立。学校没人管阅读了,我就让他每天给我读两页the economist杂志,内容
随他选。他唯一的长处就是非常有毅力,天天读不间断。多次问他喜欢吗,答案总是读
这个杂志有利于快速睡眠。
上面这些近距离的观察和我所认为的gifted不一样。

Gifted
GT

【在 s***n 的大作中提到】
: 我有几个问题,如果你孩子二年级的时候自学七年级的数学,三年级的时候学九年级的
: 数学,还是GT/honor级的,你为什么认为他不是Gifted? 如果你当初不认为他是Gifted
: ,你又为什么让他超前五六个年级学数学?另外,如果他能自发地去学习无聊的数学,
: 你为什么认为他对数学不感兴趣?
: 我儿子是GT娃,身边GT的朋友同学队友也很多。我对GT教育比较感兴趣,特别是GT娃早
: 期的表现,鉴定和培养,所以我经常在孩子们一块上课比赛搞活动时,和其它GT娃家长
: 交流,了解这些GT娃的成长经历。像你孩子的情况,我觉得不用智商测试都可以认定GT
: 了,所以我有些奇怪是什么原因让你认为他不是GT。

t******l
发帖数: 10908
52
Working memory 差会比较粗心,但反之不成立,粗心不一定是 working memory 差。
比如 intuitive thinker 都会比较粗心一些,因为 implicit-memory “抓大放小” /
“见林不见树” 的特征,以及相应的大脑潜意识里优先权的设置。
另外从进化优势的角度,“细心” 本身并不是进化优势,“快糙猛” 标枪出手,然后
大块吃肉,才是进化优势。

:LZ的孩子working memory还在平均线以上,所以孩子对数学感兴趣的话,AMC系列完
t******l
发帖数: 10908
53
我都不用心理学的啥智商测试。
我家两娃都常常心算,不用草稿纸。working memory 差不可能心算。但她们的正确率
都偏低。
其实 working memory 不差的,多用心算,反而会正确率偏低。working memory 差的
,但笔算习惯好,正确率只高不低。例子就是初中数学三八红旗手女生。
我觉得心理学是对于 neuro-atypical 的研究的比较多,加上目前本身对大脑研究的科
学手段比较原始,这样对 neuro-typical 的娃娃,常常搞出按常识看都有点匪夷所思
的结论。

:Working memory 差会比较粗心,但反之不成立,粗心不一定是 working memory 差。
t*******s
发帖数: 48
54
Homeschool 真的是非常brave 的决定,我们家孩子讨厌我在家里教。当然话说回来我
没有能力教他, 比起上班来说,教自己孩子太challenge 了

也是

【在 t******l 的大作中提到】
: 其实我对我家大娃的 AMC 成绩,也是有些 surprised,但是远远不到 shocked。因为
: 毕竟是相对周围娃不错,而不是说绝对 percentiles 特别高。
: 再加上我家大娃学校,以前课内数学成绩常常波动很大,但也不是一律低分。她比较
: careless,并且 math explanation 方面很差。所以也可能就是稍有不同的学习方法/
: 途径。这结果也能自圆其说,虽然可能确实是比较 unique 一些(其他类似 AMC 分数
: 的娃,平时课堂数学不会像她那样不靠谱)。
: 但我觉得上面这些学习途径的差别,还是个细节问题。而娃分数有时差一点,说白了也
: 就是我家领导心态不爽,经常鸡飞狗跳一下。
: 但是大方向而言,我家两娃 fit their home school so well, even proud their
: home school, and already made friends。这就是我家没事鸡飞狗跳一下的领导,也是

t******l
发帖数: 10908
55
你提到的那些 ADD / ADHD 但 AMC 成绩不错的娃,你有没有近距离观察过,他们/她们
在解 AMC 题目的时候,包括学习阶段,更倾向于是笔算还是心算?

:LZ的孩子working memory还在平均线以上,所以孩子对数学感兴趣的话,AMC系列完全
t******l
发帖数: 10908
56
我说的 home school,是相对 lottery school gifted school 而言的,用词不当容易
误解,我去改一下。

:Homeschool 真的是非常brave 的决定,我们家孩子讨厌我在家里教。当然话说回来我
:没有能力教他, 比起上班来说,教自己孩子太challenge 了
t******l
发帖数: 10908
57
我改了原贴,用 “普通公立学校”,这样消除二义性误会。我娃不是 homeschooling。

:我娃不是 homeschooling,我娃正常上普通大众学校。我就是在家里有针对性的对概
念层次给加点料,这样娃能更好地适应学校。
t******l
发帖数: 10908
58
我同意你说的 “ gifted 不同于 “早会” ” 的观点。
但你后面举的例子跟 gifted 与否关系不大。反而是有点像 “普通娃心态与否” 的判
断标准,除了那个 21 点之外。普通娃不会纠结 21 点这么枯燥的东西。

:你的问题问的很好!为什么我从来没觉得他属于gifted这一类,要么就是我对gifted
这个定义有误解,要么就是他真的不是,或者说顶多算是smart这个范畴内。主要观察如
:下:
t******l
发帖数: 10908
59
阅读方面,我家大娃在三年级/四年级横扫图书馆 juvenile non-fiction section 的
时候,把阅读当成看电视那样,每天无数小时数。
但大娃五年级以后就不怎么去图书馆了。现在只借学校要求阅读的 fiction 的阅读量。
我觉得那个就是个认知阶段,娃常常是三分钟热度。不能拿热度期当持之以恒来看。

:你的问题问的很好!为什么我从来没觉得他属于gifted这一类,要么就是我对gifted
这个定义有误解,要么就是他真的不是,或者说顶多算是smart这个范畴内。主要观察如
:下:
t******l
发帖数: 10908
60
Slow processing speed 的测试手段,我查了一下,我真觉得不用说啥了。
http://www.2enewsletter.com/article_2013_05_slow_processing.html
Reading Fluency. For three minutes the student quickly reads simple
sentences and answers yes or no to each.
我真心说一句,用这点排查弱智可以(simple simple 再 simple),用这点测 GT,违
反人类大脑进化的一个基本原则:
在给定能量限制下,大脑是为 perform complex task 而优化的,这样大脑在 perform
simple task 的时候有时还 clumsy。但在实践中,simple task 一般用脊髓靠 fine
motor 盲打就可以完成的,大脑不会为此而优化,否则就是 neuro-atypical。

:LZ的孩子working memory还在平均线以上,所以孩子对数学感兴趣的话,AMC系列完全
相关主题
二年级女儿的report我招,我是猪
觉得美帝数学进度慢的进来看一下老师说我女儿是gifted kid (转载)
分数应用题忍不住上来问问,我娃的数学是啥水平(6岁)?
进入Parenting版参与讨论
t******l
发帖数: 10908
61
或者更直接的说,fast reader 是指 read complex yet meaningful material in an
astonishing speed with superior critical thinking。
而心理学家的看法,就好比昌平挖沙挖的又快又好,就可以去当小马工了。。。虽然简
单的单句读太慢到弱智也是不行,但是 Homo Sapiens 大脑的 perceptual memory ,
根本不是为读简单的单句而优化的。

:Slow processing speed 的测试手段,我查了一下,我真觉得不用说啥了。
t******l
发帖数: 10908
62
或者一个简单的但不太确切的比方,“早会” 就好比是 “穷人的孩子早当家”,而
“gifted“ 就好比是 ”迈入豪门之恩恩怨怨“,不完全是同一范畴的概念。// run

:我同意你说的 “ gifted 不同于 “早会” ” 的观点。
t*******s
发帖数: 48
63
我家孩子阅读的兴趣非常杂,一二三四年级时也是拿起书就能好几个小时在那里,什么
也听不见。四年级第二学期Minecraft 占了几乎所有仅有的一点空闲的时间。但是从来
没看到他在Minecraft online game上有任何impressive 的project 显示出来

量。
gifted
察如

【在 t******l 的大作中提到】
: 阅读方面,我家大娃在三年级/四年级横扫图书馆 juvenile non-fiction section 的
: 时候,把阅读当成看电视那样,每天无数小时数。
: 但大娃五年级以后就不怎么去图书馆了。现在只借学校要求阅读的 fiction 的阅读量。
: 我觉得那个就是个认知阶段,娃常常是三分钟热度。不能拿热度期当持之以恒来看。
:
: :你的问题问的很好!为什么我从来没觉得他属于gifted这一类,要么就是我对gifted
: 这个定义有误解,要么就是他真的不是,或者说顶多算是smart这个范畴内。主要观察如
: :下:

T********s
发帖数: 100
64
你单开一贴给大家讲一讲GT娃早期的表现,鉴定和培养吧。
还有怎样训练working memory。
还有关于你娃有时读不懂题的,你以前帖子说过的,现在找不着了。能不能再
说一说你是怎么帮助他解决这个问题的?
你的贴总是信息量很大,很值得追。咱们多push几下,让你多讲一些。

Gifted
GT

【在 s***n 的大作中提到】
: 我有几个问题,如果你孩子二年级的时候自学七年级的数学,三年级的时候学九年级的
: 数学,还是GT/honor级的,你为什么认为他不是Gifted? 如果你当初不认为他是Gifted
: ,你又为什么让他超前五六个年级学数学?另外,如果他能自发地去学习无聊的数学,
: 你为什么认为他对数学不感兴趣?
: 我儿子是GT娃,身边GT的朋友同学队友也很多。我对GT教育比较感兴趣,特别是GT娃早
: 期的表现,鉴定和培养,所以我经常在孩子们一块上课比赛搞活动时,和其它GT娃家长
: 交流,了解这些GT娃的成长经历。像你孩子的情况,我觉得不用智商测试都可以认定GT
: 了,所以我有些奇怪是什么原因让你认为他不是GT。

T********s
发帖数: 100
65
你这个孩子可能不是数学天才,但他是highly GT无疑。

AoPS

【在 t*******s 的大作中提到】
: 你的问题问的很好!为什么我从来没觉得他属于gifted这一类,要么就是我对gifted这
: 个定义有误解,要么就是他真的不是,或者说顶多算是smart这个范畴内。主要观察如
: 下:
: 1、无论是khan Academy 还是AoPS, 我觉得他对上面的各种badge和论坛的 passion 要
: 远大于数学本身的兴趣。他从来没有表现出自已要主动去上面研究各种难题。比如AoPS
: pre-algebra 书上每章开始有21点算术,他不会坚持住自己研究各种可能性。他就是
: 喜欢在这些网站上留言啊,回答与数学无关的问题
: 2、他愿意一直在这些网站上继续学习,我认为很大一个原因是他想靠这种所谓超前学
: 习的机会来维持住自认为聪明的这个特征。他1到3年级一直上一个很好的私立,数学基
: 本上都是在比他高一年级的班上,另外有一个老师专门给他上些乱七八糟的物理什么的

t******l
发帖数: 10908
66
我管我家大娃小学期间的阅读,所以我知道我家大娃虽然看起来阅读很杂,但其实是有
非常明显确切的 pattern 的。
我家大娃的 non-fiction 阅读,开始于二年级暑假快结束时,直接原因是 school
report 的字里行间,我觉得有词汇方面的 concern。
所以我给我家大娃开始课后图书馆阅读。跟目前的一年级小娃不同的是,大娃她开始图
书馆阅读时是三年级,那时她已经可以自己开始独立阅读,以及 silent reading。(
不像目前一年级小娃,需要 read to parent)。
大娃她的阅读一条平行线是开始于 juvenile non-fiction 的 biography topic,从大
约 lexile 500L 左右的 biography 开始,到三年级的 X'mas 的时候她对 Anne Frank
的 the diary of a young girl (lexile 1080L) 感兴趣,反复借出来好几次,之后
text complexity 不再是图书馆阅读的瓶颈。之后读一些 juvenile non-fiction
history 之类。
她课后图书馆阅读的另一条平行线,是 juvenile non-fiction 的 animal topic,以
及 human body topic。她花很多时间在 human body topic。而 astronomy /
cosmology 也看一些,比如 13 planets 等, 但总之兴趣不高。
她五年级就基本不再课后图书馆阅读。到现在七年级,她图书馆阅读不多,但开始有一
些,topic 换成 young adult fiction book。
我家大娃三年级在学校的课内阅读,最喜欢是 mystery / history fiction。
我个人觉得我家大娃的阅读 path 其实非常非常有其内在合理性和自洽性,就是折射一
个普通娃的心理(以及生理)成长过程,受 curiosity 和 self-awareness 的两重推
动。只是跟学校和大众期望的路线(比如哈利波特这型)稍有不同而已。

:我家孩子阅读的兴趣非常杂,一二三四年级时也是拿起书就能好几个小时在那里,什
么也听不见。四年级第二学期Minecraft 占了几乎所有仅有的一点空闲的时间。但是从
来没看到他在Minecraft online game上有任何impressive 的project 显示出来
t*******s
发帖数: 48
67
我喜欢看你这段关于阅读的过程。我们是从十来页picture book 快速进入Harry
Potter 1 , 中间大概就经历了个把本Magic Tree house。估计刚进一年级时太小,并
没有完全读懂第一本Harry Potter, 突然立刻转入画各种植物动物和人体细胞的图,在
YouTube 上看各种各样关于细胞的video,这个经历过去后就有一年多时间在专心看
human body Wikipedia 这本书。这段细胞类的阶段让他词汇量快速的累积上来,尤其
是各种细胞的名字,单词特别长。在二年级快结束时迅速看完所有harry potter。一二
三年级每周大大小小的书总能读二十多本吧。我觉得小朋友读书尽量不要限制题材,稍
微引导一下大方向就可以了。他的WISC 上的VCI 是155(>99.9%) 估计基本上就是他的
实际阅读水平。

Frank


【在 t******l 的大作中提到】
: 我管我家大娃小学期间的阅读,所以我知道我家大娃虽然看起来阅读很杂,但其实是有
: 非常明显确切的 pattern 的。
: 我家大娃的 non-fiction 阅读,开始于二年级暑假快结束时,直接原因是 school
: report 的字里行间,我觉得有词汇方面的 concern。
: 所以我给我家大娃开始课后图书馆阅读。跟目前的一年级小娃不同的是,大娃她开始图
: 书馆阅读时是三年级,那时她已经可以自己开始独立阅读,以及 silent reading。(
: 不像目前一年级小娃,需要 read to parent)。
: 大娃她的阅读一条平行线是开始于 juvenile non-fiction 的 biography topic,从大
: 约 lexile 500L 左右的 biography 开始,到三年级的 X'mas 的时候她对 Anne Frank
: 的 the diary of a young girl (lexile 1080L) 感兴趣,反复借出来好几次,之后

t******l
发帖数: 10908
68
我觉得你跟我说的娃,虽然有一些类似的地方,但到细节之处就截然不同了。具体在
Human body topic 的 library non-fiction reading 而言:
我家大娃三年级在 human body 方面,基本从来不看 cell 之类的 “抽象” 的内容。
大娃三年级在 human body 方面,最喜欢的三大类:
一类是 children growth / nutrition topic,比如掉个牙齿,吃菜长肉之类的。
另一类就是 human intelligence topic,比如 “How to be a genius / written by
John Woodward” 那本借出来过无数次。
再一类就是 disease 之类的,比如 flu, teeth cavity, eczema, allergy, ADD/ADHD
, Autism, dyslexia, leukemia, faulty hearts 等等等等。
总之都是直接跟她的日常生活有直接间接联系的,她才有兴趣去看。

:我喜欢看你这段关于阅读的过程。我们是从十来页picture book 快速进入Harry
:Potter 1 , 中间大概就经历了个把本Magic Tree house。估计刚进一年级时太小,并
t******l
发帖数: 10908
69
另外我家大娃从没看过哈利波特。她学校里或图书馆里,翻了翻就送回去了。
她对哈利波特不感兴趣。我对大娃的潜意识的猜测是:“how those stuff related to
me?"

:我喜欢看你这段关于阅读的过程。我们是从十来页picture book 快速进入Harry
:Potter 1 , 中间大概就经历了个把本Magic Tree house。估计刚进一年级时太小,并
t******l
发帖数: 10908
70
其实我曾经试过加强娃在 cellular level 的 human body reading,但最后发现不行。
我后来觉得娃认为的重要与否、有用与否,跟大人的角度并不一样。

:我喜欢看你这段关于阅读的过程。我们是从十来页picture book 快速进入Harry
:Potter 1 , 中间大概就经历了个把本Magic Tree house。估计刚进一年级时太小,并
相关主题
忍不住上来问问,我娃的数学是啥水平(6岁)?放养妈进来看看
陶天才论天才 (转载)最新研究证明心算AIME大幅度促进心理和情感健康
NIH 证实天才和疯子都源于同一型基因:DARPP-32来看一下各位的观点
进入Parenting版参与讨论
s***n
发帖数: 1280
71
这个真不好说。没看过他们的草稿纸。
其中一个我比较熟的孩子的特点是做题很快,但准确率稍差。AMC10 提前做完题目,成
绩 100+, 没到AIME线。Mathcounts抢答赛成绩很好。

完全

【在 t******l 的大作中提到】
: 你提到的那些 ADD / ADHD 但 AMC 成绩不错的娃,你有没有近距离观察过,他们/她们
: 在解 AMC 题目的时候,包括学习阶段,更倾向于是笔算还是心算?
:
: :LZ的孩子working memory还在平均线以上,所以孩子对数学感兴趣的话,AMC系列完全

t******l
发帖数: 10908
72
就拿你知道的这个例子,你怎么能确定是该准确率是 working memory 的问题所致,而
不是大脑在速度和准确率之间的折中所致?

:这个真不好说。没看过他们的草稿纸。
s***n
发帖数: 1280
73
是有一些孩子心算不行只能笔算,但这些孩子心算不行,大多是数学水平问题,不是
working memory问题。真正working memory差的普通孩子,心算笔算都不行。
很多孩子倾向笔算只是因为习惯了笔算。GT孩子也一样,心算笔算大多是习惯养成问题
,和working memory强弱关系已经不大。我的观察,小娃图省事图快,多心算;大娃稳
重些了,会多用笔算。
一般认为ADHD的孩子working memory差。但很多ADHD的GT娃,可以用他们gifted的能力
,特别是在逻辑推理和大脑处理速度方面的优势,来弥补working memory的不足,所以
在表现上,ADHD的HGT孩子反而会显得心算和抢答快,快枪手,但他们的相对正确率(相
对其他GT娃)会随着题目难度加大快速下降。

差。

【在 t******l 的大作中提到】
: 我都不用心理学的啥智商测试。
: 我家两娃都常常心算,不用草稿纸。working memory 差不可能心算。但她们的正确率
: 都偏低。
: 其实 working memory 不差的,多用心算,反而会正确率偏低。working memory 差的
: ,但笔算习惯好,正确率只高不低。例子就是初中数学三八红旗手女生。
: 我觉得心理学是对于 neuro-atypical 的研究的比较多,加上目前本身对大脑研究的科
: 学手段比较原始,这样对 neuro-typical 的娃娃,常常搞出按常识看都有点匪夷所思
: 的结论。
:
: :Working memory 差会比较粗心,但反之不成立,粗心不一定是 working memory 差。

t******l
发帖数: 10908
74
我觉得你后边的结论关于 ADHD 和 working memory 的 causation 层次上是错误的,
也就是我认为在这点上,ADHD 跟 working memory 不一定有直接的因果关系。
当然其实这个错误的源头不是你,而是我们的心理学研究的 sampling 的 selection
bias 造成的。
我刚才查了 wiki 的 working memory,觉得你的观点的源头是 wiki 上的一个测试。
但是我觉得那个测试有 sampling bias,测试结果本身是错误的。我先发出这贴,我下
一贴详细说。

【在 s***n 的大作中提到】
: 是有一些孩子心算不行只能笔算,但这些孩子心算不行,大多是数学水平问题,不是
: working memory问题。真正working memory差的普通孩子,心算笔算都不行。
: 很多孩子倾向笔算只是因为习惯了笔算。GT孩子也一样,心算笔算大多是习惯养成问题
: ,和working memory强弱关系已经不大。我的观察,小娃图省事图快,多心算;大娃稳
: 重些了,会多用笔算。
: 一般认为ADHD的孩子working memory差。但很多ADHD的GT娃,可以用他们gifted的能力
: ,特别是在逻辑推理和大脑处理速度方面的优势,来弥补working memory的不足,所以
: 在表现上,ADHD的HGT孩子反而会显得心算和抢答快,快枪手,但他们的相对正确率(相
: 对其他GT娃)会随着题目难度加大快速下降。
:

t******l
发帖数: 10908
75
这个 wiki 上说的测试是:
http://en.wikipedia.org/wiki/Working_memory#Measures_and_correlates
[quote]
These authors asked people to compare written statements about the relations
between several variables to graphs illustrating the same or a different
relation, as in the following sentence:
"If the cake is from France, then it has more sugar if it is made with
chocolate than if it is made with cream, but if the cake is from Italy, then
it has more sugar if it is made with cream than if it is made of chocolate"
.
This statement describes a relation between three variables (country,
ingredient, and amount of sugar), which is the maximum most individuals can
understand. The capacity limit apparent here is obviously not a memory limit
(all relevant information can be seen continuously) but a limit to how many
relationships are discerned simultaneously.
[/quote]

【在 t******l 的大作中提到】
: 我觉得你后边的结论关于 ADHD 和 working memory 的 causation 层次上是错误的,
: 也就是我认为在这点上,ADHD 跟 working memory 不一定有直接的因果关系。
: 当然其实这个错误的源头不是你,而是我们的心理学研究的 sampling 的 selection
: bias 造成的。
: 我刚才查了 wiki 的 working memory,觉得你的观点的源头是 wiki 上的一个测试。
: 但是我觉得那个测试有 sampling bias,测试结果本身是错误的。我先发出这贴,我下
: 一贴详细说。

t******l
发帖数: 10908
76
然后我试了不用草稿纸,mentally process(等价于心算)这个坑爹长句(其实主要原
因是我懒,我刚才拿着手机坐在沙发上,懒得去拿笔和草稿纸)。于是我 scan 了这句
句子三遍还是四遍:
"If the cake is from France, then it has more sugar if it is made with
chocolate than if it is made with cream, but if the cake is from Italy, then
it has more sugar if it is made with cream than if it is made of chocolate"
然后我直接在脑子里,没有真正走那个 author 想要的心中画个图论 graph。我直接在
大脑里把这句话给 reduce 成:
In France, chocolate cake has more sugar than cream cake.
In Italy, chocolate cake has less sugar than cream cake.
嘎简单的事儿。

relations
then
chocolate"

【在 t******l 的大作中提到】
: 这个 wiki 上说的测试是:
: http://en.wikipedia.org/wiki/Working_memory#Measures_and_correlates
: [quote]
: These authors asked people to compare written statements about the relations
: between several variables to graphs illustrating the same or a different
: relation, as in the following sentence:
: "If the cake is from France, then it has more sugar if it is made with
: chocolate than if it is made with cream, but if the cake is from Italy, then
: it has more sugar if it is made with cream than if it is made of chocolate"
: .

t******l
发帖数: 10908
77
那我现在说为啥 wiki 上测试结果的 selection biased,我觉得原因可能是原因
sample 的都是 lexile < 1600L 的 reader。
如果 sampling lexile > 1600L+ 的 reader,下面是可能发生的情况。
第一遍 scan,陷入军医们的 trap,试图在脑子里创建 graph。
但在此同时,以前 reading 的 implicit memory / sub-consciousness 会 kick in,
intuitively "feel" 老军医的这个 sentence never happen in past reading。
然后大脑立即切换优先权,不再按照老军医的陷阱去画 graph,而是大脑把 focus(主
要能量)转向另一个问题:“what is the heck of this weird and never-seen-
before-in-past-reading sentence?”
先发出来,我下贴继续说。

then

【在 t******l 的大作中提到】
: 然后我试了不用草稿纸,mentally process(等价于心算)这个坑爹长句(其实主要原
: 因是我懒,我刚才拿着手机坐在沙发上,懒得去拿笔和草稿纸)。于是我 scan 了这句
: 句子三遍还是四遍:
: "If the cake is from France, then it has more sugar if it is made with
: chocolate than if it is made with cream, but if the cake is from Italy, then
: it has more sugar if it is made with cream than if it is made of chocolate"
: 然后我直接在脑子里,没有真正走那个 author 想要的心中画个图论 graph。我直接在
: 大脑里把这句话给 reduce 成:
: In France, chocolate cake has more sugar than cream cake.
: In Italy, chocolate cake has less sugar than cream cake.

t******l
发帖数: 10908
78
然后下面一步,我觉得可能的是,大脑三个同步进程同时分析:
(1) text complexity analysis: long sentence, complex structure, high lexile
measurement
(2) content complexity analysis: cake, type, make from, more-or-less sugar
=> low content complexity
(3) pragmatics analysis (theory of mind): 这不是 AIME,老军医测试的玩意儿不
会超过猴子的智力。
Judgement / vote 的结论是: 老军医又玩 fabricated text complexity (我为啥说
“又” 涅?)。需要火速调动 British syntax/semantics simplifier module。

【在 t******l 的大作中提到】
: 那我现在说为啥 wiki 上测试结果的 selection biased,我觉得原因可能是原因
: sample 的都是 lexile < 1600L 的 reader。
: 如果 sampling lexile > 1600L+ 的 reader,下面是可能发生的情况。
: 第一遍 scan,陷入军医们的 trap,试图在脑子里创建 graph。
: 但在此同时,以前 reading 的 implicit memory / sub-consciousness 会 kick in,
: intuitively "feel" 老军医的这个 sentence never happen in past reading。
: 然后大脑立即切换优先权,不再按照老军医的陷阱去画 graph,而是大脑把 focus(主
: 要能量)转向另一个问题:“what is the heck of this weird and never-seen-
: before-in-past-reading sentence?”
: 先发出来,我下贴继续说。

t******l
发帖数: 10908
79
British syntax/semantics simplifier 拍马过来一看,说了一句:卧槽,虚词太多,
短修饰后置太坑爹,more 和 less 还分两词,顺序还反一下。骗猴子的把戏。
于是 British syntax/semantics simplifier 基于代数变换等价性,删虚词,短修饰
语改成前置,more 和 less 只用一个。这些都是纯形式变换,不需要理解意思。三下
五除二,最后调 formatter 给 text align 一下,收拾书包走人。
然后逻辑模块过来一看,卧槽,这么简单明了的事儿,老军医居然能写成 lexile
1600L+。不说了,老军医自己先去查查是不是被病人传染了 hyperlexia。// super
fast run

lexile

【在 t******l 的大作中提到】
: 然后下面一步,我觉得可能的是,大脑三个同步进程同时分析:
: (1) text complexity analysis: long sentence, complex structure, high lexile
: measurement
: (2) content complexity analysis: cake, type, make from, more-or-less sugar
: => low content complexity
: (3) pragmatics analysis (theory of mind): 这不是 AIME,老军医测试的玩意儿不
: 会超过猴子的智力。
: Judgement / vote 的结论是: 老军医又玩 fabricated text complexity (我为啥说
: “又” 涅?)。需要火速调动 British syntax/semantics simplifier module。

t******l
发帖数: 10908
80
玩笑归玩笑,但是军医学所说的大脑 working memory 的 chunking,其实也表现在从
这句坑爹句:
"If the cake is from France, then it has more sugar if it is made with
chocolate than if it is made with cream, but if the cake is from Italy, then
it has more sugar if it is made with cream than if it is made of chocolate"
到简明句:
In France, chocolate cake has more sugar than cream cake.
In Italy, chocolate cake has less sugar than cream cake.
的过程。
所以罗素又跑出来说,chunking 就是:在这句著名的 “the magic number 7, +-2”
里,老军医把 "1" 给定义了没?ZFC set theory 在哪里啊我罗素——看——不——见
——!!!

【在 t******l 的大作中提到】
: British syntax/semantics simplifier 拍马过来一看,说了一句:卧槽,虚词太多,
: 短修饰后置太坑爹,more 和 less 还分两词,顺序还反一下。骗猴子的把戏。
: 于是 British syntax/semantics simplifier 基于代数变换等价性,删虚词,短修饰
: 语改成前置,more 和 less 只用一个。这些都是纯形式变换,不需要理解意思。三下
: 五除二,最后调 formatter 给 text align 一下,收拾书包走人。
: 然后逻辑模块过来一看,卧槽,这么简单明了的事儿,老军医居然能写成 lexile
: 1600L+。不说了,老军医自己先去查查是不是被病人传染了 hyperlexia。// super
: fast run
:
: lexile

相关主题
是不是任我儿子继续迷象棋有明天考AMC 8的吗?
9岁, 如何报名考amc10算不算有数学天分
求科普: 奥数 vs. Math Olympiad vs. Math Count vs. Math circle vs. 超前学数学大家怎么推孩子的弱项?
进入Parenting版参与讨论
t******l
发帖数: 10908
81
当然,在这个 chunking 过程里,perceptual memory / implicit memory 在后台偷偷
改了一个词,把 "from" 改成 "in"。
这个 perceptual / implicit 的修改不是等价变换,这个确实增加 chunking 的效率
(减少一个 motion 动作),但是有时也会造成错误率升高。
这种错误率其实将来靠更强的 perceptual 来调整,也就是 intuitively detect 是不
是上下文合适(pragmatics-appropriate)。

then


【在 t******l 的大作中提到】
: 玩笑归玩笑,但是军医学所说的大脑 working memory 的 chunking,其实也表现在从
: 这句坑爹句:
: "If the cake is from France, then it has more sugar if it is made with
: chocolate than if it is made with cream, but if the cake is from Italy, then
: it has more sugar if it is made with cream than if it is made of chocolate"
: 到简明句:
: In France, chocolate cake has more sugar than cream cake.
: In Italy, chocolate cake has less sugar than cream cake.
: 的过程。
: 所以罗素又跑出来说,chunking 就是:在这句著名的 “the magic number 7, +-2”

t******l
发帖数: 10908
82
我前面的意思是,也就是存在另一种可能性,也就是 working memory 和 ADHD 只存在
相关性而不是因果性。
而相关性非因果性的原因有可能有一百万种,比如大脑某种多巴胺史蒂芬坑啥的分泌异
常,结果同时影响 working memory 和 ADHD。而那种史蒂芬坑分泌在正常范围内的娃
,working memory 大点小点问题都不是太大。当然这里的史蒂芬坑分泌只是个比方,
不一定是大脑内分泌系统。

【在 s***n 的大作中提到】
: 是有一些孩子心算不行只能笔算,但这些孩子心算不行,大多是数学水平问题,不是
: working memory问题。真正working memory差的普通孩子,心算笔算都不行。
: 很多孩子倾向笔算只是因为习惯了笔算。GT孩子也一样,心算笔算大多是习惯养成问题
: ,和working memory强弱关系已经不大。我的观察,小娃图省事图快,多心算;大娃稳
: 重些了,会多用笔算。
: 一般认为ADHD的孩子working memory差。但很多ADHD的GT娃,可以用他们gifted的能力
: ,特别是在逻辑推理和大脑处理速度方面的优势,来弥补working memory的不足,所以
: 在表现上,ADHD的HGT孩子反而会显得心算和抢答快,快枪手,但他们的相对正确率(相
: 对其他GT娃)会随着题目难度加大快速下降。
:

t******l
发帖数: 10908
83
其实我觉得军医学的 working memory system 里,最难解释的,就是在 non-routine
task 的时候,在 working memory 集中力量 working solution strategy A 的时候,
甚至人自己还没有意识到存在 solution strategy B 的时候,implicit memory 有时
会突然介入,让 working memory 放弃当前所有的计算结果,转向 search different
solution strategy。
而这种 non-routine 的情况,不仅仅存在 AIME,实际上存在我们每天走路上学、或者
骚扰花花草草的时候。唯一不存在的情况,确实老军医创建的智商测试,因为那是
controlled environment,repeatable,也就是不允许 non-routine situation 的存
在。
而这种 implicit memory 的从后台(潜意识)突然窜到前台(意识),强迫 working
memory 放弃当前所有计算结果,清空缓存,同时换新算法的机制,有可能跟
attention span 有关。而 attention span 有可能跟 ADHD 有关。
t******l
发帖数: 10908
84
我想你这边可能有个 sampling bias,也就是为了竞赛之类的,或多或少都训练了。所
以你提到了 “习惯” 因子的干扰。
如果是业余酱油组,就可能没有很强的 “习惯” 因子。但是业余组的问题,是根本做
不到 AMC 10 100+,所以也无从讨论。

:是有一些孩子心算不行只能笔算,但这些孩子心算不行,大多是数学水平问题,不是
t******l
发帖数: 10908
85
我觉得要排除 speed / accuracy 加强型 training 的因子。
我听说有 HGT 娃不需要额外的 speed / accuracy training,在这群里互相比较,我
觉得能排除 “外加的习惯” 因子的影响。

:是有一些孩子心算不行只能笔算,但这些孩子心算不行,大多是数学水平问题,不是
t*******s
发帖数: 48
86
我们家虽说10岁蛮积极的在上AoPS online algebra, 也能跟上课,但是我发现他心算
准确率不高,而且还非常讨厌用笔算,这也是我困惑的地方。是否gifted不重要,问题
是如何改变这种比较懒散的学习习惯。我觉得所谓HGT的孩子,心算能力肯定是super
good

【在 t******l 的大作中提到】
: 我想你这边可能有个 sampling bias,也就是为了竞赛之类的,或多或少都训练了。所
: 以你提到了 “习惯” 因子的干扰。
: 如果是业余酱油组,就可能没有很强的 “习惯” 因子。但是业余组的问题,是根本做
: 不到 AMC 10 100+,所以也无从讨论。
:
: :是有一些孩子心算不行只能笔算,但这些孩子心算不行,大多是数学水平问题,不是

t*******s
发帖数: 48
87
我觉得能进入AMC的孩子,一定是gifted的孩子,不能在归入bright 这个范畴了。你们
说说标准化考试比如SAT 99% 孩子绝对是非常聪明的,拿满分非常非常难,为什么大家
觉得很多这种孩子不算是gifted?

【在 t******l 的大作中提到】
: 我想你这边可能有个 sampling bias,也就是为了竞赛之类的,或多或少都训练了。所
: 以你提到了 “习惯” 因子的干扰。
: 如果是业余酱油组,就可能没有很强的 “习惯” 因子。但是业余组的问题,是根本做
: 不到 AMC 10 100+,所以也无从讨论。
:
: :是有一些孩子心算不行只能笔算,但这些孩子心算不行,大多是数学水平问题,不是

t*******s
发帖数: 48
88
嗯,同意你的。小朋友认为的重要与否不一样。我特别纠结他呆在YouTube 上看
Minecraft,每次看到都试图说服他要不看看什么world history 之类的video, 既然他
阅读能力很强,去看这种history 的东西又不需要基础,但是他只能坚持10分钟就又回
到Minecraft 上了。所以我家不给他computer 的password,iPad 也是不给的。我现在
对这种电子设备是敌对态度,但是还不能明显表现出来我恨这些game.

行。
,并

【在 t******l 的大作中提到】
: 其实我曾经试过加强娃在 cellular level 的 human body reading,但最后发现不行。
: 我后来觉得娃认为的重要与否、有用与否,跟大人的角度并不一样。
:
: :我喜欢看你这段关于阅读的过程。我们是从十来页picture book 快速进入Harry
: :Potter 1 , 中间大概就经历了个把本Magic Tree house。估计刚进一年级时太小,并

t******l
发帖数: 10908
89
我觉得更有意思的是,大脑的 working memory "chunking" 并没有停留下下面这步:
In France, chocolate cake has more sugar than cream cake.
In Italy, chocolate cake has less sugar than cream cake.
我们想象在没有压力(stress)的情况下,也就是没有考前死记硬背的要求。(stress
确实影响 working memory)。大脑会继续 "pragmatics-like chunking" 成下面两个
level 的信息,并且放入 long-term memory:
(higher order pragmatics "chunking"):
Different location has different order. (Order is function/LUT of location).
(lower order pragmatics "chunking"):
Location is France vs Italy.
Order is the type of cake (chocolate / cream) ordered by sugar percentage.
而具体哪个多,哪个少,大脑不一定 focus 在记忆那个具体细节。(在这个环境下,
大脑潜意识 presuming 可以回去查 wiki)
当然,我们的考试压力(stress)有可能改变大脑的 high-order (pragmatics)
chunking 的行为。这样是不是对有部分人群,环境的压力会不会引起或加重
hyperlexia-like syndrome?这个军医学估计会考虑政治正确社会影响先,而不是
scientific study。

【在 t******l 的大作中提到】
: 当然,在这个 chunking 过程里,perceptual memory / implicit memory 在后台偷偷
: 改了一个词,把 "from" 改成 "in"。
: 这个 perceptual / implicit 的修改不是等价变换,这个确实增加 chunking 的效率
: (减少一个 motion 动作),但是有时也会造成错误率升高。
: 这种错误率其实将来靠更强的 perceptual 来调整,也就是 intuitively detect 是不
: 是上下文合适(pragmatics-appropriate)。
:
: then
: ”

t******l
发帖数: 10908
90
回答 “SAT 99% 孩子绝对是非常聪明的,拿满分非常非常难,为什么大家觉得很多这
种孩子不算是gifted”。
这个打个比方,“老实大量读经” 能背诵几百万字经书的,抛开社会上政治正确陪笑
脸口是心非不说,大伙儿一般潜意识里认为是 savant 而不是 gifted。
这是因为大部分人内心对 gifted 的认可,最终是决定于 “theory-of-mind” (put
oneself on other person's show)。(当然还有 "extended object-permanence" 的
影响,不过这个是潜意识就不说了)。
而 neuro-typical 的人的 “theory-of-mind”,通常是把 "superior higher-order
thinking skill" 的,认可为 gifted;而把 "superior lower-order thinking skill
but wo/ any higher-order thinking skill" 的,归成 savant。

【在 t*******s 的大作中提到】
: 我觉得能进入AMC的孩子,一定是gifted的孩子,不能在归入bright 这个范畴了。你们
: 说说标准化考试比如SAT 99% 孩子绝对是非常聪明的,拿满分非常非常难,为什么大家
: 觉得很多这种孩子不算是gifted?

相关主题
少年智力开发觉得美帝数学进度慢的进来看一下
[转载] 为什么说usamo 简单分数应用题
二年级女儿的report我招,我是猪
进入Parenting版参与讨论
t******l
发帖数: 10908
91
我有另一个大胆假设。也可能 WIC-V 的 working memory 测试,测试的并不是
working memory 本身,测试的是 working memory 的 domain-general chunking 能力
(correlation 还是 causation 另说)。
这是因为我们的考试,包括智商测试的其他项目,更多的是 controlled environment
,也就是更多的是 routine fashion, predictable。
虽然大部分情况下,人的大脑有自我保护系统,即使在考试环境下,也不会过度发展
veli veli domain-specific 的 chunking(俗称,新东方型死记硬背红宝书狠刷题型
应考)。关于这点保护,看看大部分人考完 GRE 立马把红宝书给扔了烧了踩了埋了此
生不再碰,俗称 neuro-typical。
但上面这个保护是 “大部分情况”,并不是所有。如果这个保护机制失效,那么所有
跟周围考试风格类似的考试,可能都会被 veli veli domain-specific 的 chunking
给击破。而唯一的例外,就是给一个尼玛匪夷所思从来没见过的测试,虽然这玩意儿是
routine,但由于被测试者对这个测试闻所未闻,造成了 de facto 的 non-routine /
domain-general,结果 veli veli domain-specific 的 chunking 就挂了。

【在 s***n 的大作中提到】
: 是有一些孩子心算不行只能笔算,但这些孩子心算不行,大多是数学水平问题,不是
: working memory问题。真正working memory差的普通孩子,心算笔算都不行。
: 很多孩子倾向笔算只是因为习惯了笔算。GT孩子也一样,心算笔算大多是习惯养成问题
: ,和working memory强弱关系已经不大。我的观察,小娃图省事图快,多心算;大娃稳
: 重些了,会多用笔算。
: 一般认为ADHD的孩子working memory差。但很多ADHD的GT娃,可以用他们gifted的能力
: ,特别是在逻辑推理和大脑处理速度方面的优势,来弥补working memory的不足,所以
: 在表现上,ADHD的HGT孩子反而会显得心算和抢答快,快枪手,但他们的相对正确率(相
: 对其他GT娃)会随着题目难度加大快速下降。
:

s***n
发帖数: 1280
92
谢谢解释。你的GT标准没什么问题,只是更适合判定大娃。
很多Gifted小娃只是在智力水平上相比同龄儿童 Gifted,他们的情商和心理成熟度未
必超前。你说的1,2点其实在Gifted小娃中很普遍。
其实知道Gifted小娃的这些特点后,家长可以利用这些特点有意识地帮助GT小娃学习。
比如很多GT小娃自学的时候,喜欢冒进,不喜欢重复学过的内容或去做基础性的练习。
这样往往会造成这些GT孩子基础不牢靠,影响以后的发展。这个时候家长完全可以以半
游戏的形式或通过各种趣味奖励让GT孩子去做那些巩固基础的练习,帮助他们打好基础。
兴趣和擅长是相辅相成的。记得以前某个网友说过,如果某个项已孩子只有兴趣但并不
擅长,孩子的兴趣很容易在失败中逐渐消磨掉。而擅长的项目会让孩子不断得到内心的
满足和各种形式的鼓励。对于这些项目,家长只要正确的引导,会比较容易让孩子产生
真正的兴趣。而这种兴趣,哪怕一开始是追逐外在的东西,久而久之也可能内化为
passion。就像男女交朋友,一开始可能只是被外表的东西吸引,有好感,处得久了才
可能成passion,真爱。Gifted小娃对长项很多还在好感阶段,不一定表现出发自内心
的passion,但如果这种好感兴趣能持续到大娃阶段,就很可能成passion了。
你第3点提到的那些GT孩子其实就是你孩子,不过是他长大以后的样子。试想他小学三
年级就开始学高中代数了,那到了高中,他数学想拿A+还需要学吗?GT孩子没有不学就
会的,他们只是领悟力强学得快或学得早。
你说的第4点,不是所有GT孩子都有这个特点,但这绝对是给大多数GT孩子加分的特点


AoPS

【在 t*******s 的大作中提到】
: 你的问题问的很好!为什么我从来没觉得他属于gifted这一类,要么就是我对gifted这
: 个定义有误解,要么就是他真的不是,或者说顶多算是smart这个范畴内。主要观察如
: 下:
: 1、无论是khan Academy 还是AoPS, 我觉得他对上面的各种badge和论坛的 passion 要
: 远大于数学本身的兴趣。他从来没有表现出自已要主动去上面研究各种难题。比如AoPS
: pre-algebra 书上每章开始有21点算术,他不会坚持住自己研究各种可能性。他就是
: 喜欢在这些网站上留言啊,回答与数学无关的问题
: 2、他愿意一直在这些网站上继续学习,我认为很大一个原因是他想靠这种所谓超前学
: 习的机会来维持住自认为聪明的这个特征。他1到3年级一直上一个很好的私立,数学基
: 本上都是在比他高一年级的班上,另外有一个老师专门给他上些乱七八糟的物理什么的

t******l
发帖数: 10908
93
veli veli domain-specific 的问题,确实是考试,至少是 lower-order 低年级的考
试,一点儿问题也没有,因为更多是 routine,“low-order 刷题” 的效率高。
但我们每天上学走路时,惹个把花花草草这种简单的事儿,倒是 non-routine,因为花
花草草的反应千奇百怪。veli veli domain-specific 对这种就会显得无所适从、手足
无措。
或者如果打一个比方的话,veli veli domain-specific 的问题,就好比 GRE 逻辑拿
满分,但日常生活开车买菜侃大山的逻辑真心不行的,那种。。。// 哈哈,super
fast run 了,请勿追杀,谢谢。。。

environment

【在 t******l 的大作中提到】
: 我有另一个大胆假设。也可能 WIC-V 的 working memory 测试,测试的并不是
: working memory 本身,测试的是 working memory 的 domain-general chunking 能力
: (correlation 还是 causation 另说)。
: 这是因为我们的考试,包括智商测试的其他项目,更多的是 controlled environment
: ,也就是更多的是 routine fashion, predictable。
: 虽然大部分情况下,人的大脑有自我保护系统,即使在考试环境下,也不会过度发展
: veli veli domain-specific 的 chunking(俗称,新东方型死记硬背红宝书狠刷题型
: 应考)。关于这点保护,看看大部分人考完 GRE 立马把红宝书给扔了烧了踩了埋了此
: 生不再碰,俗称 neuro-typical。
: 但上面这个保护是 “大部分情况”,并不是所有。如果这个保护机制失效,那么所有

t******l
发帖数: 10908
94
那 high-order chunking skill 会不会影响 attention。其实一般人并不了解 ADHD/
ADD,但就是对于 neuro-typical 的人群而言,chunking 对于大脑保持 focus 非常重
要。因为人类大脑的自我保护机制,对于 scattered / un-comprehensible info,大
脑会自动放弃 focus,以免超频太多而烧毁。
或者那前面的实际例子,下面的 before / after chunking,哪个大脑更加能保持
focus,更加记得住大概,并且用大概信息 predict 以及 proper shift focus,吧:
========= Before chunking ==================================
"If the cake is from France, then it has more sugar if it is made with
chocolate than if it is made with cream, but if the cake is from Italy, then
it has more sugar if it is made with cream than if it is made of chocolate"
======== Lower-order chunking ==============================
In France, chocolate cake has more sugar than cream cake.
In Italy, chocolate cake has less sugar than cream cake.
======== Higher-order chunking, can be used for prediction,
focus shifting, long-term memory conversion =========
(higher order pragmatics "chunking"):
Different location has different order. (Order is function/LUT of location).
(lower order pragmatics "chunking"):
Location is France vs Italy.
Order is the type of cake (chocolate / cream) ordered by sugar percentage.

【在 t******l 的大作中提到】
: veli veli domain-specific 的问题,确实是考试,至少是 lower-order 低年级的考
: 试,一点儿问题也没有,因为更多是 routine,“low-order 刷题” 的效率高。
: 但我们每天上学走路时,惹个把花花草草这种简单的事儿,倒是 non-routine,因为花
: 花草草的反应千奇百怪。veli veli domain-specific 对这种就会显得无所适从、手足
: 无措。
: 或者如果打一个比方的话,veli veli domain-specific 的问题,就好比 GRE 逻辑拿
: 满分,但日常生活开车买菜侃大山的逻辑真心不行的,那种。。。// 哈哈,super
: fast run 了,请勿追杀,谢谢。。。
:
: environment

t******l
发帖数: 10908
95
回到数学问题上,比如解析几何直线的 point-slope 概念(其中 slope 是 rise-over
-run),其实就是 chunking。
而区别于 “死记硬背直线方程型 chunking” 的区别,就是 domain-specific vs
domain-
general。争议的说,point-slope 的 chunking,可以一直延伸到 微积分/运动学/甚
至曲面几何等等的 perceptual processing。
而死记硬背直线方程的 chunking,you know,高考考完就可以 decay 了。// run
类似的,complete a square vs 二次方程求根公式。
当然,domain-specific chunking 也有其优点,最大的优点就是迅速准确。
其实我自己高考绝对不会去 complete a square 或者 point-slope,直接考前把公式
给背个滚瓜烂熟,然后高考备考一年把题目给刷成滚瓜烂熟用脊髓就可以刷刷刷做完。
否则我就是大沙比。
但高考一旦考完,我立马跑到数学老师那里,把那些公式统统还给高中数学老师。而高
中数学老师此时会心地翘起了两个大拇指,曰:“neuro-typical”。。。

then

【在 t******l 的大作中提到】
: 那 high-order chunking skill 会不会影响 attention。其实一般人并不了解 ADHD/
: ADD,但就是对于 neuro-typical 的人群而言,chunking 对于大脑保持 focus 非常重
: 要。因为人类大脑的自我保护机制,对于 scattered / un-comprehensible info,大
: 脑会自动放弃 focus,以免超频太多而烧毁。
: 或者那前面的实际例子,下面的 before / after chunking,哪个大脑更加能保持
: focus,更加记得住大概,并且用大概信息 predict 以及 proper shift focus,吧:
: ========= Before chunking ==================================
: "If the cake is from France, then it has more sugar if it is made with
: chocolate than if it is made with cream, but if the cake is from Italy, then
: it has more sugar if it is made with cream than if it is made of chocolate"

t******l
发帖数: 10908
96
刚才敲的有点太快太乱,补充/修正一下就是:
"domain-specific vs domain-general" 以及 "routine vs non-routine" 都是表象。
其背后的实质因子,应该是 lower-order chunking vs higher-order chunking。
举个例子就是 superior on "lexicon (词汇) chunking", but poorly on "pragmatic
(上下文语境) chunking",如果这种不对称的情况如果严重的话,可能会造成问题。
先不回去改原帖了。

over

【在 t******l 的大作中提到】
: 回到数学问题上,比如解析几何直线的 point-slope 概念(其中 slope 是 rise-over
: -run),其实就是 chunking。
: 而区别于 “死记硬背直线方程型 chunking” 的区别,就是 domain-specific vs
: domain-
: general。争议的说,point-slope 的 chunking,可以一直延伸到 微积分/运动学/甚
: 至曲面几何等等的 perceptual processing。
: 而死记硬背直线方程的 chunking,you know,高考考完就可以 decay 了。// run
: 类似的,complete a square vs 二次方程求根公式。
: 当然,domain-specific chunking 也有其优点,最大的优点就是迅速准确。
: 其实我自己高考绝对不会去 complete a square 或者 point-slope,直接考前把公式

t******l
发帖数: 10908
97
“去看这种history 的东西又不需要基础”
我记起来我家大娃在三年级最初看的 biography 的系列是 “History Maker Bios”。
BTW:这里的 Bios 是指 biography(传记),而不是 biology (生物)。// run
“History Maker Bios”,这套书的 lexile 相对较低,记忆中是 lexile 400L -
700L 不等。大娃她在 “The Diary of a Young Girl: Anne Frank” (lexile 1080L)
之前就看了。
我家大娃三年级看的娃版 History,lexile 一般是 800L 左右。在 Anne Frank 那本
之后几个月后才看的。
当然现在她也不看娃版 history 了,过了那个村了。
我的意思是,对我们大人来说,“看这种history 的东西又不需要基础”。但认知过程
中的娃娃不一样,除了要 leixcon / syntax / semantics 的基础以外,还需要
pragmatics / schema 的基础。而且 lexile 和 pragmatics / schema 不是一回事,
会有交叉现象。

【在 t*******s 的大作中提到】
: 嗯,同意你的。小朋友认为的重要与否不一样。我特别纠结他呆在YouTube 上看
: Minecraft,每次看到都试图说服他要不看看什么world history 之类的video, 既然他
: 阅读能力很强,去看这种history 的东西又不需要基础,但是他只能坚持10分钟就又回
: 到Minecraft 上了。所以我家不给他computer 的password,iPad 也是不给的。我现在
: 对这种电子设备是敌对态度,但是还不能明显表现出来我恨这些game.
:
: 行。
: ,并

t******l
发帖数: 10908
98
lexile 和 pragmatics / schema 的交叉现象,不仅仅在 non-fiction。fiction 也很
多。
比如我印象里比较确切的,就是 "A wrinkle in time" 这本(我家大娃小学喜欢的
fiction 实在不多,这对我也成了一个 problem)。虽然这本 lexile 相对较低,但我
家大娃是在很多 higher lexile 的 fiction 后面才读的。

【在 t******l 的大作中提到】
: “去看这种history 的东西又不需要基础”
: 我记起来我家大娃在三年级最初看的 biography 的系列是 “History Maker Bios”。
: BTW:这里的 Bios 是指 biography(传记),而不是 biology (生物)。// run
: “History Maker Bios”,这套书的 lexile 相对较低,记忆中是 lexile 400L -
: 700L 不等。大娃她在 “The Diary of a Young Girl: Anne Frank” (lexile 1080L)
: 之前就看了。
: 我家大娃三年级看的娃版 History,lexile 一般是 800L 左右。在 Anne Frank 那本
: 之后几个月后才看的。
: 当然现在她也不看娃版 history 了,过了那个村了。
: 我的意思是,对我们大人来说,“看这种history 的东西又不需要基础”。但认知过程

t*******s
发帖数: 48
99
你下面这段看法我太同意了!!! 你一定是观察过类似的多个孩子!我这几年就觉得他是
在走马观花的在网上自学。数学是一个例子,他学其它的也是一样,非常快,不求质量
,而且讨厌有人在旁边教他。任何学习他基本上拒绝重复,除非他意识他的的方法不正
确需要改进,否则他是不会听大人意见举一反三的。有时候即便是他不会做,别人给他
思路他也拒绝接受,认为你的办法不好。我经常反复说这个社会需要hard working and
quality jobs but not smart and speed. 估计听多了他就直接ignore 我这种声音。
我认为的HGT孩子 就是他们立刻能高效率高准确率的走到答案的终点处,他肯定不是这
种。

******比如很多GT小娃自学的时候,喜欢冒进,不喜欢重复学过的内容或去做基础性的
练习。这样往往会造成这些GT孩子基础不牢靠,影响以后的发展。******

【在 s***n 的大作中提到】
: 谢谢解释。你的GT标准没什么问题,只是更适合判定大娃。
: 很多Gifted小娃只是在智力水平上相比同龄儿童 Gifted,他们的情商和心理成熟度未
: 必超前。你说的1,2点其实在Gifted小娃中很普遍。
: 其实知道Gifted小娃的这些特点后,家长可以利用这些特点有意识地帮助GT小娃学习。
: 比如很多GT小娃自学的时候,喜欢冒进,不喜欢重复学过的内容或去做基础性的练习。
: 这样往往会造成这些GT孩子基础不牢靠,影响以后的发展。这个时候家长完全可以以半
: 游戏的形式或通过各种趣味奖励让GT孩子去做那些巩固基础的练习,帮助他们打好基础。
: 兴趣和擅长是相辅相成的。记得以前某个网友说过,如果某个项已孩子只有兴趣但并不
: 擅长,孩子的兴趣很容易在失败中逐渐消磨掉。而擅长的项目会让孩子不断得到内心的
: 满足和各种形式的鼓励。对于这些项目,家长只要正确的引导,会比较容易让孩子产生

t*******s
发帖数: 48
100
你有没有引导她精度?比如说文章后面有问题,需要选答案?否则怎么衡量孩子的阅读
quality?

【在 t******l 的大作中提到】
: lexile 和 pragmatics / schema 的交叉现象,不仅仅在 non-fiction。fiction 也很
: 多。
: 比如我印象里比较确切的,就是 "A wrinkle in time" 这本(我家大娃小学喜欢的
: fiction 实在不多,这对我也成了一个 problem)。虽然这本 lexile 相对较低,但我
: 家大娃是在很多 higher lexile 的 fiction 后面才读的。

相关主题
老师说我女儿是gifted kid (转载)NIH 证实天才和疯子都源于同一型基因:DARPP-32
忍不住上来问问,我娃的数学是啥水平(6岁)?放养妈进来看看
陶天才论天才 (转载)最新研究证明心算AIME大幅度促进心理和情感健康
进入Parenting版参与讨论
C********e
发帖数: 2327
101
这个verbal真是强,肯定特爱读书

【在 t*******s 的大作中提到】
: 仔细再看了看报告,的确是测5个部分。每部分包含两个单项。分值如下。仔细想想,
: part 4 的确符合家长多年观察和从前老师的各种抱怨。其他分值高的部分,家长没明
: 显觉得他本身match 这些分值。希望这里给各个家长一个借鉴
: Part 1: Verbal Comprehension ( Similarities & Vocabulary) - 155/>99.9%
: Part 2: Visual Spatial (Block Design & Visual Puzzles) – 138/99%
: Part 3: Fluid Reasoning (Matrix Reasoning & Figure Weights) – 144/8
: Part 4: Working Memory (Digit Span & Picture Span) – 107/68%
: Part 5: Processing Speed (Symbol Search & Coding) – 129/97%

t******l
发帖数: 10908
102
那两年我 track 大娃的 progress 的(否则我也不会有那些记忆)。
但我的风格是 “乱中取胜”(play the middle game like a magician)。我没有啥
定式问题啥的,比较随心。就是跟她闲聊,但偶尔也会做某一小段我所认为的 text
analysis 给娃看(通常我 sense 有需要的时候)。
如果我自己搞不清楚,也会把娃的书拿过来看一下再聊,但这是 on-demand,选择有需
要的页数,而不是事先把娃书看一遍。
fiction 我基本不管,因为有学校管。但 fiction 的话,我先看 wiki 上故事介绍(
严重不建议娃自己这么做。这是对于大人没时间读娃的整本书,先读一下故事介绍后,
可以 on-demand 跳页看)。

【在 t*******s 的大作中提到】
: 你有没有引导她精度?比如说文章后面有问题,需要选答案?否则怎么衡量孩子的阅读
: quality?

C********e
发帖数: 2327
103
大家对gifted标准不一样吧,是不是清北里的学生都是gifted的?肯定都是聪明的,但
是算不算gifted就得看个人定义了

【在 t*******s 的大作中提到】
: 我觉得能进入AMC的孩子,一定是gifted的孩子,不能在归入bright 这个范畴了。你们
: 说说标准化考试比如SAT 99% 孩子绝对是非常聪明的,拿满分非常非常难,为什么大家
: 觉得很多这种孩子不算是gifted?

t*******s
发帖数: 48
104
我猜如果是正常途径进入top2的非北上广及二线城市外的孩子基本上算是gifted了吧。
因为这个范围内的孩子教育资源比较弱一些

【在 C********e 的大作中提到】
: 大家对gifted标准不一样吧,是不是清北里的学生都是gifted的?肯定都是聪明的,但
: 是算不算gifted就得看个人定义了

t******l
发帖数: 10908
105
其实教育资源强的地方,按人口比例而言,gifted 更多一些。
因为崇尚 “老实大量读经” 的毕竟是少数 neuro-atypical 家长。绝大部分 neuro-
typical 家长的家长,内心潜意识还是希望用手头教育资源推 giftedness (higher
order skill) 而不是 savantedness (lower order skill)。
如果拿现实的例子的话,如果没有这边的 County / City library system 的话,那我
娃可能也只能要么 “老实大量读武侠”,要么干脆就厌学不读书了。

:我猜如果是正常途径进入top2的非北上广及二线城市外的孩子基本上算是gifted了吧
。因为这个范围内的孩子教育资源比较弱一些
t******l
发帖数: 10908
106
另外对于 neuro-typical 的家长,想知道自家娃是 gifted or not,这是徒劳的。因
为大脑的 self-awareness。
但另一方面,neuro-typical 家长的大脑的 self-awareness,会促使家长 “推
giftedness” 而不是 “推 savantedness”。这甚至能相当程度的抵制追藤社会的
peer pressure。也是属于大脑的自我保护。
而从这个角度说,neuro-typical 的家长,在决定是不是把娃送去 GT 学校 GT 班的时
候,潜意识里是跟据自家娃对 "普通 vs GT" 两种不同环境的 fitness 的优
劣比较,而不是根据自己对自家娃的 giftedness 的程度的判定。
C********e
发帖数: 2327
107
neuro-typical 是啥样的?

【在 t******l 的大作中提到】
: 另外对于 neuro-typical 的家长,想知道自家娃是 gifted or not,这是徒劳的。因
: 为大脑的 self-awareness。
: 但另一方面,neuro-typical 家长的大脑的 self-awareness,会促使家长 “推
: giftedness” 而不是 “推 savantedness”。这甚至能相当程度的抵制追藤社会的
: peer pressure。也是属于大脑的自我保护。
: 而从这个角度说,neuro-typical 的家长,在决定是不是把娃送去 GT 学校 GT 班的时
: 候,潜意识里是跟据自家娃对 "普通 vs GT" 两种不同环境的 fitness 的优
: 劣比较,而不是根据自己对自家娃的 giftedness 的程度的判定。

t******l
发帖数: 10908
108
这个在 “没有压力(stress)” 的条件,又揭示了军医学测试的 selection bias,也
就是说军医学拉人去智商测试这个本身,造成了 implicit stress。这使得大脑的
working memory switch mode,更多 work on short-term explicit item,或多或少
的差别。
打个比方就是皇上要召邻家自然美女进宫,于是军医们去 California DMV 调加州驾照
照片数据库人肉搜索,而不是去三里屯街上以罗马假日的风情搞偷拍。
于是皇上一看,尼玛挑出来的几乎清一色北影厂二围三线整容女。龙颜大怒,把军医学
给拉出端午们 tjjtds。。。

stress
).

【在 t******l 的大作中提到】
: 我觉得更有意思的是,大脑的 working memory "chunking" 并没有停留下下面这步:
: In France, chocolate cake has more sugar than cream cake.
: In Italy, chocolate cake has less sugar than cream cake.
: 我们想象在没有压力(stress)的情况下,也就是没有考前死记硬背的要求。(stress
: 确实影响 working memory)。大脑会继续 "pragmatics-like chunking" 成下面两个
: level 的信息,并且放入 long-term memory:
: (higher order pragmatics "chunking"):
: Different location has different order. (Order is function/LUT of location).
: (lower order pragmatics "chunking"):
: Location is France vs Italy.

t******l
发帖数: 10908
109
然后这个最搞笑的,我觉得就是把 centralized phonological loop 看成核心部分。
http://en.wikipedia.org/wiki/Baddeley%27s_model_of_working_memory#Phonological_loop
其实我并不反对 working memory 依赖于 neuron path looping 的概念。但是
centralized phonological loop 如果是 working memory 核心部分的话,这在进化优
势上根本说不通。
你说史前 hunter-gather 口中念念有词 “牛二定律标枪击中狮子” 三百遍后,我看
标枪不用出手了,直接已经是狮子的点心了。

【在 t******l 的大作中提到】
: 这个在 “没有压力(stress)” 的条件,又揭示了军医学测试的 selection bias,也
: 就是说军医学拉人去智商测试这个本身,造成了 implicit stress。这使得大脑的
: working memory switch mode,更多 work on short-term explicit item,或多或少
: 的差别。
: 打个比方就是皇上要召邻家自然美女进宫,于是军医们去 California DMV 调加州驾照
: 照片数据库人肉搜索,而不是去三里屯街上以罗马假日的风情搞偷拍。
: 于是皇上一看,尼玛挑出来的几乎清一色北影厂二围三线整容女。龙颜大怒,把军医学
: 给拉出端午们 tjjtds。。。
:
: stress

t******l
发帖数: 10908
110
这个打个不确切的比方,就好比普通的通用中央处理器 CPU,需要的时候,也可以用
SSE 向量指令集,加几条超级标量流水线,算个 FFT 也能算出来。稍微费点电就是了。
但是军医们一看见 FFT 结果出来了,于是你一言我一语的热烈讨论,该通用中央处理
器 CPU 里的硬件 FFT 是如何运作的。。。于是马工系教授跟旁边 TA 嘀咕了一句,“
我看这个班的体系结构课,你就全给零蛋吧”。。。// super fast run

【在 t******l 的大作中提到】
: 然后这个最搞笑的,我觉得就是把 centralized phonological loop 看成核心部分。
: http://en.wikipedia.org/wiki/Baddeley%27s_model_of_working_memory#Phonological_loop
: 其实我并不反对 working memory 依赖于 neuron path looping 的概念。但是
: centralized phonological loop 如果是 working memory 核心部分的话,这在进化优
: 势上根本说不通。
: 你说史前 hunter-gather 口中念念有词 “牛二定律标枪击中狮子” 三百遍后,我看
: 标枪不用出手了,直接已经是狮子的点心了。

相关主题
最新研究证明心算AIME大幅度促进心理和情感健康9岁, 如何报名考amc10
来看一下各位的观点求科普: 奥数 vs. Math Olympiad vs. Math Count vs. Math circle vs. 超前学数学
是不是任我儿子继续迷象棋有明天考AMC 8的吗?
进入Parenting版参与讨论
t******l
发帖数: 10908
111
我觉得要研究 working memory 的真实高层 function,首先得考虑进化优势。
从进化优势的角度,应该去三里屯偷拍 hunter-gather 群体协作,在瞬息万变不太可
预知的 non-routine 的 mother nature 环境下,如何用标枪打下野牛然后大块吃肉,
这过程中,大脑的 working memory 是如何运作的。
当然,上面这个比较难。三里屯美女会告骚扰和肖像权。。。所以另一个办法就是,看
neuro-typical 的刷题不多的,如何在与蒋军的斗争中学习,在不限时间的环境下,
甚至转头睡一觉也没关系,如何学习求解自己闻所未闻的从来没见过题型的 “AIME 15
道心算填空题”。

了。

【在 t******l 的大作中提到】
: 这个打个不确切的比方,就好比普通的通用中央处理器 CPU,需要的时候,也可以用
: SSE 向量指令集,加几条超级标量流水线,算个 FFT 也能算出来。稍微费点电就是了。
: 但是军医们一看见 FFT 结果出来了,于是你一言我一语的热烈讨论,该通用中央处理
: 器 CPU 里的硬件 FFT 是如何运作的。。。于是马工系教授跟旁边 TA 嘀咕了一句,“
: 我看这个班的体系结构课,你就全给零蛋吧”。。。// super fast run

t******l
发帖数: 10908
112
从这个角度看,要在三里屯打野牛的变化环境中生存,大脑高层 working memory 首先
功能并不是死机硬背电话号码死循环,而是 chunking。。。从一大堆风吹草动里,
chunking 出那只大胸野牛,然后 chunking 出短句通知手持炮筒的同伴,以及大致方
向特征。。。然后手持炮筒的同伴利用语言交流的 clue / cue,在一大堆动物里
chunking 出那只大胸野牛,然后差不多就手一摁咔嚓一声,好像有点歪。。。立马打
开 LCD chunking 一下看看能不能回去剪辑一下就好。。。看看好像行。。。于是不在
那只大胸野牛上继续纠缠,把注意力转向第二只可能的小峰娇牛。。。

15

【在 t******l 的大作中提到】
: 我觉得要研究 working memory 的真实高层 function,首先得考虑进化优势。
: 从进化优势的角度,应该去三里屯偷拍 hunter-gather 群体协作,在瞬息万变不太可
: 预知的 non-routine 的 mother nature 环境下,如何用标枪打下野牛然后大块吃肉,
: 这过程中,大脑的 working memory 是如何运作的。
: 当然,上面这个比较难。三里屯美女会告骚扰和肖像权。。。所以另一个办法就是,看
: neuro-typical 的刷题不多的,如何在与蒋军的斗争中学习,在不限时间的环境下,
: 甚至转头睡一觉也没关系,如何学习求解自己闻所未闻的从来没见过题型的 “AIME 15
: 道心算填空题”。
:
: 了。

c***x
发帖数: 1826
113

础。
挺好,挺好,这个版面至少还有郭靖镇守襄阳城。
昨天去看了一部电影 《Accountant》,原来完全当做娱乐片看的,看完了倒是发现浓
浓的父子情。这个楼里的父母,也许有兴趣可以一看。

【在 s***n 的大作中提到】
: 谢谢解释。你的GT标准没什么问题,只是更适合判定大娃。
: 很多Gifted小娃只是在智力水平上相比同龄儿童 Gifted,他们的情商和心理成熟度未
: 必超前。你说的1,2点其实在Gifted小娃中很普遍。
: 其实知道Gifted小娃的这些特点后,家长可以利用这些特点有意识地帮助GT小娃学习。
: 比如很多GT小娃自学的时候,喜欢冒进,不喜欢重复学过的内容或去做基础性的练习。
: 这样往往会造成这些GT孩子基础不牢靠,影响以后的发展。这个时候家长完全可以以半
: 游戏的形式或通过各种趣味奖励让GT孩子去做那些巩固基础的练习,帮助他们打好基础。
: 兴趣和擅长是相辅相成的。记得以前某个网友说过,如果某个项已孩子只有兴趣但并不
: 擅长,孩子的兴趣很容易在失败中逐渐消磨掉。而擅长的项目会让孩子不断得到内心的
: 满足和各种形式的鼓励。对于这些项目,家长只要正确的引导,会比较容易让孩子产生

t******l
发帖数: 10908
114
所以从这个角度说,心算 AIME 15 道天空题,之所以要心算,既不是放着草稿纸不用
,也不是心里 phonological loop 九九乘法表死循环。。。这心算真正的目的,是在
整 higher order non-routine chunking 来对付闻所未闻的题目。
而这种 higher order non-routine chunking 应该还分成前台和后台。前台是按既定
方针进行 explicit 的 chunking,干实事。而后台 implicit memory 的任务就是盯的
前台的行为,同时大略分析前台扔出来的垃圾。。。如果发现此路看起来不通,前台还
在走火入魔,就一把拉下消防报警器,于是前台考试到半当中被迫退出放风。
而回到为啥要心算。。。我就问上面这种 chunking 写得下草稿纸么?

【在 t******l 的大作中提到】
: 从这个角度看,要在三里屯打野牛的变化环境中生存,大脑高层 working memory 首先
: 功能并不是死机硬背电话号码死循环,而是 chunking。。。从一大堆风吹草动里,
: chunking 出那只大胸野牛,然后 chunking 出短句通知手持炮筒的同伴,以及大致方
: 向特征。。。然后手持炮筒的同伴利用语言交流的 clue / cue,在一大堆动物里
: chunking 出那只大胸野牛,然后差不多就手一摁咔嚓一声,好像有点歪。。。立马打
: 开 LCD chunking 一下看看能不能回去剪辑一下就好。。。看看好像行。。。于是不在
: 那只大胸野牛上继续纠缠,把注意力转向第二只可能的小峰娇牛。。。
:
: 15

s***n
发帖数: 1280
115
我比较熟的GT娃,大多是Highly Gifted 或 Exceptionally Gifted 的数学娃。他们碰
到难题时,大致有两种不同的对策。一些孩子会很快转向老师或教材或他人寻求解题思
路,技巧甚至答案;另一些孩子会很努力地自己找思路,不喜欢别人的帮助和提示。这
两个对策各有优劣,但对GT小娃我更希望看到后者。因为后者更愿意用脑思考,更愿意
挑战自己,这些孩子的大脑会有更多的机会锻炼。从长远的角度看,后者更可能日后成
功。
自然也有些GT孩子拒绝别人主要是因为自以为是。这是一个非天才专有的通病。不过我
们不能因为孩子有了这个通病,就无视孩子的天分。

and

【在 t*******s 的大作中提到】
: 你下面这段看法我太同意了!!! 你一定是观察过类似的多个孩子!我这几年就觉得他是
: 在走马观花的在网上自学。数学是一个例子,他学其它的也是一样,非常快,不求质量
: ,而且讨厌有人在旁边教他。任何学习他基本上拒绝重复,除非他意识他的的方法不正
: 确需要改进,否则他是不会听大人意见举一反三的。有时候即便是他不会做,别人给他
: 思路他也拒绝接受,认为你的办法不好。我经常反复说这个社会需要hard working and
: quality jobs but not smart and speed. 估计听多了他就直接ignore 我这种声音。
: 我认为的HGT孩子 就是他们立刻能高效率高准确率的走到答案的终点处,他肯定不是这
: 种。
:
: ******比如很多GT小娃自学的时候,喜欢冒进,不喜欢重复学过的内容或去做基础性的

t*******s
发帖数: 48
116
我家孩子应该是你下面讲的第一种:就是遇到难题他会立马问,但是随着你的hint往下
走,他会非常快的stop你的hint 然后自己继续后面的步骤;也有一种情况就是如果你
的hint一开始和他不对路,他立马拒绝听下去。他多次在学校纠正老师,这点就会引起
老师反感。我一直在寻求如何能使他耐心听完别人的讲解不打断,至今没有找出合适的
办法,所以这一年我基本上不再回答比较复杂的academic 问题,他都是自己Google 或
其它途径。从你的看法,我觉得你太强了,的确是认真观察过孩子

【在 s***n 的大作中提到】
: 我比较熟的GT娃,大多是Highly Gifted 或 Exceptionally Gifted 的数学娃。他们碰
: 到难题时,大致有两种不同的对策。一些孩子会很快转向老师或教材或他人寻求解题思
: 路,技巧甚至答案;另一些孩子会很努力地自己找思路,不喜欢别人的帮助和提示。这
: 两个对策各有优劣,但对GT小娃我更希望看到后者。因为后者更愿意用脑思考,更愿意
: 挑战自己,这些孩子的大脑会有更多的机会锻炼。从长远的角度看,后者更可能日后成
: 功。
: 自然也有些GT孩子拒绝别人主要是因为自以为是。这是一个非天才专有的通病。不过我
: 们不能因为孩子有了这个通病,就无视孩子的天分。
:
: and

t******l
发帖数: 10908
117
这个跟天才与否无关,这个是 theory of mind,也就为了适应社会而 tell white lie。
事实上,说真心话,99.99% 的高中数学老师,数学不会比班上前几名的娃的数学更好
,更不要说跟 USAMO 500 强比。这种情况数学老师其实就是个 education service,
听讲课比看
书更容易,效率更高而已。
当然个别大学老师确实很自以为是,其实大学生都知道那些自以为是的老师的课,听了
比不听更糊涂。真心的说。。。其实那型老师就是 “早会”,外加老师事先知道答案
属于作弊级,其实不真懂,或者说,真不懂。

:我家孩子应该是你下面讲的第一种:就是遇到难题他会立马问,但是随着你的hint往
下走,他会非常快的stop你的hint 然后自己继续后面的步骤;也有一种情况就是如果你
:的hint一开始和他不对路,他立马拒绝听下去。他多次在学校纠正老师,这点就会引
起老师反感。我一直在寻求如何能使他耐心听完别人的讲解不打断,至今没有找出合适
的办法,所以这一年我基本上不再回答比较复杂的academic 问题,他都是自己Google
或其它途径。从你的看法,我觉得你太强了,的确是认真观察过孩子??
t******l
发帖数: 10908
118
我觉得简单的就是 apply social rules。你要听别人讲解就不要随便打断,你要是觉
得别人的讲解不值得听,那就不要去要求讲解,你自己一个人抗就是了。
当然如果同时还想出成绩、比赛拿高分的话,有难度。

:我家孩子应该是你下面讲的第一种:就是遇到难题他会立马问,但是随着你的hint往
下走,他会非常快的stop你的hint 然后自己继续后面的步骤;也有一种情况就是如果你
:的hint一开始和他不对路,他立马拒绝听下去。他多次在学校纠正老师,这点就会引
起老师反感。我一直在寻求如何能使他耐心听完别人的讲解不打断,至今没有找出合适
的办法,所以这一年我基本上不再回答比较复杂的academic 问题,他都是自己Google
或其它途径。从你的看法,我觉得你太强了,的确是认真观察过孩子??
t******l
发帖数: 10908
119
我个人的看法,父母不是专业教师,主要的帮助还是小学阶段的打基础。
另外父母作为不是专业教师,对于 AMC 10 Problem 15 以后的题目,要讲题的话一定
要自己把题做一遍。即使没做出来,也要花上足够的时间做题后才看答案。千万不要看
着 non-routine 题目的答案给娃讲解,否则这等于欺骗,实话实说。
专职教师不一样,因为专业教师要参加教育方面的培训,而不仅仅知道题目的答案就可
以了。换言之,专职教师不仅知道答案,还得知道教学法。这就是为啥高中数学老师能
教水平比自己高很多的数学尖子。

:我家孩子应该是你下面讲的第一种:就是遇到难题他会立马问,但是随着你的hint往
下走,他会非常快的stop你的hint 然后自己继续后面的步骤;也有一种情况就是如果你
:的hint一开始和他不对路,他立马拒绝听下去。他多次在学校纠正老师,这点就会引
起老师反感。我一直在寻求如何能使他耐心听完别人的讲解不打断,至今没有找出合适
的办法,所以这一年我基本上不再回答比较复杂的academic 问题,他都是自己Google
或其它途径。从你的看法,我觉得你太强了,的确是认真观察过孩子??
t*******s
发帖数: 48
120
同意!所以四年级后我不再太多参与他自学的事情。这里再赞一下AoPS, 我看过几次他
们上课的笔记,偶尔和他一起上过几次课,真心觉得不错!太符合大脑不安分的孩子了

果你

【在 t******l 的大作中提到】
: 我个人的看法,父母不是专业教师,主要的帮助还是小学阶段的打基础。
: 另外父母作为不是专业教师,对于 AMC 10 Problem 15 以后的题目,要讲题的话一定
: 要自己把题做一遍。即使没做出来,也要花上足够的时间做题后才看答案。千万不要看
: 着 non-routine 题目的答案给娃讲解,否则这等于欺骗,实话实说。
: 专职教师不一样,因为专业教师要参加教育方面的培训,而不仅仅知道题目的答案就可
: 以了。换言之,专职教师不仅知道答案,还得知道教学法。这就是为啥高中数学老师能
: 教水平比自己高很多的数学尖子。
:
: :我家孩子应该是你下面讲的第一种:就是遇到难题他会立马问,但是随着你的hint往
: 下走,他会非常快的stop你的hint 然后自己继续后面的步骤;也有一种情况就是如果你

相关主题
算不算有数学天分[转载] 为什么说usamo 简单
大家怎么推孩子的弱项?二年级女儿的report
少年智力开发觉得美帝数学进度慢的进来看一下
进入Parenting版参与讨论
s***n
发帖数: 1280
121
其实看你的描述,我觉得你孩子更像第二种情况 - 喜欢用脑思考的孩子。他碰到难题
发问,可能是因为题目对他来说太难了,也可能是因为没学过相应的基础知识或学得不
扎实。指导这样的孩子,关键是你的hint不能给得太多,要点到为止。这些孩子真正享
受的是战胜挑战后的成就感。所以他们喜欢智力脑力上的挑战,也倾向于独立地去战胜
这些挑战。他们碰到难题的时候,你给的hint越多,他的fun和成就感就越少。你孩子
对hint的反应是很正常的第二类HGT小娃的反应。你这篇文章提到一些情况其实更多地
是指导老师的问题。他们没有针对GT小娃的特点来教学。
自然很多事情都有两面性。完全拒绝hint,拒绝学习别人,也会阻碍孩子的发展,特别
是那些源于自大的粗暴拒绝。我觉得解决办法主要是带孩子去见世面。参加各种竞赛见
见难题,接触下那些给他hint都不容易做出来的题,意识到自己的不足;参加各种数学
活动和团体竞赛,见识下其他数学娃和数学高手。这些数学娃在一起解题会有很多讨论
。讨论中会有很多hint出来,但这些hint不是教学式的hint,而是讨论探索性质的hint
。GT娃对这类hint的排斥性要小很多。久而久之,他们会对hint本身会不那么反感了,
也能更好更坦然地接受教学式hint。
以前讨论构建式教育,我提到构建式教育适合于GT娃,就是因为构建式教育推崇以讨论
地形式来构建,把很多教学hint有意识地化为讨论探索性质的hint,更适于第二类GT娃
的特点。

;1

【在 t*******s 的大作中提到】
: 我家孩子应该是你下面讲的第一种:就是遇到难题他会立马问,但是随着你的hint往下
: 走,他会非常快的stop你的hint 然后自己继续后面的步骤;也有一种情况就是如果你
: 的hint一开始和他不对路,他立马拒绝听下去。他多次在学校纠正老师,这点就会引起
: 老师反感。我一直在寻求如何能使他耐心听完别人的讲解不打断,至今没有找出合适的
: 办法,所以这一年我基本上不再回答比较复杂的academic 问题,他都是自己Google 或
: 其它途径。从你的看法,我觉得你太强了,的确是认真观察过孩子

t******l
发帖数: 10908
122
其实网上答案本身就是 hint。
所以父母要给 hint 的话,就算娃愿意花时间在数学上,那父母给 hint 首先也得
create value。如果父母给的 hint 跟网上能查到答案半斤八两,大娃一般就直接对网
上答案就是了。
所以一般都是小娃比大娃看起来更愿意接受 hint,但其实真相是小娃大字都不识几个。

【在 s***n 的大作中提到】
: 其实看你的描述,我觉得你孩子更像第二种情况 - 喜欢用脑思考的孩子。他碰到难题
: 发问,可能是因为题目对他来说太难了,也可能是因为没学过相应的基础知识或学得不
: 扎实。指导这样的孩子,关键是你的hint不能给得太多,要点到为止。这些孩子真正享
: 受的是战胜挑战后的成就感。所以他们喜欢智力脑力上的挑战,也倾向于独立地去战胜
: 这些挑战。他们碰到难题的时候,你给的hint越多,他的fun和成就感就越少。你孩子
: 对hint的反应是很正常的第二类HGT小娃的反应。你这篇文章提到一些情况其实更多地
: 是指导老师的问题。他们没有针对GT小娃的特点来教学。
: 自然很多事情都有两面性。完全拒绝hint,拒绝学习别人,也会阻碍孩子的发展,特别
: 是那些源于自大的粗暴拒绝。我觉得解决办法主要是带孩子去见世面。参加各种竞赛见
: 见难题,接触下那些给他hint都不容易做出来的题,意识到自己的不足;参加各种数学

t******l
发帖数: 10908
123
但从进化优势的角度,还是存在第三种可能性,也就是人类大脑在 hunter-gatherer
以后继续进化。想想我们现代社会的 collective intelligence,远远大于 hunter-
gatherer 年代。
但另一方面,hunter-gatherer 以后的人类社会的一个巨大的变化,就是社会分工(比
如男耕女织,比如少奇同志曾经对时传祥同志语重心长地说)。
那如果在社会分工后的年代,人类大脑还在继续不断的进化。那这种进化很可能同时也
意味着分化。
也就是说,很有可能的故事是,人类的大脑,在 post hunter-gatherer age,分化为
spatial thinker vs verbal thinker 两种。
其中 spatial thinker 保持 hunter-gatherer 的大脑的基本体系结构,升级成增强版。
而 verbal thinker 转成以 phonological loop 为核心的体系结构,比如 Sex and
the city 里的女主角们。
这样是不是各种理论就世界大同/皆大欢喜了?

【在 t******l 的大作中提到】
: 从这个角度看,要在三里屯打野牛的变化环境中生存,大脑高层 working memory 首先
: 功能并不是死机硬背电话号码死循环,而是 chunking。。。从一大堆风吹草动里,
: chunking 出那只大胸野牛,然后 chunking 出短句通知手持炮筒的同伴,以及大致方
: 向特征。。。然后手持炮筒的同伴利用语言交流的 clue / cue,在一大堆动物里
: chunking 出那只大胸野牛,然后差不多就手一摁咔嚓一声,好像有点歪。。。立马打
: 开 LCD chunking 一下看看能不能回去剪辑一下就好。。。看看好像行。。。于是不在
: 那只大胸野牛上继续纠缠,把注意力转向第二只可能的小峰娇牛。。。
:
: 15

t******l
发帖数: 10908
124
我觉得当中插一句的是,spatial thinker vs verbal thinker 在 take hints 的风格
和喜好,我想可能也有不同。
verbal thinker 可能更倾向于 “冰箱贴” (linguistic instruction) 型的 hint。
而 spatial thinker 可能更倾向于 spatial-temporal cue / clue 型的 hint,
encoded as not only verbal language, but also non-verbal language.


版。

【在 t******l 的大作中提到】
: 但从进化优势的角度,还是存在第三种可能性,也就是人类大脑在 hunter-gatherer
: 以后继续进化。想想我们现代社会的 collective intelligence,远远大于 hunter-
: gatherer 年代。
: 但另一方面,hunter-gatherer 以后的人类社会的一个巨大的变化,就是社会分工(比
: 如男耕女织,比如少奇同志曾经对时传祥同志语重心长地说)。
: 那如果在社会分工后的年代,人类大脑还在继续不断的进化。那这种进化很可能同时也
: 意味着分化。
: 也就是说,很有可能的故事是,人类的大脑,在 post hunter-gatherer age,分化为
: spatial thinker vs verbal thinker 两种。
: 其中 spatial thinker 保持 hunter-gatherer 的大脑的基本体系结构,升级成增强版。

t******l
发帖数: 10908
125
如果从这个角度看,对 spatial thinker 而言,Baddeley's model of working
memory 仍旧 work。只是要把核心从 phonological loop 改成 visuospatial
sketchpad。同时把 visuospatial sketchpad 强化成 visual spatial-temporal
sketchpad。
http://en.wikipedia.org/wiki/Baddeley%27s_model_of_working_memory#Logie.27s_elaboration_of_the_visuospatial_sketchpad
[quote]
Logie has proposed that the visuo-spatial sketchpad can be further
subdivided into two components:
1. The visual cache, which stores information about form and color.
2. The inner scribe, which deals with spatial and movement information. It
also rehearses information in the visual cache and transfers information to
the central executive.[17]
[/quote]


版。

【在 t******l 的大作中提到】
: 但从进化优势的角度,还是存在第三种可能性,也就是人类大脑在 hunter-gatherer
: 以后继续进化。想想我们现代社会的 collective intelligence,远远大于 hunter-
: gatherer 年代。
: 但另一方面,hunter-gatherer 以后的人类社会的一个巨大的变化,就是社会分工(比
: 如男耕女织,比如少奇同志曾经对时传祥同志语重心长地说)。
: 那如果在社会分工后的年代,人类大脑还在继续不断的进化。那这种进化很可能同时也
: 意味着分化。
: 也就是说,很有可能的故事是,人类的大脑,在 post hunter-gatherer age,分化为
: spatial thinker vs verbal thinker 两种。
: 其中 spatial thinker 保持 hunter-gatherer 的大脑的基本体系结构,升级成增强版。

t******l
发帖数: 10908
126
这里要做的改动,是 visual cache 里也要加入 temporal information。否则 visual
cache 里没有 temporal info,inner scribe 无中生有出来一个么?因为:
首先,人脑没有大猩猩的照相式记忆,visual cache 里面不可能把同一个形状在时间
轴上的一秒钟存 25 张照片。
另外就算能存 25 张照片,这也违反 Piaget 老先生的 object permanence on moving
objects (cue)。
不过话说回来,大部分军医们 understand temporal information in a veli veli
poor fashion。一般看到 spatial 就直接理解成 spatial-temporal 就行了。
我觉得可能的原因,是大部分军医们是 verbal thinker,而我猜 verbal thinker 的
temporal information 可能大部分是 encoding / chunking 在 phonological loop
那边,而不是 visuospatial sketchpad 那边。

【在 t******l 的大作中提到】
: 如果从这个角度看,对 spatial thinker 而言,Baddeley's model of working
: memory 仍旧 work。只是要把核心从 phonological loop 改成 visuospatial
: sketchpad。同时把 visuospatial sketchpad 强化成 visual spatial-temporal
: sketchpad。
: http://en.wikipedia.org/wiki/Baddeley%27s_model_of_working_memory#Logie.27s_elaboration_of_the_visuospatial_sketchpad
: [quote]
: Logie has proposed that the visuo-spatial sketchpad can be further
: subdivided into two components:
: 1. The visual cache, which stores information about form and color.
: 2. The inner scribe, which deals with spatial and movement information. It

t******l
发帖数: 10908
127
这个军医们对 temporal 的视而不见,其实也影响 linguistic processing。
就拿前面的蛋糕含糖量的例子。军医要求画个马工 graph,看起来好像是 spatial
thinker,但是 chunking 成三个变量(包括 more vs less),这还是暴露了 verbal
thinker 的本质。
而 spatial thinker 最终 chunking 成两个变量,location vs order。
这里的 location 是 spatial 变量,是不是军医都一样。
这里的 order,军医 chunking 成两个变量 (cake type vs sugar) 的 "more vs less
",但非军医最终 chunking 成一个变量叫 “order”,而 “order” 可以是一个单一
chunk 的 temporal 变量。
至于为啥 “order” 可以是一个单一 chunk 的 temporal 变量,这个小学问题下一贴
说。我这里先重新贴出前面的 chunking。
========= Before chunking ==================================
"If the cake is from France, then it has more sugar if it is made with
chocolate than if it is made with cream, but if the cake is from Italy, then
it has more sugar if it is made with cream than if it is made of chocolate"
======== Lower-order chunking ==============================
In France, chocolate cake has more sugar than cream cake.
In Italy, chocolate cake has less sugar than cream cake.
======== Higher-order chunking, can be used for prediction,
focus shifting, long-term memory conversion =========
(higher order pragmatics "chunking"):
Different location has different order. (Order is function/LUT of location).
(lower order pragmatics "chunking"):
Location is France vs Italy.
Order is the type of cake (chocolate / cream) ordered by sugar percentage.

visual
moving


【在 t******l 的大作中提到】
: 这里要做的改动,是 visual cache 里也要加入 temporal information。否则 visual
: cache 里没有 temporal info,inner scribe 无中生有出来一个么?因为:
: 首先,人脑没有大猩猩的照相式记忆,visual cache 里面不可能把同一个形状在时间
: 轴上的一秒钟存 25 张照片。
: 另外就算能存 25 张照片,这也违反 Piaget 老先生的 object permanence on moving
: objects (cue)。
: 不过话说回来,大部分军医们 understand temporal information in a veli veli
: poor fashion。一般看到 spatial 就直接理解成 spatial-temporal 就行了。
: 我觉得可能的原因,是大部分军医们是 verbal thinker,而我猜 verbal thinker 的
: temporal information 可能大部分是 encoding / chunking 在 phonological loop

t******l
发帖数: 10908
128
对于为啥 “order” 可以是一个单一 chunk 的 temporal 变量,如果既不是
Spatio-temporal synesthesia (involuntarily mapping between spatial and
temporal),也不是 verbal thinker 型的军医们 (unable to mapping between
spatial and temporal whatsoever!!),这两种极端的情况的话,那就必然知道
spatial vs temporal 的最大区别是:“The arrow of time”。
http://en.wikipedia.org/wiki/Synesthesia#Spatio-temporal_synesthesia

verbal
less

【在 t******l 的大作中提到】
: 这个军医们对 temporal 的视而不见,其实也影响 linguistic processing。
: 就拿前面的蛋糕含糖量的例子。军医要求画个马工 graph,看起来好像是 spatial
: thinker,但是 chunking 成三个变量(包括 more vs less),这还是暴露了 verbal
: thinker 的本质。
: 而 spatial thinker 最终 chunking 成两个变量,location vs order。
: 这里的 location 是 spatial 变量,是不是军医都一样。
: 这里的 order,军医 chunking 成两个变量 (cake type vs sugar) 的 "more vs less
: ",但非军医最终 chunking 成一个变量叫 “order”,而 “order” 可以是一个单一
: chunk 的 temporal 变量。
: 至于为啥 “order” 可以是一个单一 chunk 的 temporal 变量,这个小学问题下一贴

t******l
发帖数: 10908
129
当然,“The arrow of time” 这玩意儿又回到了我们的 cosmology 的难题了 hia
hia hia。
不过这里不是在说 cosmology,这里是说 Piaget 的 object-permanence。有争议的说
,“the arrow of time” defines “common agreed direction”(common
knowledge by Piaget's object-permanence, for spatial thinker。而不是 “你知
道我知道天外飞仙知道、歪果仁一到就统统报销” 的那型)。
而 counting,需要一个 “common agreed direction”(外加可以 tick tick tick)。
当然上面都是物理学牛角尖。简单的说,counting 是基于时间的单向流动(The arrow
of time 实际上是指 arrow and flow,漏掉 flow 的属于 pragmatics 不及格)。
而 order 是基于 counting(先不说无理数/实数连续性公理这种钻牛角尖的玩意儿)
,所以 order 本身,就是可以 chunk 成单一变量的 temporal 变量。
t******l
发帖数: 10908
130
而这种 spatial-temporal mapping 是高中解析几何里,最最基本的直线的 point-
slope 概念的基础。
slope 是啥?是 rise-over-run。rise 是 temporal operation 不是?run 也是
temporal operation 不是?(当然,军医可能看到 operation 就头大。那我就用一年
级的小娃的语言,这两个词都是 action word 不是?)
可是且慢,高中解析几何里没有 time axis 啊,又不是运动学。但这词可不是我发明
的,网上到处都是,也就是 neuro-typical 的都是这么理解高中解析几何的。
那为啥 neuro-typical 的 Homo Sapiens,为啥要用运动学来理解静态的解析几何。。
。真相就是 neuro-typical 的 Homo Sapiens 本质上都是禽兽。。。敲错了,本质上
都是动物。。。又敲错了,本质上都是符合 mirror-neuron 之 Piaget 的 object-
permanence on motion objects 的。

)。
arrow

【在 t******l 的大作中提到】
: 当然,“The arrow of time” 这玩意儿又回到了我们的 cosmology 的难题了 hia
: hia hia。
: 不过这里不是在说 cosmology,这里是说 Piaget 的 object-permanence。有争议的说
: ,“the arrow of time” defines “common agreed direction”(common
: knowledge by Piaget's object-permanence, for spatial thinker。而不是 “你知
: 道我知道天外飞仙知道、歪果仁一到就统统报销” 的那型)。
: 而 counting,需要一个 “common agreed direction”(外加可以 tick tick tick)。
: 当然上面都是物理学牛角尖。简单的说,counting 是基于时间的单向流动(The arrow
: of time 实际上是指 arrow and flow,漏掉 flow 的属于 pragmatics 不及格)。
: 而 order 是基于 counting(先不说无理数/实数连续性公理这种钻牛角尖的玩意儿)

相关主题
分数应用题忍不住上来问问,我娃的数学是啥水平(6岁)?
我招,我是猪陶天才论天才 (转载)
老师说我女儿是gifted kid (转载)NIH 证实天才和疯子都源于同一型基因:DARPP-32
进入Parenting版参与讨论
t******l
发帖数: 10908
131
但另一个猜想是,what if spatial thinkers could also process lots (not all,
for sure) linguistic information in "visual spatial-temporal sketchpad"。就
比如前面那个蛋糕含糖量的 chunking?

【在 t******l 的大作中提到】
: 而这种 spatial-temporal mapping 是高中解析几何里,最最基本的直线的 point-
: slope 概念的基础。
: slope 是啥?是 rise-over-run。rise 是 temporal operation 不是?run 也是
: temporal operation 不是?(当然,军医可能看到 operation 就头大。那我就用一年
: 级的小娃的语言,这两个词都是 action word 不是?)
: 可是且慢,高中解析几何里没有 time axis 啊,又不是运动学。但这词可不是我发明
: 的,网上到处都是,也就是 neuro-typical 的都是这么理解高中解析几何的。
: 那为啥 neuro-typical 的 Homo Sapiens,为啥要用运动学来理解静态的解析几何。。
: 。真相就是 neuro-typical 的 Homo Sapiens 本质上都是禽兽。。。敲错了,本质上
: 都是动物。。。又敲错了,本质上都是符合 mirror-neuron 之 Piaget 的 object-

t******l
发帖数: 10908
132
一个更大胆的猜想是,what if 我们所有人都没有看到最关键的地方。What if 大脑发
展/大脑类型最大的差别,既然不在于语言,也不在于空间形状,而是在于大脑如何处
理 “时间信息”(perceptual of temporal, probably largely "implicitly"
process temporal info)。
从这个角度看,Piaget 的 object-permanence,其实并不是在空间上 permanence,其
实是在时间轴上 permanence, implicitly。
而 Piaget 的 conservation,同理,只是在更高的层次。
而娃娃开始意识到 life vs death,just means permanence only within a
timeframe, implicitly.
而从进化优势上说,当 hunter-gatherer 看到狮子从左边进去,predict 会从右边出
来,这极大的增加生存概率。better understand temporal, implicitly。
而当 hunter-gatherer 看到蜜桃青涩的时候,估计再过三个月蜜桃就成熟了,所以三
个月以后可以对这李丽珍撸上一发了。这也极大的增加生存概率。并且让 color
vision 更 useful。
没有任何其他的动物能在时间轴上面比人类做更远更好的预测(个别单个事例除外),
这可能也是另一个证据 better (implicitly) perceive temporal info maybe is the
fundamental information processing of different brains.

【在 t******l 的大作中提到】
: 但另一个猜想是,what if spatial thinkers could also process lots (not all,
: for sure) linguistic information in "visual spatial-temporal sketchpad"。就
: 比如前面那个蛋糕含糖量的 chunking?

t******l
发帖数: 10908
133
我刚才做了 online 的 single 2-back task 的测试,测试的结果我 completely
failed on that test,或者说,零蛋。这都不用说客观测试结果了,实际上在 4 到 5
个符号以后,my brain completely lost, cannot even finish the test。
如果我 2-back task 做的很差,那可以争议说我比较弱智。但我这是 unable to
complete the task at all,按照军医学我就不是弱智的问题,而是说我是残疾人,需
要专人照顾起居穿衣的问题。
这唯一可能的解释就是,就是存在两种类型的大脑 (1) "sex-and-the-city's verbal
thinker" (2) "hunter-gatherer's spatial-temporal thinker"。而 phonological
loop 确实是 verbal thinker's working memory 的核心部分,但 spatial-temporal
thinker don't even need a phonological loop in their working memory.


版。

【在 t******l 的大作中提到】
: 但从进化优势的角度,还是存在第三种可能性,也就是人类大脑在 hunter-gatherer
: 以后继续进化。想想我们现代社会的 collective intelligence,远远大于 hunter-
: gatherer 年代。
: 但另一方面,hunter-gatherer 以后的人类社会的一个巨大的变化,就是社会分工(比
: 如男耕女织,比如少奇同志曾经对时传祥同志语重心长地说)。
: 那如果在社会分工后的年代,人类大脑还在继续不断的进化。那这种进化很可能同时也
: 意味着分化。
: 也就是说,很有可能的故事是,人类的大脑,在 post hunter-gatherer age,分化为
: spatial thinker vs verbal thinker 两种。
: 其中 spatial thinker 保持 hunter-gatherer 的大脑的基本体系结构,升级成增强版。

t******l
发帖数: 10908
134
我觉得有一个可能的解释,是 hunter-gatherer's spatial-temporal thinker 在数量
上比较少,(hunter-gatherer 年代的人类人口其实也确实不多),普查型 sample 正
常人的时候 sample 不到。这样去医院 sample 看医生的 twice exceptional 的时候
,这帽子可以随便扣就是了(反正是智商测试本来就是那个精神病院测试给外推出来的
)。

5
verbal
temporal

【在 t******l 的大作中提到】
: 我刚才做了 online 的 single 2-back task 的测试,测试的结果我 completely
: failed on that test,或者说,零蛋。这都不用说客观测试结果了,实际上在 4 到 5
: 个符号以后,my brain completely lost, cannot even finish the test。
: 如果我 2-back task 做的很差,那可以争议说我比较弱智。但我这是 unable to
: complete the task at all,按照军医学我就不是弱智的问题,而是说我是残疾人,需
: 要专人照顾起居穿衣的问题。
: 这唯一可能的解释就是,就是存在两种类型的大脑 (1) "sex-and-the-city's verbal
: thinker" (2) "hunter-gatherer's spatial-temporal thinker"。而 phonological
: loop 确实是 verbal thinker's working memory 的核心部分,但 spatial-temporal
: thinker don't even need a phonological loop in their working memory.

t******l
发帖数: 10908
135
我觉得这个结果是 self-telling 的。其中有一个例子就是 silent-reading。
根据 Baddeley's model 的 phonological-looping driven working memory,silent-
reading 跟 read aloud 的区别只是 motor function is suppressed。
但是有争议的是,这个 model 无法解释 super fast silent reading。马工系说,就
算大脑的 comprehension 速度可以依赖 intuitive 而飞速提高,这 phonological-
looping 的 strictly sequential fashion,就是不折不扣的冯诺伊曼瓶颈(von
Neumann bottleneck)。

5
verbal
temporal

【在 t******l 的大作中提到】
: 我刚才做了 online 的 single 2-back task 的测试,测试的结果我 completely
: failed on that test,或者说,零蛋。这都不用说客观测试结果了,实际上在 4 到 5
: 个符号以后,my brain completely lost, cannot even finish the test。
: 如果我 2-back task 做的很差,那可以争议说我比较弱智。但我这是 unable to
: complete the task at all,按照军医学我就不是弱智的问题,而是说我是残疾人,需
: 要专人照顾起居穿衣的问题。
: 这唯一可能的解释就是,就是存在两种类型的大脑 (1) "sex-and-the-city's verbal
: thinker" (2) "hunter-gatherer's spatial-temporal thinker"。而 phonological
: loop 确实是 verbal thinker's working memory 的核心部分,但 spatial-temporal
: thinker don't even need a phonological loop in their working memory.

t******l
发帖数: 10908
136
INTEL 说了,在马工史上,在商业上解决冯诺伊曼瓶颈(von Neumann bottleneck)最
最成功的方案,就是 INTEL 的超级标量流水线结构。
在超级标量流水线结构里,复杂机器码指令一进来就被打成更小的微指令,数据事先
prefetch 到 L2 Cache,然后被硬件流水线预测/分配器扔到二十条不同的超级标量流
水线上并行计算(外加数据相关性处理)。。。当然,N-back 可能麻烦点。指令都已
经被打散成微指令不算,都已经扔到二十条超级标量流水线的某一条上,不再保持输入
的 sequential order 了。数据的问题是 L2 Cache 基本上不会马上就刷回 RAM 去的
,且等。。。但客户就是点名要超级标量流水线结构,吞吐量大的同时,还能兼容复杂
指令集(CISC)、甚至伪向量指令混腔集。

silent-

【在 t******l 的大作中提到】
: 我觉得这个结果是 self-telling 的。其中有一个例子就是 silent-reading。
: 根据 Baddeley's model 的 phonological-looping driven working memory,silent-
: reading 跟 read aloud 的区别只是 motor function is suppressed。
: 但是有争议的是,这个 model 无法解释 super fast silent reading。马工系说,就
: 算大脑的 comprehension 速度可以依赖 intuitive 而飞速提高,这 phonological-
: looping 的 strictly sequential fashion,就是不折不扣的冯诺伊曼瓶颈(von
: Neumann bottleneck)。
:
: 5
: verbal

t******l
发帖数: 10908
137
我觉得虽然 silent reading 的时候,谁都不知道大脑在干啥。但是有一个 self-
telling 的,就是看起来 spatial-temporal thinker 在 learn-to-read 向 read-to-
learn 转型的 3rd/4th grade decay point 的时候,读很多图书馆的阅读。
而有争议的说,这阶段 brain 可能在 re-training itself, 把 phonological-loop
based silent reading 转型成 super-scalar-(parallel)pipelines based silent
reading。

【在 t******l 的大作中提到】
: INTEL 说了,在马工史上,在商业上解决冯诺伊曼瓶颈(von Neumann bottleneck)最
: 最成功的方案,就是 INTEL 的超级标量流水线结构。
: 在超级标量流水线结构里,复杂机器码指令一进来就被打成更小的微指令,数据事先
: prefetch 到 L2 Cache,然后被硬件流水线预测/分配器扔到二十条不同的超级标量流
: 水线上并行计算(外加数据相关性处理)。。。当然,N-back 可能麻烦点。指令都已
: 经被打散成微指令不算,都已经扔到二十条超级标量流水线的某一条上,不再保持输入
: 的 sequential order 了。数据的问题是 L2 Cache 基本上不会马上就刷回 RAM 去的
: ,且等。。。但客户就是点名要超级标量流水线结构,吞吐量大的同时,还能兼容复杂
: 指令集(CISC)、甚至伪向量指令混腔集。
:

t******l
发帖数: 10908
138
另一个可以直接观察的,neuro-typical 的小娃拼 jigsaw puzzle 的过程(指 48
piece 及以上,并且是第一或第二次拼没见过的)。
其实 jigsaw puzzle 如果用 phonological loop 型 sequential 穷举法拼的话,这都
不需要 working memory,所有 piece 反正一览无余。
但 neuro-typical 的小娃从来就不会这么干,拼的顺序都是 “乱中取胜”,大块的同
时还有好几个。而你要是观察小娃拼的时候,你会 feel 这玩意儿确实需要 working
memory。所以问题是 working memory 这时在干嘛?
我觉得有争议的说,这时候 working memory 的主要任务是在 process temporal
information,在这里就是 order information。并且还有几条不同的 timeline(几个
大块),甚至可能还有 alternative timeline(潜在可能的大块)。而 working
memory 在 process 了 temporal information 以后,implicitly attach 在某些
piece 上(implicitly 指不是真的脑子里给标记上 1 2 3 4 这么 explicit),然后
把中间结果送到 visual-spatial sketchpad 这个 L-2 cache 里缓存(visual-
spatial sketchpad 可以大大超过 magic 7 的限制,并且可以存相对而言相当长的时
间,可能是 local neuron looping path。但是不想 the magic 7 那样容易 explicit
的取出来 formulated to a language 还一点不出错)。

to-

【在 t******l 的大作中提到】
: 我觉得虽然 silent reading 的时候,谁都不知道大脑在干啥。但是有一个 self-
: telling 的,就是看起来 spatial-temporal thinker 在 learn-to-read 向 read-to-
: learn 转型的 3rd/4th grade decay point 的时候,读很多图书馆的阅读。
: 而有争议的说,这阶段 brain 可能在 re-training itself, 把 phonological-loop
: based silent reading 转型成 super-scalar-(parallel)pipelines based silent
: reading。

t******l
发帖数: 10908
139
但是正如 INTEL 的超级标量流水线里,数据相关性不仅是个问题,而是个大问题;
visual-spatial sketchpad,相对 phonological loop 而言,temporal information
不仅是个问题,并且还是个大问题。
所以有争议的大胆假设,hunter-gatherer 型的 spatial thinker 的 working memory
,在基本模块结构上可能跟 baddeley's model of working memory 差不多,但在功能
和数据流上可能大相径庭。比如一个可能的模型是:
phonological loop 的职责主要是 holding input information (verbal and
temporal) for very short time, waiting for central executive。phonological
loop 并不处理信息,central executive 一旦拿走了信息,phonological loop 立即
扔掉信息 free memory。也就是说,technically not really "looping",基本以单向
模式为主。
central executive 的一个重要职能,是从 phonological loop 里不断取走信息,同
时 process temporal information,包括 phonological loop 里的前后顺序(order
就是 temporal information 的一种)。同时 encoding / chunking temporal
information,然后把信息送到 visual-spatial sketchpad 里。
而 visual-spatial sketch pad 里没有 central loop,因为 temporal information
已经被 encoding,不再需要 central loop(central loop 比如 phonological loop
,一个大功能就是 keep the order,也就是 keep temporal information)。visual-
spatial sketch pad 只需要 local neuron-path-loop 不断 refresh local D-RAM 就
可以了,所以存的比较久。同时 implicit/intuitive processing 也能够并行搞超级
标量流水线。缺点是 explicit 的取出来不容易,因为要把 encoded temporal
information 重新排序成 phonological looping order,军医学俗称:formulate a
language。

【在 t******l 的大作中提到】
: 另一个可以直接观察的,neuro-typical 的小娃拼 jigsaw puzzle 的过程(指 48
: piece 及以上,并且是第一或第二次拼没见过的)。
: 其实 jigsaw puzzle 如果用 phonological loop 型 sequential 穷举法拼的话,这都
: 不需要 working memory,所有 piece 反正一览无余。
: 但 neuro-typical 的小娃从来就不会这么干,拼的顺序都是 “乱中取胜”,大块的同
: 时还有好几个。而你要是观察小娃拼的时候,你会 feel 这玩意儿确实需要 working
: memory。所以问题是 working memory 这时在干嘛?
: 我觉得有争议的说,这时候 working memory 的主要任务是在 process temporal
: information,在这里就是 order information。并且还有几条不同的 timeline(几个
: 大块),甚至可能还有 alternative timeline(潜在可能的大块)。而 working

t******l
发帖数: 10908
140
而这个猜想的证实或者证伪,是有实际意义的。因为如果这猜想证实的话,那就意味着:
For "sex-and-the-city's verbal thinker"'s working memory, the capability of
"looping/holding" information in their (presumed) phonological loop is "1st-
order critical". An inferiority on the "looping/holding" could cause
learning disabilities, and maybe need specialized training to correct/
improve.
For "hunter-gatherer's spatial-temporal thinker"'s working memory, instead,
the capability of "chunking / encoding temporal information" in their (
hypothesized) central executive is "1st-order critical". An inferiority on "
chunking / encoding temporal information" could cause learning disabilities,
and maybe need specialized training to correct/improve.
当然如果被证伪的话,上面的 “分类后因菜施浇” 就不成立。

information
memory

【在 t******l 的大作中提到】
: 但是正如 INTEL 的超级标量流水线里,数据相关性不仅是个问题,而是个大问题;
: visual-spatial sketchpad,相对 phonological loop 而言,temporal information
: 不仅是个问题,并且还是个大问题。
: 所以有争议的大胆假设,hunter-gatherer 型的 spatial thinker 的 working memory
: ,在基本模块结构上可能跟 baddeley's model of working memory 差不多,但在功能
: 和数据流上可能大相径庭。比如一个可能的模型是:
: phonological loop 的职责主要是 holding input information (verbal and
: temporal) for very short time, waiting for central executive。phonological
: loop 并不处理信息,central executive 一旦拿走了信息,phonological loop 立即
: 扔掉信息 free memory。也就是说,technically not really "looping",基本以单向

相关主题
NIH 证实天才和疯子都源于同一型基因:DARPP-32来看一下各位的观点
放养妈进来看看是不是任我儿子继续迷象棋
最新研究证明心算AIME大幅度促进心理和情感健康9岁, 如何报名考amc10
进入Parenting版参与讨论
t*******s
发帖数: 48
141
站内短信你了一个问题,请查收。谢谢回答。

【在 s***n 的大作中提到】
: 其实看你的描述,我觉得你孩子更像第二种情况 - 喜欢用脑思考的孩子。他碰到难题
: 发问,可能是因为题目对他来说太难了,也可能是因为没学过相应的基础知识或学得不
: 扎实。指导这样的孩子,关键是你的hint不能给得太多,要点到为止。这些孩子真正享
: 受的是战胜挑战后的成就感。所以他们喜欢智力脑力上的挑战,也倾向于独立地去战胜
: 这些挑战。他们碰到难题的时候,你给的hint越多,他的fun和成就感就越少。你孩子
: 对hint的反应是很正常的第二类HGT小娃的反应。你这篇文章提到一些情况其实更多地
: 是指导老师的问题。他们没有针对GT小娃的特点来教学。
: 自然很多事情都有两面性。完全拒绝hint,拒绝学习别人,也会阻碍孩子的发展,特别
: 是那些源于自大的粗暴拒绝。我觉得解决办法主要是带孩子去见世面。参加各种竞赛见
: 见难题,接触下那些给他hint都不容易做出来的题,意识到自己的不足;参加各种数学

t******l
发帖数: 10908
142
我觉得这个猜想在进化学上可能也说得过去的原因,是 hunter-gatherer 的模式是从
1.8 million 年前,到 10,000 年前。这一来说明 hunter-gatherer 的进化选择应该
已经是一个稳定选择。二来这 “一万年” 大脑很难发生大的 “基本结构性改变”,
但这 “一万年” 可能足够大脑发生 “功能性改变/分化” 了。

information
memory

【在 t******l 的大作中提到】
: 但是正如 INTEL 的超级标量流水线里,数据相关性不仅是个问题,而是个大问题;
: visual-spatial sketchpad,相对 phonological loop 而言,temporal information
: 不仅是个问题,并且还是个大问题。
: 所以有争议的大胆假设,hunter-gatherer 型的 spatial thinker 的 working memory
: ,在基本模块结构上可能跟 baddeley's model of working memory 差不多,但在功能
: 和数据流上可能大相径庭。比如一个可能的模型是:
: phonological loop 的职责主要是 holding input information (verbal and
: temporal) for very short time, waiting for central executive。phonological
: loop 并不处理信息,central executive 一旦拿走了信息,phonological loop 立即
: 扔掉信息 free memory。也就是说,technically not really "looping",基本以单向

t******l
发帖数: 10908
143
我测了一下下面这个 link 里的 Backward Corsi Block Test。
My "spatial loop": (Backward Corsi Block Test) is better than 2-back,but
the longest I do is only 4 (and I have to click very fast to avoid fading
the spatial phonological loop). I cannot even do 5. (Average people do 6 - 8
).
http://www.memorylosstest.com/free-working-memory-tests-online/



【在 t******l 的大作中提到】
: 我觉得这个猜想在进化学上可能也说得过去的原因,是 hunter-gatherer 的模式是从
: 1.8 million 年前,到 10,000 年前。这一来说明 hunter-gatherer 的进化选择应该
: 已经是一个稳定选择。二来这 “一万年” 大脑很难发生大的 “基本结构性改变”,
: 但这 “一万年” 可能足够大脑发生 “功能性改变/分化” 了。
:
: information
: memory

t******l
发帖数: 10908
144
但我可以成功的把下面的 forward digital span 做到 8,但用一个 cheat。
我第一次做 8 的时候,我 fail 了。但我 instantly aware 一个 cheat,也就是每次
新数字出现的时候,我 instantly refresh 所有之前的数字。也就是我心里默念:
2
26
263
2632
26329
2632924
26329243
then I succeed by using "rolling refreshing D-RAM" and "single word-like
chunking". 吼吼吼。
http://www.memorylosstest.com/free-working-memory-tests-online/

the

【在 t******l 的大作中提到】
: 我测了一下下面这个 link 里的 Backward Corsi Block Test。
: My "spatial loop": (Backward Corsi Block Test) is better than 2-back,but
: the longest I do is only 4 (and I have to click very fast to avoid fading
: the spatial phonological loop). I cannot even do 5. (Average people do 6 - 8
: ).
: http://www.memorylosstest.com/free-working-memory-tests-online/
:
: 从

n****f
发帖数: 3580
145
你好闲。

【在 t******l 的大作中提到】
: 但我可以成功的把下面的 forward digital span 做到 8,但用一个 cheat。
: 我第一次做 8 的时候,我 fail 了。但我 instantly aware 一个 cheat,也就是每次
: 新数字出现的时候,我 instantly refresh 所有之前的数字。也就是我心里默念:
: 2
: 26
: 263
: 2632
: 26329
: 2632924
: 26329243

t******l
发帖数: 10908
146
我一边修 bug 一边做这个。。。其实我只是想知道我自己是不是阿斯伯格。。。

【在 n****f 的大作中提到】
: 你好闲。
n****f
发帖数: 3580
147
信可能是发给另一位的。

【在 t******l 的大作中提到】
: 我一边修 bug 一边做这个。。。其实我只是想知道我自己是不是阿斯伯格。。。
t******l
发帖数: 10908
148
I see,我没看回帖下面的附贴。

【在 n****f 的大作中提到】
: 信可能是发给另一位的。
t******l
发帖数: 10908
149
OK 用上面相同的 "rolling D-RAM refresh cheat",我能做到 5 没有问题了。
就是每次新黑点出来的时候,按 forward order refresh 之前所有的黑点,最后一次
reverse order click。就过了 5。我再试试。

8

【在 t******l 的大作中提到】
: 我测了一下下面这个 link 里的 Backward Corsi Block Test。
: My "spatial loop": (Backward Corsi Block Test) is better than 2-back,but
: the longest I do is only 4 (and I have to click very fast to avoid fading
: the spatial phonological loop). I cannot even do 5. (Average people do 6 - 8
: ).
: http://www.memorylosstest.com/free-working-memory-tests-online/
:
: 从

t******l
发帖数: 10908
150
OK, 我成功的稳定地能做 6 个,但是用了一个更复杂的 cheat。
首先我用前面做 5 个的 cheat,但是连续三次全失败了。于是我默念了三遍 Piaget
的 object-permanence on motion objects 以后,观察了以后发现用一个新的 cheat
。这个 cheat 比较复杂,详细这么说:
首先我不仅仅记住点,而且记住前后两点之间的连线,想象成一个 motion object w/
path hided。成为一个 open-end polygon w/ construction order. (chunking?)
然后我就在新点出来的时候,不短 rolling refresh 那个 open-end polygon w/
construction order.
在最后一个点出来以后,我最后一遍 forward refresh 那个 “open-end polygon w/
construction order”(如果不确信的话,refresh 两遍好了)。然后 reverse click
on the vertexes。(之前我漏了这先 forward refresh 一次,导致连续 fail 了两
次)。
用了这个 cheat,能稳定做 6 个。终于成为正常人而不是弱智啦,哈哈哈哈哈哈哈。
不过军医学不是说这个测试是本能,不需要也无法 learn 的?



【在 t******l 的大作中提到】
: OK 用上面相同的 "rolling D-RAM refresh cheat",我能做到 5 没有问题了。
: 就是每次新黑点出来的时候,按 forward order refresh 之前所有的黑点,最后一次
: reverse order click。就过了 5。我再试试。
:
: 8

相关主题
求科普: 奥数 vs. Math Olympiad vs. Math Count vs. Math circle vs. 超前学数学大家怎么推孩子的弱项?
有明天考AMC 8的吗?少年智力开发
算不算有数学天分[转载] 为什么说usamo 简单
进入Parenting版参与讨论
t******l
发帖数: 10908
151
看上去这个 directed open-end polygon encoding 能够 work 到 8 个,但是还不稳
定,要多玩可能会稳定。
也就是说,practice enhance spatial-temporal encoding?

cheat
/
/
click

【在 t******l 的大作中提到】
: OK, 我成功的稳定地能做 6 个,但是用了一个更复杂的 cheat。
: 首先我用前面做 5 个的 cheat,但是连续三次全失败了。于是我默念了三遍 Piaget
: 的 object-permanence on motion objects 以后,观察了以后发现用一个新的 cheat
: 。这个 cheat 比较复杂,详细这么说:
: 首先我不仅仅记住点,而且记住前后两点之间的连线,想象成一个 motion object w/
: path hided。成为一个 open-end polygon w/ construction order. (chunking?)
: 然后我就在新点出来的时候,不短 rolling refresh 那个 open-end polygon w/
: construction order.
: 在最后一个点出来以后,我最后一遍 forward refresh 那个 “open-end polygon w/
: construction order”(如果不确信的话,refresh 两遍好了)。然后 reverse click

t******l
发帖数: 10908
152
用类似的办法,我觉得搞定 n-back task 应该也行。也就是直接把图形先 encode 成
音节字母啥的,然后 keep rolling refresh。多练点 encoding 就无往而不利。当然
,我不浪费时间了。
但这是不是可以开 “新东方 for 智商测试” 了?广告词我想好了,就是 “从 85 到
180 不是梦!!”

:看上去这个 directed open-end polygon encoding 能够 work 到 8 个,但是还不稳
:定,要多玩可能会稳定。
t******l
发帖数: 10908
153
我现在又去做一遍这个 test,同时我不再 explicitly 做这个 "rolling refresh
cheat"。我甚至主观上 suppress 这个 "rolling refresh cheat",想知道没有 "
rolling refresh cheat" 的时候,我的 digital span 能做对几个的长度,其实我是
想知道 digital span 能 hold 几秒。(我不期望我能做对 8 个的长度)
But magically, 我开始的 8 个的长度就立即做对了,重复两次都是对的。。。但我有
模糊的感觉,我在做 implicit "rolling refresh cheat",并且似乎是快得多的
rolling refresh 速度,这种 implicit rolling refresh 似乎无法主观简单 shutoff。
我觉得如果这个是真的,就能解释为啥娃有时候不爱 take hint。也就是说,just by
knowing it can be solved by "rolling refresh cheat", the brain could
rehearse it in background when people are doing something else. And after
couple hours, the brain may be able to perform it in subconscious / implicit
level.

【在 t******l 的大作中提到】
: 但我可以成功的把下面的 forward digital span 做到 8,但用一个 cheat。
: 我第一次做 8 的时候,我 fail 了。但我 instantly aware 一个 cheat,也就是每次
: 新数字出现的时候,我 instantly refresh 所有之前的数字。也就是我心里默念:
: 2
: 26
: 263
: 2632
: 26329
: 2632924
: 26329243

t******l
发帖数: 10908
154
但我试了一下重做这个 Corsi Block Test, suppress 前面的那个 rolling refresh D
-RAM cheat 以及 open-end polygon cheat。
And interestingly, this time I can suppress 80% of the cheat (still not 100%
)。I completely failed on length of 8, then I back to length 5, and found I
could do some of length 5 (I cannot do length 5 at all before I am aware
those two cheats), but with the failure rate around 50% or 60% or so.
先不说其他的,我觉得首先军医学认为智商测试是 completely repeatable 的这点上
,我觉得军医们可以怀疑智商测试的人生了。。。我觉得我们的 brain relentlessly
change / training its capability of encoding / chunking,这就有点像 Star
Trek 里的 Borg。The brain adapt, if the test is predictable。

cheat
/
/
click

【在 t******l 的大作中提到】
: OK, 我成功的稳定地能做 6 个,但是用了一个更复杂的 cheat。
: 首先我用前面做 5 个的 cheat,但是连续三次全失败了。于是我默念了三遍 Piaget
: 的 object-permanence on motion objects 以后,观察了以后发现用一个新的 cheat
: 。这个 cheat 比较复杂,详细这么说:
: 首先我不仅仅记住点,而且记住前后两点之间的连线,想象成一个 motion object w/
: path hided。成为一个 open-end polygon w/ construction order. (chunking?)
: 然后我就在新点出来的时候,不短 rolling refresh 那个 open-end polygon w/
: construction order.
: 在最后一个点出来以后,我最后一遍 forward refresh 那个 “open-end polygon w/
: construction order”(如果不确信的话,refresh 两遍好了)。然后 reverse click

t******l
发帖数: 10908
155
但这可能有引发了另一个怀疑。具体这么说:
The shape of open-end polygon 就是一个 spatial encoding of temporal info (
phonological order of the vertex)。所以有争议的说,这个有可能是用 visual-
spatial sketchpad 的 shape memory,通过 encoding temporal info,至少是
partially
assist phonological order。
而这个容易被主观 suppress,是因为这个 polygon encoding 更 complex,not that
easy to learn。
所以,上面的 digital span,也很难知道到底是 implicitly articulate loop 呢?
还是大脑自我训练出了更新版的 encoding temporal,可以利用 visual-spatial
sketchpad 来 assist articulate loop,或者干脆是两者兼而有之。

D
100%
I
relentlessly

【在 t******l 的大作中提到】
: 但我试了一下重做这个 Corsi Block Test, suppress 前面的那个 rolling refresh D
: -RAM cheat 以及 open-end polygon cheat。
: And interestingly, this time I can suppress 80% of the cheat (still not 100%
: )。I completely failed on length of 8, then I back to length 5, and found I
: could do some of length 5 (I cannot do length 5 at all before I am aware
: those two cheats), but with the failure rate around 50% or 60% or so.
: 先不说其他的,我觉得首先军医学认为智商测试是 completely repeatable 的这点上
: ,我觉得军医们可以怀疑智商测试的人生了。。。我觉得我们的 brain relentlessly
: change / training its capability of encoding / chunking,这就有点像 Star
: Trek 里的 Borg。The brain adapt, if the test is predictable。

s***n
发帖数: 1280
156
我其实也想写,但我太懒,更愿意被动回答问题,不愿意主动写。我一年前答应别人写
的东西到现在还没写...
我没训练过自己或孩子的working memory。我只是以前看过ADHD/ADD方面的书,后来和
一些ADHD+HGT娃家长交流过,知道有这方面的训练课程和书籍。
每个孩子读不懂题的原因不太一样。我儿子是因为偏科,生活常识差。他备考AMC 8的
时候,做错题主要是因为误解题目意思或者不知道题目背景常识,不是数学。所以我辅
导他AMC 8的时候,主要是讲解语法和介绍常识。背后的想法是利用他喜欢擅长的科目
补他不喜欢不擅长的。我当时的情况只是备考,所以读不懂题这个问题可以通过短期针
对性练习和教学"解决",真正意义上的解决还是要靠外延孩子的兴趣爱好和知识面,这
个就是长期的活了。

【在 T********s 的大作中提到】
: 你单开一贴给大家讲一讲GT娃早期的表现,鉴定和培养吧。
: 还有怎样训练working memory。
: 还有关于你娃有时读不懂题的,你以前帖子说过的,现在找不着了。能不能再
: 说一说你是怎么帮助他解决这个问题的?
: 你的贴总是信息量很大,很值得追。咱们多push几下,让你多讲一些。
:
: Gifted
: GT

t******l
发帖数: 10908
157
对于(Reverse) Corsi Block Test 我发现了一个更新的 cheat,可以保证稳稳当当做
出 8 个。
http://www.memorylosstest.com/free-working-memory-tests-online/
我的最新 cheat 是,这个格子虽然歪曲过,但实际上大致上是一个 3 row 4 column
的矩阵。
这样就干脆对矩阵上的格子在内心编号, 1 - 10 , a , b。
然后把 8 个黑点切成前后两组,每组四个。对于前面四个黑点,每个出来的时候给编
号,然后心里正向 phonological loop,比如 45A8。但是第五个开始就不编号,跟踪
位置,同时心里不断 phonological loop 来 refresh 前面的四个编码。然后 8 个全
结束后,立即先把后面四个标记上,动作要快省得 working memory fade。之后就可以
慢悠悠的把 45A8 给 reverse order 成 8A54,然后一个一个点击,这样能保证以极大
概率做对 8 个。
军医啊,虽然我们没有 FFT 硬件处理器,但我们一条超级标量流水线,外加一套 SSE
指令集,双管齐下,还是干掉了不是?

D
100%
I
relentlessly

【在 t******l 的大作中提到】
: 但我试了一下重做这个 Corsi Block Test, suppress 前面的那个 rolling refresh D
: -RAM cheat 以及 open-end polygon cheat。
: And interestingly, this time I can suppress 80% of the cheat (still not 100%
: )。I completely failed on length of 8, then I back to length 5, and found I
: could do some of length 5 (I cannot do length 5 at all before I am aware
: those two cheats), but with the failure rate around 50% or 60% or so.
: 先不说其他的,我觉得首先军医学认为智商测试是 completely repeatable 的这点上
: ,我觉得军医们可以怀疑智商测试的人生了。。。我觉得我们的 brain relentlessly
: change / training its capability of encoding / chunking,这就有点像 Star
: Trek 里的 Borg。The brain adapt, if the test is predictable。

t*******s
发帖数: 48
158
今天花了四个小时读了一些关于提高working memory 的文章,真是说法不一。我很多
时候都完全泄气了,觉得这根本没法改变。以至于现在只要和小朋友讲话,我就立马觉
得他的思维根本不在谈话内容上。本来他这方面就问题严重,他还边上网课边在
YouTube 上整他的各种歌曲还外加在网课上聊天,我每次看到真是是恨的心脏痛,还不
能吭声

【在 s***n 的大作中提到】
: 我其实也想写,但我太懒,更愿意被动回答问题,不愿意主动写。我一年前答应别人写
: 的东西到现在还没写...
: 我没训练过自己或孩子的working memory。我只是以前看过ADHD/ADD方面的书,后来和
: 一些ADHD+HGT娃家长交流过,知道有这方面的训练课程和书籍。
: 每个孩子读不懂题的原因不太一样。我儿子是因为偏科,生活常识差。他备考AMC 8的
: 时候,做错题主要是因为误解题目意思或者不知道题目背景常识,不是数学。所以我辅
: 导他AMC 8的时候,主要是讲解语法和介绍常识。背后的想法是利用他喜欢擅长的科目
: 补他不喜欢不擅长的。我当时的情况只是备考,所以读不懂题这个问题可以通过短期针
: 对性练习和教学"解决",真正意义上的解决还是要靠外延孩子的兴趣爱好和知识面,这
: 个就是长期的活了。

t******l
发帖数: 10908
159
上 YouTube 跟 working memory 没有关系吧。
我家大娃最近每天花无数时间看着网上的漫画画漫画,我也就随着天性了。
我觉得关键是会不会造成问题。如果本来也不会造成啥问题,强扭反而造成问题,那还
不如随天性,大方向调整一下就差不多了。

:今天花了四个小时读了一些关于提高working memory 的文章,真是说法不一。我很多
:时候都完全泄气了,觉得这根本没法改变。以至于现在只要和小朋友讲话,我就立马
觉得他的思维根本不在谈话内容上。本来他这方面就问题严重,他还边上网课边在
t*******s
发帖数: 48
160
问题是弄YouTube 的同时还在上课,我时常怀疑他思维根本就不在上课。所以提醒过多
次这两个事情不能统一时间做,要么专心在YouTube 上,要么认真上课。这也是
working memory 低的孩子共有的问题,就是executive skill 差,不能高效安排执行
多个事情。

很多

【在 t******l 的大作中提到】
: 上 YouTube 跟 working memory 没有关系吧。
: 我家大娃最近每天花无数时间看着网上的漫画画漫画,我也就随着天性了。
: 我觉得关键是会不会造成问题。如果本来也不会造成啥问题,强扭反而造成问题,那还
: 不如随天性,大方向调整一下就差不多了。
:
: :今天花了四个小时读了一些关于提高working memory 的文章,真是说法不一。我很多
: :时候都完全泄气了,觉得这根本没法改变。以至于现在只要和小朋友讲话,我就立马
: 觉得他的思维根本不在谈话内容上。本来他这方面就问题严重,他还边上网课边在

相关主题
二年级女儿的report我招,我是猪
觉得美帝数学进度慢的进来看一下老师说我女儿是gifted kid (转载)
分数应用题忍不住上来问问,我娃的数学是啥水平(6岁)?
进入Parenting版参与讨论
t******l
发帖数: 10908
161
你这个上网课时同时干其他的事比较特殊。但一般而言,同时做两件事,跟 working
memory 的 executive skill 没有直接关系。
我家两娃在家里从小就是“一边一边”,同时干两件事(学校另说,有 joint
attention。实际上学校里我的不知道,只知道娃不 cause problem 就是了)。但这并
不造成问题,一个例子是拼 jigsaw puzzle,“一边一边” 也拼出来了,我就一般不
干预。
这种情况,我觉得不是 inferior working memory 造成的,而是 different type of
working memory。比如一种可能就是 working memory process faster but also
fading faster,造成 internal speed 和 external speed 的 out-of-sync。

【在 t*******s 的大作中提到】
: 问题是弄YouTube 的同时还在上课,我时常怀疑他思维根本就不在上课。所以提醒过多
: 次这两个事情不能统一时间做,要么专心在YouTube 上,要么认真上课。这也是
: working memory 低的孩子共有的问题,就是executive skill 差,不能高效安排执行
: 多个事情。
:
: 很多

t******l
发帖数: 10908
162
我另一点不同意军医学的,就是对 giftedness / learning disability 的定义。军医
学认为 Achievement Score 比 IQ score 低的话,是 learning disability。
但我觉得这是荒谬的。而这种荒谬性,会增大对娃的 stress level。而现代认知神经
医学(非军医学),认为长期的 stress level 是 decrease working memory
capacity 和 increase chance of being neuro-atypical 的重要原因。
当然说明一下这里的 stress level 不是不干事儿,而是说,Do you wanna reading
just for showoff for you Dad? Or you think you can read so you can learn a
lot, and it more or less a fun.
再说明一下这里的 stress level 不是不训练,而是说,有张有驰、文武之道。要给
brain 足够的时间 "idle" / "task-negative"(不是说不干事,而是说放松无压力的
干)。
t******l
发帖数: 10908
163
我如果用体育做一个比方,比如我家娃去年在学校里是 top sprinter,学校的 4x100
接力队。同时是 top long jumper(仅仅是去年,这个随着娃的发育每年都不一样)。
但我家大娃的 endurance running 非常差,经过学校体育课练习,也就是比平均好一
些(平均指去掉那些根本不咋锻炼的娃)。
那军医学就会说,这个 poor endurance 是会造成 sports learning disability 的原
因。
但医学不会这么说,医学会说,这个 poor endurance 是 trait。不是说不需要改进锻
炼,但学校体育课跟大众一起锻炼,达标就行了。实际上如果 over-estimate so
called giftedness,stress your kid too much,最终可能 end up 在 injury /
wear out 到 连 sprinting 也可能都不行了。
t******l
发帖数: 10908
164
但话说回来,成功也是需要冒风险和付出代价的。但成功的利益也是巨大的。
所以推藤校还是推州立,没有绝对的对错好坏,各家自己的选择。But you generally
get what you paid for, unless luckily win the lottery。
而我写这样,也是因为我家过去实在的争议,也鸡飞狗跳过无数次。

:我如果用体育做一个比方,比如我家娃去年在学校里是 top sprinter,学校的
4x100 接力队。同时是 top long jumper(仅仅是去年,这个随着娃的发育每年都不一
样)。
t******l
发帖数: 10908
165
这个网站的(Reverse) Corsi Block Test 好像能 randomly 出现 grid pattern。3
row 4 column 不是每次都适用。
但俺观察以后,发现如果 3 row 4 column 不适用,那 5-5-2 就会适用。比如这个我
觉 3 row 4 column 不行的,改成 5-5-2。(两款必有一款适用)。比如在附件图形里。
Test 过,虽然难一点,但一样可行,可以准确搞定 8 个的长度。
http://www.memorylosstest.com/free-working-memory-tests-online/

【在 t******l 的大作中提到】
: 对于(Reverse) Corsi Block Test 我发现了一个更新的 cheat,可以保证稳稳当当做
: 出 8 个。
: http://www.memorylosstest.com/free-working-memory-tests-online/
: 我的最新 cheat 是,这个格子虽然歪曲过,但实际上大致上是一个 3 row 4 column
: 的矩阵。
: 这样就干脆对矩阵上的格子在内心编号, 1 - 10 , a , b。
: 然后把 8 个黑点切成前后两组,每组四个。对于前面四个黑点,每个出来的时候给编
: 号,然后心里正向 phonological loop,比如 45A8。但是第五个开始就不编号,跟踪
: 位置,同时心里不断 phonological loop 来 refresh 前面的四个编码。然后 8 个全
: 结束后,立即先把后面四个标记上,动作要快省得 working memory fade。之后就可以

t******l
发帖数: 10908
166
我觉得军医学还是有一个反制办法,就是每单次出现的格子 pattern 都不一样,然后
出现格子 pattern 以后立马开始,不给 brain 以 observe pattern and generate
visual cue 的时间。也就是说,purely randomness。
但这就暴露了所谓的智商测试,其实是军医们把精神病院排查弱智的玩意儿,外延一下
就来筛选 GT 班。
或者海蜇头说,purely randomness 就是熵无穷大,热寂。。。生命本身在热寂里有意
义么?

里。

【在 t******l 的大作中提到】
: 这个网站的(Reverse) Corsi Block Test 好像能 randomly 出现 grid pattern。3
: row 4 column 不是每次都适用。
: 但俺观察以后,发现如果 3 row 4 column 不适用,那 5-5-2 就会适用。比如这个我
: 觉 3 row 4 column 不行的,改成 5-5-2。(两款必有一款适用)。比如在附件图形里。
: Test 过,虽然难一点,但一样可行,可以准确搞定 8 个的长度。
: http://www.memorylosstest.com/free-working-memory-tests-online/

t******l
发帖数: 10908
167
我重新又测了一次,不用 numerical encoding,我可以稳定做到 6 个。做 7 个有一
半以上错误率。
超过五个的策略,是先记住前面 4 个或 5 个,不断在心里刷新(rolling D-RAM
refresh)。同时用眼睛的余光看最后两个。然后先把最后两个 instinctively 点上,
然后 refresh 一下 DRAM,consciously reverse order 点前面 4 个或 5 个。
这样做 7 个还挺玄的。所以最终还是 numerical encoding 大幅度胜出。用
numerical encoding 做 8 个几乎轻松不带错。但需要 encoding,建立 schema,不能
出新 pattern 以后立马开始,要给大概一分钟(具体时间不确定)观察格子 pattern
和建立 schema (visual cue)。
http://en.wikipedia.org/wiki/Memory_span#From_simple_span_to_complex_span

【在 t******l 的大作中提到】
: 我觉得军医学还是有一个反制办法,就是每单次出现的格子 pattern 都不一样,然后
: 出现格子 pattern 以后立马开始,不给 brain 以 observe pattern and generate
: visual cue 的时间。也就是说,purely randomness。
: 但这就暴露了所谓的智商测试,其实是军医们把精神病院排查弱智的玩意儿,外延一下
: 就来筛选 GT 班。
: 或者海蜇头说,purely randomness 就是熵无穷大,热寂。。。生命本身在热寂里有意
: 义么?
:
: 里。

t******l
发帖数: 10908
168
其实这个 (Reverse) Corsi Block Test 测试结果其实挺符合 wiki 里的这段的。也就
是说,complex span (encoding) matters much more:
http://en.wikipedia.org/wiki/Memory_span#From_simple_span_to_complex_span
[quote]
From simple span to complex span[edit]
Research in the 1970s has shown that memory span with digits and words is
only weakly related to performance in complex cognitive tasks such as text
comprehension, which are assumed to depend on short-term memory.[28] This
questioned the interpretation of memory span as a measure of the capacity of
a central short-term memory or working memory. Daneman and Carpenter
introduced an extended version of the memory span task which they called
reading span.[29]
The reading span task was the first instance of the family of complex span
tasks, which differ from the traditional simple span tasks by adding a
processing demand to the requirement to remember a list of items. In complex
span tasks encoding of the memory items (e.g., words) alternates with brief
processing episodes (e.g., reading sentences). For example, the operation
span task combines verification of brief mathematical equations such as "2+6
/2 = 5?" with memory for a word or a letter that follows immediately after
each equation.[30] Complex-span tasks have also been shown to be closely
related to many other aspects of complex cognitive performance besides
language comprehension, among other things to measures of fluid intelligence
.[31][32]
[/quote]

pattern

【在 t******l 的大作中提到】
: 我重新又测了一次,不用 numerical encoding,我可以稳定做到 6 个。做 7 个有一
: 半以上错误率。
: 超过五个的策略,是先记住前面 4 个或 5 个,不断在心里刷新(rolling D-RAM
: refresh)。同时用眼睛的余光看最后两个。然后先把最后两个 instinctively 点上,
: 然后 refresh 一下 DRAM,consciously reverse order 点前面 4 个或 5 个。
: 这样做 7 个还挺玄的。所以最终还是 numerical encoding 大幅度胜出。用
: numerical encoding 做 8 个几乎轻松不带错。但需要 encoding,建立 schema,不能
: 出新 pattern 以后立马开始,要给大概一分钟(具体时间不确定)观察格子 pattern
: 和建立 schema (visual cue)。
: http://en.wikipedia.org/wiki/Memory_span#From_simple_span_to_complex_span

t******l
发帖数: 10908
169
或者简单的说,普通娃 (neuro-typical) 的 working memory,大一点小一点、快一点
慢一点、持久一点与否,只要在正常范围内,就是个 trait。而大脑高层的 encoding
会针对自身的 trait 量身定做符合自身特点的 encoding,最终都成为普通娃。
所以我现在觉得,普通娃就真心别去 WISC-V 智商测试选出来的 GT 班了。理由我不说
了,你懂的。

of

【在 t******l 的大作中提到】
: 其实这个 (Reverse) Corsi Block Test 测试结果其实挺符合 wiki 里的这段的。也就
: 是说,complex span (encoding) matters much more:
: http://en.wikipedia.org/wiki/Memory_span#From_simple_span_to_complex_span
: [quote]
: From simple span to complex span[edit]
: Research in the 1970s has shown that memory span with digits and words is
: only weakly related to performance in complex cognitive tasks such as text
: comprehension, which are assumed to depend on short-term memory.[28] This
: questioned the interpretation of memory span as a measure of the capacity of
: a central short-term memory or working memory. Daneman and Carpenter

t******l
发帖数: 10908
170
我觉得你说的 working memory 可能有一定道理。但这可能不是 working memory 本身
(对应于 simple memory-span test),而是 working memory 的 higher-order
encoding(对应于 complex memory-span test)。
而另一个方面,普通人(neuro-typical)在做 working memory / memory-span test
的时候,受 proactive interference(过去经验)on brain encoding level的影响,
上来做 memory-span test 的时候,可能会有一种 “Holy-crap! I cannot even do
it!” 的感觉(因为跟自己平时的 working memory brain encoding 方式大相径庭)。
但是 proactive interference(过去经验)on brain encoding level 对普通人而言
,是一把双刃剑。在两三天以后,也正是因为 new proactive interference(过去经
验)on brain encoding level, so now I think I can dance, and more and more
fluently.

【在 s***n 的大作中提到】
: 是有一些孩子心算不行只能笔算,但这些孩子心算不行,大多是数学水平问题,不是
: working memory问题。真正working memory差的普通孩子,心算笔算都不行。
: 很多孩子倾向笔算只是因为习惯了笔算。GT孩子也一样,心算笔算大多是习惯养成问题
: ,和working memory强弱关系已经不大。我的观察,小娃图省事图快,多心算;大娃稳
: 重些了,会多用笔算。
: 一般认为ADHD的孩子working memory差。但很多ADHD的GT娃,可以用他们gifted的能力
: ,特别是在逻辑推理和大脑处理速度方面的优势,来弥补working memory的不足,所以
: 在表现上,ADHD的HGT孩子反而会显得心算和抢答快,快枪手,但他们的相对正确率(相
: 对其他GT娃)会随着题目难度加大快速下降。
:

相关主题
忍不住上来问问,我娃的数学是啥水平(6岁)?放养妈进来看看
陶天才论天才 (转载)最新研究证明心算AIME大幅度促进心理和情感健康
NIH 证实天才和疯子都源于同一型基因:DARPP-32来看一下各位的观点
进入Parenting版参与讨论
t******l
发帖数: 10908
171
实际上我刚才还 test 了那个最不适应的 2-back task。
我在这玩意儿上没有更多时间 quota 了,于是我实验和观察以及在大脑里 simulate
各种方案 20 分钟。这个 2-back task 没啥 trick,但是 by look and think, I
improved a lot, in matching and performing the task。我目前大概是
instinctively click 和 consciously click 结合,可以做到大概 30% - 50% 的正确
率我想。最重要的是,我不再会 lost focus,能一直做到完成。即使当中 confuse 了
,也能设当前为起点重新 re-start process information and re-gain/keep focused
,而不是直接点退出或者重启。

test
)。

【在 t******l 的大作中提到】
: 我觉得你说的 working memory 可能有一定道理。但这可能不是 working memory 本身
: (对应于 simple memory-span test),而是 working memory 的 higher-order
: encoding(对应于 complex memory-span test)。
: 而另一个方面,普通人(neuro-typical)在做 working memory / memory-span test
: 的时候,受 proactive interference(过去经验)on brain encoding level的影响,
: 上来做 memory-span test 的时候,可能会有一种 “Holy-crap! I cannot even do
: it!” 的感觉(因为跟自己平时的 working memory brain encoding 方式大相径庭)。
: 但是 proactive interference(过去经验)on brain encoding level 对普通人而言
: ,是一把双刃剑。在两三天以后,也正是因为 new proactive interference(过去经
: 验)on brain encoding level, so now I think I can dance, and more and more

T********s
发帖数: 100
172
有些人确实可以一心二用。
比如我家娃爸,能一边唱歌一边写论文,论文写的飞快,歌词也唱的一字不差。
或者一边看电视,一边掏出个小纸片来算算公式,然后电视剧情一点不漏。
我就跟他恰好相反,他一唱歌我就完全不能集中注意力看书,不厌其烦。
他的working memory也不好,常常自己写的paper,推算的公式都记不住。
不知道是不是跟一心二用有关系。

【在 t*******s 的大作中提到】
: 问题是弄YouTube 的同时还在上课,我时常怀疑他思维根本就不在上课。所以提醒过多
: 次这两个事情不能统一时间做,要么专心在YouTube 上,要么认真上课。这也是
: working memory 低的孩子共有的问题,就是executive skill 差,不能高效安排执行
: 多个事情。
:
: 很多

t******l
发帖数: 10908
173
你的第一段是 highway hypothesis,也就是一个是熟能生巧到半自动(Automaticity
)。
你这第二个记公式是 long term memory 而不是 working memory。
当然 highway hypothesis 状态是容易导致遗忘。
http://en.wikipedia.org/wiki/Highway_hypnosis

:有些人确实可以一心二用。
T********s
发帖数: 100
174
你太强了!随便什么话题都能让你找到理论出处。
一边写paper,一边唱歌应该是highway hypothesis,唱歌是个半自动行为。
看电视的时候推导公式有点费解。我觉得很难用unconcious minds 来记住电视剧情,
恐怕更不能用它来推数学公式。可能还是concious minds的快速溜号。

Automaticity

【在 t******l 的大作中提到】
: 你的第一段是 highway hypothesis,也就是一个是熟能生巧到半自动(Automaticity
: )。
: 你这第二个记公式是 long term memory 而不是 working memory。
: 当然 highway hypothesis 状态是容易导致遗忘。
: http://en.wikipedia.org/wiki/Highway_hypnosis
:
: :有些人确实可以一心二用。
: :

t******l
发帖数: 10908
175
韩剧三百集。。。

:你太强了!随便什么话题都能让你找到理论出处。
t******l
发帖数: 10908
176
当然普通人全神贯注也不一定记得住剧情,但对于 “Hyper-韩剧剧情-nia” 确实有可
能就是个 Automaticity 的事。

:韩剧三百集。。。
t******l
发帖数: 10908
177
我现在有个 hypothesis,也许能解释你说的 ADHD 的 HGT 现象。。
这个 hypothesis 是,假设有两个虚构的娃,一个是 hyperlexia,另一个是 neuro-
typical,在除了导致 hyerlexia 的因子以外,两娃的其他智力情商因子都差不多。
那下一贴从这个角度开始。

【在 s***n 的大作中提到】
: 是有一些孩子心算不行只能笔算,但这些孩子心算不行,大多是数学水平问题,不是
: working memory问题。真正working memory差的普通孩子,心算笔算都不行。
: 很多孩子倾向笔算只是因为习惯了笔算。GT孩子也一样,心算笔算大多是习惯养成问题
: ,和working memory强弱关系已经不大。我的观察,小娃图省事图快,多心算;大娃稳
: 重些了,会多用笔算。
: 一般认为ADHD的孩子working memory差。但很多ADHD的GT娃,可以用他们gifted的能力
: ,特别是在逻辑推理和大脑处理速度方面的优势,来弥补working memory的不足,所以
: 在表现上,ADHD的HGT孩子反而会显得心算和抢答快,快枪手,但他们的相对正确率(相
: 对其他GT娃)会随着题目难度加大快速下降。
:

t******l
发帖数: 10908
178
现在这个 hypothesis 问一个 what if:
What if 这两个娃的所有的 working memory / long term memory 等等等等,如果不
考虑 encoding 的 raw power,都一模一样。连 brain 的 total encoding 的 raw
power 也一样,唯一的差别是 the way of encoding (encoding algorithm) is
different。
也就是说,hyperlexia's encoding algorithm focused on lexicon level, 而 neuro
-typical's encoding algorithm balanced between lexicon to pragmatics to
critical thinking.
So, the question is, (in this hypothetical case), what will happen?

【在 t******l 的大作中提到】
: 我现在有个 hypothesis,也许能解释你说的 ADHD 的 HGT 现象。。
: 这个 hypothesis 是,假设有两个虚构的娃,一个是 hyperlexia,另一个是 neuro-
: typical,在除了导致 hyerlexia 的因子以外,两娃的其他智力情商因子都差不多。
: 那下一贴从这个角度开始。

t******l
发帖数: 10908
179
首先考虑小学低年级/中年级,先讨论 long term memory,开始 school learning。
很多人认为 school learning 是从零开始。但 Piaget 老先生的 object-permanence
理论,告诉我们娃在开始 school learning 前面五年的 experience,是一个很大的因
子。而 experience 是双刃剑,既能够 build schema,但同时也能造成 proactive
interference:
http://en.wikipedia.org/wiki/Interference_theory#Proactive_interference
[quote]
Proactive interference[edit]
Proactive interference is the "forgetting [of information] due to
interference from the traces of events or learning that occurred prior to
the materials to be remembered."[5] Proactive interference occurs when, in
any given context, past memories inhibit an individual’s full potential to
retain new memories. It has been hypothesized that forgetting working
memories would be non-existent if not for proactive interference
[/quote]

neuro

【在 t******l 的大作中提到】
: 现在这个 hypothesis 问一个 what if:
: What if 这两个娃的所有的 working memory / long term memory 等等等等,如果不
: 考虑 encoding 的 raw power,都一模一样。连 brain 的 total encoding 的 raw
: power 也一样,唯一的差别是 the way of encoding (encoding algorithm) is
: different。
: 也就是说,hyperlexia's encoding algorithm focused on lexicon level, 而 neuro
: -typical's encoding algorithm balanced between lexicon to pragmatics to
: critical thinking.
: So, the question is, (in this hypothetical case), what will happen?

j*l
发帖数: 448
180
大家来议一议,你希望你娃长大成一个潮水么?
潮水小时是受了啥教育才这么精益求精,谨小慎微,生怕娃们走了弯路?

permanence

【在 t******l 的大作中提到】
: 首先考虑小学低年级/中年级,先讨论 long term memory,开始 school learning。
: 很多人认为 school learning 是从零开始。但 Piaget 老先生的 object-permanence
: 理论,告诉我们娃在开始 school learning 前面五年的 experience,是一个很大的因
: 子。而 experience 是双刃剑,既能够 build schema,但同时也能造成 proactive
: interference:
: http://en.wikipedia.org/wiki/Interference_theory#Proactive_interference
: [quote]
: Proactive interference[edit]
: Proactive interference is the "forgetting [of information] due to
: interference from the traces of events or learning that occurred prior to

相关主题
是不是任我儿子继续迷象棋有明天考AMC 8的吗?
9岁, 如何报名考amc10算不算有数学天分
求科普: 奥数 vs. Math Olympiad vs. Math Count vs. Math circle vs. 超前学数学大家怎么推孩子的弱项?
进入Parenting版参与讨论
t******l
发帖数: 10908
181
我后面还会加塞一个 HyerPragmatixia,也有问题。
这事老实说也不是我乐意干,是学校老师经常来一句 “我丑话都说了,你丫自己看着
办!”。

【在 j*l 的大作中提到】
: 大家来议一议,你希望你娃长大成一个潮水么?
: 潮水小时是受了啥教育才这么精益求精,谨小慎微,生怕娃们走了弯路?
:
: permanence

t******l
发帖数: 10908
182
现在先看 neuro-typical 娃在 learn-to-read 阶段:
neuro-typical 娃的 brain 一看 learn-to-read,WTF,发音和拼写的关系太捉摸不定
了,similarity 加 conflict 一笔糊涂账啊,外加还有词意的干扰。所以 Brain 大佬
说了,encoding 的问题不解决,long term memory 不让上 (proactive interference
)。
于是 working memory 和 encoding 就在小学低年级累死累活n年。但往好处想就是:
第一,这就相当于 working memory 的 recreational sports。强身健体增加 working
memory 的 capacity。
第二,累死累活就是有事干。天天能够有意义地忙着完成大乃给交公粮任务,就不会
mind wandering 到处找小三不是?// run
或者,简单的说,这种 “没有困难创造困难也要上” 的 proactive interference,
有时候也是 brain 的大智若愚的体现。

permanence

【在 t******l 的大作中提到】
: 首先考虑小学低年级/中年级,先讨论 long term memory,开始 school learning。
: 很多人认为 school learning 是从零开始。但 Piaget 老先生的 object-permanence
: 理论,告诉我们娃在开始 school learning 前面五年的 experience,是一个很大的因
: 子。而 experience 是双刃剑,既能够 build schema,但同时也能造成 proactive
: interference:
: http://en.wikipedia.org/wiki/Interference_theory#Proactive_interference
: [quote]
: Proactive interference[edit]
: Proactive interference is the "forgetting [of information] due to
: interference from the traces of events or learning that occurred prior to

t******l
发帖数: 10908
183
而对于那个虚构的 hyperlexia,其 encoding 主要在 lexicon level。也就是说,
context info 的考虑很少。这样的话,反而就不太容易造成 proactive interference
,想记就记。
所以就基本上把前面一贴 inverse 一下就是。

interference
working

【在 t******l 的大作中提到】
: 现在先看 neuro-typical 娃在 learn-to-read 阶段:
: neuro-typical 娃的 brain 一看 learn-to-read,WTF,发音和拼写的关系太捉摸不定
: 了,similarity 加 conflict 一笔糊涂账啊,外加还有词意的干扰。所以 Brain 大佬
: 说了,encoding 的问题不解决,long term memory 不让上 (proactive interference
: )。
: 于是 working memory 和 encoding 就在小学低年级累死累活n年。但往好处想就是:
: 第一,这就相当于 working memory 的 recreational sports。强身健体增加 working
: memory 的 capacity。
: 第二,累死累活就是有事干。天天能够有意义地忙着完成大乃给交公粮任务,就不会
: mind wandering 到处找小三不是?// run

t******l
发帖数: 10908
184
在小学低年级/中年级之后,下面发生的就是 “两次 decay 加当中晃点一次” 的理论。
对于 neuro-typical 娃,decay 点就是 new encoding algorithm release to market
,外加 working memory hardware upgraded。Long term memory 这时终于可以放风出
来乱咬。俗称:“从 learn-to-read 迈入 read-to-learn”。

interference
working

【在 t******l 的大作中提到】
: 现在先看 neuro-typical 娃在 learn-to-read 阶段:
: neuro-typical 娃的 brain 一看 learn-to-read,WTF,发音和拼写的关系太捉摸不定
: 了,similarity 加 conflict 一笔糊涂账啊,外加还有词意的干扰。所以 Brain 大佬
: 说了,encoding 的问题不解决,long term memory 不让上 (proactive interference
: )。
: 于是 working memory 和 encoding 就在小学低年级累死累活n年。但往好处想就是:
: 第一,这就相当于 working memory 的 recreational sports。强身健体增加 working
: memory 的 capacity。
: 第二,累死累活就是有事干。天天能够有意义地忙着完成大乃给交公粮任务,就不会
: mind wandering 到处找小三不是?// run

j*l
发帖数: 448
185
快回那个commoncore~~~~~~~~~~~~~~~~~~~~

【在 t******l 的大作中提到】
: 我后面还会加塞一个 HyerPragmatixia,也有问题。
: 这事老实说也不是我乐意干,是学校老师经常来一句 “我丑话都说了,你丫自己看着
: 办!”。

t******l
发帖数: 10908
186
我现在不是正在这个线程上探索 “手中无 core,心中有 core” 的问题么?

【在 j*l 的大作中提到】
: 快回那个commoncore~~~~~~~~~~~~~~~~~~~~
t******l
发帖数: 10908
187
当然,对纯理论假想的 hyperlexia 的 case,还是 inverse 一下。
但在后面的 decay 点上,可能还会出现一个新问题 -- retroactive interference:
http://en.wikipedia.org/wiki/Interference_theory#Retroactive_interference
[quote]
Retroactive interference[edit]
Retroactive interference (RI) is a phenomenon that occurs when newly learned
information interferes with and impedes the recall of previously learned
information.[19] RI is a result of decreased recall of the primary studied
functions due to the learning and recall of succeeding functions.
[/quote]

论。
market

【在 t******l 的大作中提到】
: 在小学低年级/中年级之后,下面发生的就是 “两次 decay 加当中晃点一次” 的理论。
: 对于 neuro-typical 娃,decay 点就是 new encoding algorithm release to market
: ,外加 working memory hardware upgraded。Long term memory 这时终于可以放风出
: 来乱咬。俗称:“从 learn-to-read 迈入 read-to-learn”。
:
: interference
: working

t******l
发帖数: 10908
188
或者进一步解释就是,lower-order encoding 虽然简便快捷,但是缺乏 structure 和
hierarchy,导致在 problem size 不断增大的时候,发生更多的相似和冲突的问题。
或者简单的马工学比方,就是 hash table 的 hashing function 过于简化,虽然计算
hash index 快,但是 hash table 装到半满的时候,一大堆的层出不穷的 hashing
index conflict。最后为了防止用户不耐烦直接按 ctrl-C 了,一旦 time out 就不再
resolve index conflict 直接往 table 里闭着眼睛写,于是就把过去的一个数据给
覆盖掉了,俗称:retroactive interference induced unlearning。

learned

【在 t******l 的大作中提到】
: 当然,对纯理论假想的 hyperlexia 的 case,还是 inverse 一下。
: 但在后面的 decay 点上,可能还会出现一个新问题 -- retroactive interference:
: http://en.wikipedia.org/wiki/Interference_theory#Retroactive_interference
: [quote]
: Retroactive interference[edit]
: Retroactive interference (RI) is a phenomenon that occurs when newly learned
: information interferes with and impedes the recall of previously learned
: information.[19] RI is a result of decreased recall of the primary studied
: functions due to the learning and recall of succeeding functions.
: [/quote]

t******l
发帖数: 10908
189
这个 proactive interference vs retroactive interference,其实可以拿前面的 “
用 numeral indexing 来解 (Reverse) Corsi Block Task/Test” 这个生动的例子来
做比方。(reverse corsi block online test link 如下):
http://www.memorylosstest.com/free-working-memory-tests-online/
下面用马工学的角度类比,假设开始时是整整齐齐的 3x4 (3 row, 4 column) 对齐的
布置(online test 不是整齐对准的,FYI)。那在佳能无敌兔的 10M 的 bitmap 进来
以后,有两种 encoding 算法:
(1)Bitmap=>Binning indexing algorithm(就是画格子):
http://en.wikipedia.org/wiki/Bitmap_index#Binning
(2)Lossless Straight-line RLE (Run-length encoding) indexing algorithm :
http://en.wikipedia.org/wiki/Run-length_encoding
这个时候,Bitmap=>Binning algorithm 最快,也不需要学。(当然人眼无法 binning
12 个位置,working memory 已经超载了,这里是假设)。
(下贴待续)。
t******l
发帖数: 10908
190
然后突然出现以下附件里的这种歪曲不对准的格子(图上已经编号,给了 visual cue
,但测试是是空白格)。比如对于格子 2, 3, 7。
(下贴待续)
相关主题
少年智力开发觉得美帝数学进度慢的进来看一下
[转载] 为什么说usamo 简单分数应用题
二年级女儿的report我招,我是猪
进入Parenting版参与讨论
t******l
发帖数: 10908
191
在这种情况下,前两次尝试的时候,两种算法可能都太 2, 3, 7 上犯错,因为太靠近
了。但背后的原因是不一样的:
binning algorithm 是因为过于靠近(相似性和矛盾性),以至于 binning 到同一个
index 里,导致后面的覆盖前面的数据,属于 retroactive interference。
而 straight RLE algorithm 是因为 straight line match 不上格子,导致过去非常
成功的算法本身,在斗争动态新形势下 confused,属于 proactive interference。
(下贴待续)

cue

【在 t******l 的大作中提到】
: 然后突然出现以下附件里的这种歪曲不对准的格子(图上已经编号,给了 visual cue
: ,但测试是是空白格)。比如对于格子 2, 3, 7。
: (下贴待续)

t******l
发帖数: 10908
192
而继续学习的时候,binning 算法比较麻烦,如果把 binning 变成更 fine的话,确实
能防止 conflict index,但是更 fine 的 binning require more memory,可能导致
working memory overflow。两难问题。
而 RLE 算法就不会遇到两难推理,因为 RLE 可以把 straight running line 改成
curved running line,形成 5-5-2 pattern。而根据 Piaget 老先生的 object
permanence on moving object 理论,curved running line 可以 preprocess by
object permanence 而 encoding 在 visual-spatial sketchpad 里,只要格子位置不
变,curved line encoding 不用改。
另外 RLE 算法改成 curved running line 后,实际上更多的使用了 visual-spatial
sketchpad(保存 curved running line -- visual cue,还要 match / counting
following visual cue),但这也不造成 stress(反正 visual-spatial sketchpad
闲着也是闲着)。而根据 “自愿交足大乃公粮,就自然抵制 mind-wandering 出去找
小三” 的理论。这也降低 mind-wandering 的概率不是。// run



【在 t******l 的大作中提到】
: 在这种情况下,前两次尝试的时候,两种算法可能都太 2, 3, 7 上犯错,因为太靠近
: 了。但背后的原因是不一样的:
: binning algorithm 是因为过于靠近(相似性和矛盾性),以至于 binning 到同一个
: index 里,导致后面的覆盖前面的数据,属于 retroactive interference。
: 而 straight RLE algorithm 是因为 straight line match 不上格子,导致过去非常
: 成功的算法本身,在斗争动态新形势下 confused,属于 proactive interference。
: (下贴待续)
:
: cue

t******l
发帖数: 10908
193
现在加塞一个假想的理论上可能出现的 HyerPragmatixia disorder。
HyerPragmatixia disorder,顾名思义,就是过去的 pragmatics (上下文相关语境)
encoding 太强大,导致造成 proactive interference,给 lexicon 学习带来困难,
导致当前学习阶段看上去就像 dyslexia 一样。
t******l
发帖数: 10908
194
但如果仔细分析,hyperpragmatixia disorder 跟 dyslexia 还是会有差别,表现在:
hyperpragmatixia disorder 在刚开始学 written lexicon 的时候,呈现正常。但当
学习到一半,建立各种 written lexicon 联系的时候,因为过去的 proactive
interference (这里是 context clue)的影响,出现困难。
在学习阶段,hyperpragmatixia disorder 通常在有 meaningful context 的情况下,
performance 比没有 meaningful context 的情况要好。
在学习阶段,hyperpragmatixia disorder 通常在 receptive written language 的
performance 要比 expressive written language 更好一些。

【在 t******l 的大作中提到】
: 现在加塞一个假想的理论上可能出现的 HyerPragmatixia disorder。
: HyerPragmatixia disorder,顾名思义,就是过去的 pragmatics (上下文相关语境)
: encoding 太强大,导致造成 proactive interference,给 lexicon 学习带来困难,
: 导致当前学习阶段看上去就像 dyslexia 一样。

t******l
发帖数: 10908
195
下面是 DSM-999 对 hyperpragmatixia disorder 的治疗建议:
(1)首先是接受事实,这是个 developmental disorder / issue。帮助娃 manage it
,但不可能 get rid of it。(不要试图一脚踹飞上帝)。
(2)尽管 hyperpragmatixia disorder 学习过程中有 written lexicon 的问题,这
不等于就可以跳过普通娃 written lexicon 的阶段。hyperpragmatixia disorder 应
该 receive same amount / format of written lexicon education,即使 written
lexicon 的成绩较低一些。建议就是只要能及格,走普通班道路,接受现状。一般不用
Special Education,也不要妄想 Gifted Education。
(3)在课后,hyperpragmatixia 应该适当利用在 context / pragmatic 上的优势,
选择上更优先 contents / information / meaning 胜于 lexile text complexity。
以创建兴趣,增加 mental focus,降低 stress,在与蒋军的实战斗争中学习。
(4)由于 “在与蒋军的实战斗争中学习” 的当前效率相对比军校书本较低,
hyperpragmatixia 应当花更多的小时数,同时也还需要保证 free play / brain idle
的小时数。这样保证 decay 点到来的时候,已经准备就绪。所以于此冲突的才艺兴趣
,可以暂时让位一下。

【在 t******l 的大作中提到】
: 但如果仔细分析,hyperpragmatixia disorder 跟 dyslexia 还是会有差别,表现在:
: hyperpragmatixia disorder 在刚开始学 written lexicon 的时候,呈现正常。但当
: 学习到一半,建立各种 written lexicon 联系的时候,因为过去的 proactive
: interference (这里是 context clue)的影响,出现困难。
: 在学习阶段,hyperpragmatixia disorder 通常在有 meaningful context 的情况下,
: performance 比没有 meaningful context 的情况要好。
: 在学习阶段,hyperpragmatixia disorder 通常在 receptive written language 的
: performance 要比 expressive written language 更好一些。

t******l
发帖数: 10908
196
好了,俺现在可以告一段落了。
t******l
发帖数: 10908
197
我小时候数学一屁股红叉,所以我觉得我受的是 “生于忧患” 教育。
而我家领导过去一直醉心于各种 “三十六计招招制敌”,以及高压政策,妄图能把娃
的红叉消灭在源头,我确实担心会造成 de facto “decay 于安乐” 教育。
所以古人云:这看起来不是 “谨小慎微” 的差别,这是 “南辕 vs 北辙” 之路线差
别。

【在 j*l 的大作中提到】
: 大家来议一议,你希望你娃长大成一个潮水么?
: 潮水小时是受了啥教育才这么精益求精,谨小慎微,生怕娃们走了弯路?
:
: permanence

j*l
发帖数: 448
198
前面忒长看不进去,就这句简短明白,你承认是你ld把你逼这样的,
不是小时decay成这样的,还要夸耀说你现在的性格不是来自你小时受你家人的培养
而是你现在的奋斗。基于这个假说,你倾向放养,但你ld不同意,这也是自洽的。
看来你ld就是pushymama群的入口,快给个2维码介绍我们加入吧
就算人多扫不进去,我们看看名字也行啊

【在 t******l 的大作中提到】
: 我小时候数学一屁股红叉,所以我觉得我受的是 “生于忧患” 教育。
: 而我家领导过去一直醉心于各种 “三十六计招招制敌”,以及高压政策,妄图能把娃
: 的红叉消灭在源头,我确实担心会造成 de facto “decay 于安乐” 教育。
: 所以古人云:这看起来不是 “谨小慎微” 的差别,这是 “南辕 vs 北辙” 之路线差
: 别。

t******l
发帖数: 10908
199
实话实说我 ld 在平均水平里算好的。真正狠角色的话,另一方一般就不管了。当然真
正狠角色自己倒是不一定知道。
另外这跟放养无关,这是 “把娃的人生看成自己的人生” 的细细的红线。不过说实话
真的过了那条红线很远的,自己无法知道的,a point if no return。真心的说。

:前面忒长看不进去,就这句简短明白,你承认是你ld把你逼这样的,
t******l
发帖数: 10908
200
另外你家娃这个年龄,你根本不需要扫微信群的二维码。你问问你家娃就知道了,娃娃
之间一来会一传十十传百,二来就算不传,empathy 和 theory of mind 其实大家都知
道。

:前面忒长看不进去,就这句简短明白,你承认是你ld把你逼这样的,
相关主题
老师说我女儿是gifted kid (转载)NIH 证实天才和疯子都源于同一型基因:DARPP-32
忍不住上来问问,我娃的数学是啥水平(6岁)?放养妈进来看看
陶天才论天才 (转载)最新研究证明心算AIME大幅度促进心理和情感健康
进入Parenting版参与讨论
t******l
发帖数: 10908
201
我写这么多其实是 summarize the past thinking,从三年级到六年级,这四年。也就
是说,我基本可以洗手不干了(对大娃而言)。
大部分普通人的,不会把实际的 planning 往网上写,sense of privacy。而树洞的话
,一般是 explosive,也不会写成完整的。
(也就是 autobiographical memory 是通常心理,hallucination 不是)。
I think this is "THE END" 了。

:前面忒长看不进去,就这句简短明白,你承认是你ld把你逼这样的,
t******l
发帖数: 10908
202
我补充一句的话,让我不解的是,举个例子比如 “老实大量读经”,这种 “推 neuro-
atypical” 是图个啥。
neuro-atypical 对将来婚姻家庭都是问题,而工作上算得快一点慢一点都不是大问题。
或者这么说吧,高斯是伟人,但有多少人愿意娶高斯当老婆吧。
从这点来说,上海人不填北清,至少是 neuro-typical。

:前面忒长看不进去,就这句简短明白,你承认是你ld把你逼这样的,
t******l
发帖数: 10908
203
前面我写的确实又臭又长。。。既然万圣节快到了,我给你的短的版本吧:
我觉得我们小时候没有 钢琴 / flash-card-老实大量读经背单词 等等那些,不像现在
被推得这么狠,至少不练出一身 musician's focal mental dystonia 这种潜在精神疾
病,还是挺幸运的。
===============================================================
更短的版本是:
IQ > 130 没啥,但是 GT 班里的 IQ > 130 就算了。因为只有有精神问题导致不能
fit school 的,才会去测 IQ 然后报名 GT 班。(或者就算小孩没问题,大人有问题
的话,on-set 迟早的事)。
===============================================================
再短的版本是:
我娃去年说,好像周围 AMC 8 比她分数高的,很大比例看起来总是让人觉得怪怪的。
当然也可能是她不想去 math club 的借口。
===============================================================
祝万圣节快乐。:-)

【在 j*l 的大作中提到】
: 前面忒长看不进去,就这句简短明白,你承认是你ld把你逼这样的,
: 不是小时decay成这样的,还要夸耀说你现在的性格不是来自你小时受你家人的培养
: 而是你现在的奋斗。基于这个假说,你倾向放养,但你ld不同意,这也是自洽的。
: 看来你ld就是pushymama群的入口,快给个2维码介绍我们加入吧
: 就算人多扫不进去,我们看看名字也行啊

t******l
发帖数: 10908
204
我的回答政治不正确,但我还是最好回答一次 summarize 一下。
neuro-typical 的,至少对于高中以前,在没有迫切需要的环境下,不会太热心于重复
训练(over-training brain's psychomotor function)。
而从心理学研究的统计数据思考,我们以前周围 straight-A 这些高分群体里,超过一
半可能是 neuro-atypical 的。这个其实符合过去高中大学的日常经验。
而现在某一型心理学猜想是,其实是高智商(high IQ)保护和防止这型 neuro-
atypical 的进入 psychosis。只要不进入 psychosis 就至少是医学上的正常人。
这个猜想能解释为啥高智商群体下一代 developmental issue 更多,比如流传的理论
数学系啥的。有一个可能的环境因子就是一代一代自觉或不自觉地 over-training,并
且是从小从起跑线开始 over-training 下一代的 psychomotor function,最后 high
IQ 也挡不住 psychosis 的 onset。当然这有很大运气成分,但人群概率趋势可能是明
显的。
而这个确实是 BBS 上匿名争论最说明问题,看看有多少人 strongly defend their
idea of ”heavily psychomotor training 从娃娃抓起“。具体是啥理由其实不重要
了。
这也可能可以部分解释 high IQ 的 teen 里,major depression 的病例在 on-set 以
前,大伙儿都说是怎么怎么阳光,给人感觉是突然出现的。按照心理学的 high IQ 最
终没挡住 psychosis on-set 的假说,这个可能的确是突然出现的,可能也有很大的偶
然因子(解释 major depression 的 “传染性“ 现象)。
summarize 一下,就此板门店结束这个为期四年的话题的探讨了。

:neuro-typical 是啥样的?
t******l
发帖数: 10908
205
而另一方面,凡事都有两面性。psychomotor 的训练的时效比,往往也跟 IQ 有关。
这个可能也能解释为啥 high IQ 群体里,至少在讨论时,有另一型对 psychomotor
training 会发生一种类似 phobia 的反应。
这是因为毕竟概率事件无法对单个个体预测,而 psychosis 一旦 onset 多半不可逆,
只能减轻症状。所以留多少 cushion room 的问题,或者说就是在 “推 eliteness”
vs “推 neuro-typical“ 之间折中在那个点,也确实是一个不会有定论的话题。

:neuro-typical 是啥样的?
t******l
发帖数: 10908
206
这个 "psychomotor addict" vs "psychomotor phobia" 之争,最终白刃战于 "小学运
动学难题(比如追击问题)" 上。(所以我也总结一下)。
"psychomotor addict" 对小学运动学难题(比如追击问题)的热衷,表现在热情总结
"小学追击问题之三十六计招招制敌"。
而 "psychomotor phobia" 出于对 “psychomotor => psychosis” 的担心,追随 “
natural math learning” 的理念,认为 point-slope 的概念才是大多数 neuro-
typical 的普通人 naturally-mathematicize human-brain's perception of the
world around it 的道路。。。同时 Piaget 的 object-permanence 和 conservation
说:“Please test the readiness of kid's perception, help them establish
perception, before mathematicize it”。。。(当然后面的故事,是 Piaget 不幸
被 "psychomotor addict" 从背后扎了一刀,送到急症室里去了)。
但是我觉得最最发人深省的,不管 BBS 上对 psychomotor GT 班多么热衷,事实上在
我们过去身边的初中高中(道听途说的不算),上下几个年级,没有哪怕就一个,能沿
着 "小学追击问题之三十六计招招制敌" 的 psychomotor 之路,打入 ChnPhO
National Top 5000 的(要知道运动学可是 PhO 的基础的基础)。
但是插一句,另一方面,在同样的高考 stress(如果不是更多 stress)的情况下,我
们那时的 top students 看起来比现在更 mentally happier / healthier(至少相对
著名的 Palo Alto High 而言)。。。当然一个原因可能是那时候既没有钢琴也没人追
GT,竞赛都是玩票。(侬晓得上海宁基本认为高分学生填北大清华、不填家门口州立
的,是戆大)。。。



【在 t******l 的大作中提到】
: 而另一方面,凡事都有两面性。psychomotor 的训练的时效比,往往也跟 IQ 有关。
: 这个可能也能解释为啥 high IQ 群体里,至少在讨论时,有另一型对 psychomotor
: training 会发生一种类似 phobia 的反应。
: 这是因为毕竟概率事件无法对单个个体预测,而 psychosis 一旦 onset 多半不可逆,
: 只能减轻症状。所以留多少 cushion room 的问题,或者说就是在 “推 eliteness”
: vs “推 neuro-typical“ 之间折中在那个点,也确实是一个不会有定论的话题。
:
: :neuro-typical 是啥样的?

t******l
发帖数: 10908
207
而最后的三句话总结就是:
(1)working memory / 九九乘法表 等等之类的,都只是表象。核心问题是
psychomotor。
(2)每个具体个人的 psychomotor 能力都是有 limit 的,不管是不是主观上承认(
当然很难事前知道 limit 具体在哪里)。而当接近这个 limit 的时候,会发生的,要
么就是从 psychomotor 进入 psychosis,要么就是停止 psychomotor 的进一步发展。
(当然运气超级好的另说,那个相当于中奖,无法事前计划)。而到那天的时候,当事
人也不一定能够有意识的进行选择 psychosis 还是停止发展。
(3)而 "psychomotor phobia" 置 psychomotor 之 “快准狠” 在成绩上、在追藤时
的巨大优势不顾,而像个沙比一样热衷于 “naturally conceptualize / vocalize /
verbalize / mathematicize our brain's natural perception of the world around
it”(简称:快糙猛),也不一定就真的是一群疯子。
OVER

【在 t******l 的大作中提到】
: 这个 "psychomotor addict" vs "psychomotor phobia" 之争,最终白刃战于 "小学运
: 动学难题(比如追击问题)" 上。(所以我也总结一下)。
: "psychomotor addict" 对小学运动学难题(比如追击问题)的热衷,表现在热情总结
: "小学追击问题之三十六计招招制敌"。
: 而 "psychomotor phobia" 出于对 “psychomotor => psychosis” 的担心,追随 “
: natural math learning” 的理念,认为 point-slope 的概念才是大多数 neuro-
: typical 的普通人 naturally-mathematicize human-brain's perception of the
: world around it 的道路。。。同时 Piaget 的 object-permanence 和 conservation
: 说:“Please test the readiness of kid's perception, help them establish
: perception, before mathematicize it”。。。(当然后面的故事,是 Piaget 不幸

t******l
发帖数: 10908
208
如果再加一贴的话,我觉得也许我能够 empathy 类似 Palo Alto High 的 stressful
的环境。
就看看我这四年好了,我觉得我自己 literally in the status of psychomotor
agitation。而这个 trigger 的因子非常简单,任何时候只要娃成绩不那么好,就会来
个声音 “you need more psychomotor training, 比如 flash card / working sheet
/ etc / etc”。其实我都已经形成半条件反射。
并且更重要的是,我的 stress 其实是源于 empathy kids stress。我估计很多可能都
放弃 empathy 了,也就是说,干脆不管了。
我一开始以为是我周围小环境的问题,所以我在 bbs 和 pushy mama 微信群上 debate
,发现这个 bbs,尤其 pushy mama 微信群,对 psychomotor training 更热衷。不仅
如此,我还问了我娃其他家的情况,发现我家其实算相对而言 nice 的环境。吼吼。
要知道娃娃们没有我自己过去的经验(我毕竟可以把过去经历往桌上啪叽一拍,BBS 上
没用但是现实生活里还是最后一道釜山防线,钢钢的)。但是我不知道娃娃们没有过去
的经验,如何问答下面这两个问题或者类似的问题。
(1)这类的一触即发加强型 "psychomotor training",是不是真的有值得的人生价值
(指中产开心人生过日子,而不是说进个藤校)。
(2)将来就公司里混个日子,有没有必要把 math 啥的搞成那么强大?
当然我自己有釜山防线,我其实无所谓。我每次被打到釜山后,回头来个仁川登陆,再
打回板门店就是了。时间长了习惯了,就自然解决我自己这个 psychomotor agitation
。但是娃娃就要看运气了(这也就是我当中干预,惹了自己一身骚的原因)。

【在 t******l 的大作中提到】
: 而最后的三句话总结就是:
: (1)working memory / 九九乘法表 等等之类的,都只是表象。核心问题是
: psychomotor。
: (2)每个具体个人的 psychomotor 能力都是有 limit 的,不管是不是主观上承认(
: 当然很难事前知道 limit 具体在哪里)。而当接近这个 limit 的时候,会发生的,要
: 么就是从 psychomotor 进入 psychosis,要么就是停止 psychomotor 的进一步发展。
: (当然运气超级好的另说,那个相当于中奖,无法事前计划)。而到那天的时候,当事
: 人也不一定能够有意识的进行选择 psychosis 还是停止发展。
: (3)而 "psychomotor phobia" 置 psychomotor 之 “快准狠” 在成绩上、在追藤时
: 的巨大优势不顾,而像个沙比一样热衷于 “naturally conceptualize / vocalize /

t******l
发帖数: 10908
209
类比 musician's hand-movement psychomotor over-training 增大 musician's
focal hand dystonia 可能,我猜测 pushy mama's phonological-loop psychomotor
over-training 增大 psychosis 的可能(或者增加 language dysfunction 的可能)
,我找到一个间接支持:
[quote]
Psychiatrist Tim Crow has argued that schizophrenia may be the evolutionary
price we pay for a left brain hemisphere specialization for language.[184]
Since psychosis is associated with greater levels of right brain hemisphere
activation and a reduction in the usual left brain hemisphere dominance, our
language abilities may have evolved at the cost of causing schizophrenia
when this system breaks down.
https://en.wikipedia.org/wiki/Causes_of_schizophrenia#Other_views
[/quote]
也就是说,如果以上假设有其合理因素的话,pushy mama's phonological-loop
psychomotor over-training 增大对 left brain hemisphere (specialization for
language) 的 stress,这样导致 greater levels of right brain hemisphere
activation and a reduction in the usual left brain hemisphere dominance,导
致增加 psychosis 的概率。。。当然另一种情况是 left brain hemisphere 抵抗这种
pushy mama stress,这样不造成 psychosis,但代价是可能会造成某型 language
dysfunction。
当然以上都是基于 brain 本身并不 perfect。反过来说就是,只要命够硬就啥都不怕
。。。当然命这玩意儿不好抄就是了。
t******l
发帖数: 10908
210
如果上面这个假设有一定道理的话,那基于 Piaget 的 extended-object-permanence,
e.g. point-slope,的 “natural math learning”,意义在于能 largely balance
left vs right brain load on mathematics tasks。。。所以这个 “natural math
learning” 的意义并不在于 math eliteness,而是在于 better maintain overall
mental health, just in case one's brain is not perfect。
http://www.diffen.com/difference/Left_Brain_vs_Right_Brain
但最后的问题是为啥 pushy mama 们 collectively hate the "point-slope"?
我觉得可能的答案是 selection bias。。。也就是在统计上,pushy mama 们 perform
not very good on math, 并且 largely unlearned most advanced math after
university graduation,所以她们 unable feel the downside of overly
phonological-loop psychomoto training。(偶尔有几个 math elite pushy mama 可
能也是 perfect brain 所以无所谓,或者 social 的原因也不说真话)。。。这个假
说成立的间接证据是,统计上大部分 dad,presumed much superior on math,统计上
对重复训练口决(phonological-loop psychomoto),远远没有 pushy mama 那么热心
。。。而这个问题是在于 pushy mama's kids could be much more advanced on math
(compared to themselves),并且 nobody know the brain is perfect or not。
我觉得这篇应该可以 conclude 这个 hypothesis 和 autobiographical memory。

psychomotor

【在 t******l 的大作中提到】
: 类比 musician's hand-movement psychomotor over-training 增大 musician's
: focal hand dystonia 可能,我猜测 pushy mama's phonological-loop psychomotor
: over-training 增大 psychosis 的可能(或者增加 language dysfunction 的可能)
: ,我找到一个间接支持:
: [quote]
: Psychiatrist Tim Crow has argued that schizophrenia may be the evolutionary
: price we pay for a left brain hemisphere specialization for language.[184]
: Since psychosis is associated with greater levels of right brain hemisphere
: activation and a reduction in the usual left brain hemisphere dominance, our
: language abilities may have evolved at the cost of causing schizophrenia

1 (共1页)
进入Parenting版参与讨论
相关主题
二年级女儿的report放养妈进来看看
觉得美帝数学进度慢的进来看一下最新研究证明心算AIME大幅度促进心理和情感健康
分数应用题来看一下各位的观点
我招,我是猪是不是任我儿子继续迷象棋
老师说我女儿是gifted kid (转载)9岁, 如何报名考amc10
忍不住上来问问,我娃的数学是啥水平(6岁)?求科普: 奥数 vs. Math Olympiad vs. Math Count vs. Math circle vs. 超前学数学
陶天才论天才 (转载)有明天考AMC 8的吗?
NIH 证实天才和疯子都源于同一型基因:DARPP-32算不算有数学天分
相关话题的讨论汇总
话题: memory话题: chunking话题: working话题: gt话题: temporal